You are on page 1of 305

UNIVERSITY OF CALICUT

SCHOOL OF DISTANCE EDUCATION


Self Learning Material
B.Sc. Mathematics
Fifth Semester
(2019 Admission Onwards)

MTS5 B06 : BASIC ANALYSIS


ii

UNIVERSITY OF CALICUT

SCHOOL OF DISTANCE EDUCATION


Self Learning Material
B.Sc. Mathematics (Fifth Semester)
(2019 Admission Onwards)
MTS5 B06 : BASIC ANALYSIS

Chapters 1 to 14 Prepared by:


Dr.Bijumon R., Associate Professor and Head,
Department of Mathematics,
Mahatma Gandhi College, Iritty
Keezhur P.O., Kannur Dt.

Chapters 15 to 18 Prepared by:


Jaison Joseph, Associate Professor,
Department of Mathematics
St. Joseph College, Devagiri, Calicut.

Scrutinized by:
Dr.Vinod Kumar P., Associate Professor,
Department of Mathematics,
Thunchan Memorial Government College, Tirur
Malappuram Dt.
Contents

1 Finite and Infinite Sets 1


1.1 Enumerating a Finite Nonempty Set . . . . . . . . . . . . 2
1.2 Countable Sets . . . . . . . . . . . . . . . . . . . . . . . . 8
1.3 Enumerating a Denumerable Set . . . . . . . . . . . . . . 9

2 The Algebraic Properties of R 24


2.1 Binary Operation . . . . . . . . . . . . . . . . . . . . . . . 25
2.2 Rational and Irrational Numbers . . . . . . . . . . . . . . 34

3 The Order Properties of R 42


3.1 The Order Properties of R . . . . . . . . . . . . . . . . . . 43
3.2 Solving Inequalities . . . . . . . . . . . . . . . . . . . . . . 52

4 Absolute Value and the Real Line 63


4.1 Absolute Value and the Real Line . . . . . . . . . . . . . . 64

iii
iv CONTENTS

5 The Completeness Property of R 81


5.1 Suprema and Infima . . . . . . . . . . . . . . . . . . . . . 82

6 Applications of the Supremum Property 94


6.1 Applications of the Supremum Property . . . . . . . . . . 94

7 Intervals 116
7.1 Intervals . . . . . . . . . . . . . . . . . . . . . . . . . . . 116

8 Sequences of Real Numbers 130


8.1 Sequences and their Limits . . . . . . . . . . . . . . . . . 130

9 Limit Theorems 151


9.1 Limit Theorems . . . . . . . . . . . . . . . . . . . . . . . . 151

10 Monotone Sequences 169


10.1 Monotone Sequences . . . . . . . . . . . . . . . . . . . . . 169

11 Subsequences and the Bolzano-Weierstrass Theorem 184


11.1 Subsequences . . . . . . . . . . . . . . . . . . . . . . . . . 184

12 The Cauchy Criterion 197


12.1 Cauchy sequence . . . . . . . . . . . . . . . . . . . . . . . 197

13 Properly Divergent Sequences 211


13.1 Properly Divergent Sequences . . . . . . . . . . . . . . . . 211

14 Open and Closed Sets in R 218


14.1 Neighborhood . . . . . . . . . . . . . . . . . . . . . . . . . 218
15 Complex Numbers and their Properties 237
15.1 Complex Plane . . . . . . . . . . . . . . . . . . . . . . . . 242
15.2 Polar Form of Complex Numbers . . . . . . . . . . . . . . 246
15.2.1 Principal nth Root . . . . . . . . . . . . . . . . . . 251
15.3 Sets of Points in the Complex Plane . . . . . . . . . . . . 252
15.3.1 Circles . . . . . . . . . . . . . . . . . . . . . . . . . 252

16 Complex Functions 259


16.1 Real and Imaginary Parts . . . . . . . . . . . . . . . . . . 261

17 Complex Functions as Mappings 264


17.1 Mappings . . . . . . . . . . . . . . . . . . . . . . . . . . . 265
17.2 Linear Mappings: . . . . . . . . . . . . . . . . . . . . . . 271
17.3 Special Power Functions . . . . . . . . . . . . . . . . . . . 275

18 Limit of a Function of Complex Variable 280


18.1 Continuity . . . . . . . . . . . . . . . . . . . . . . . . . . . 290
18.2 Branches . . . . . . . . . . . . . . . . . . . . . . . . . . . 292

Syllabus 295
Chapter 1
Finite and Infinite Sets

In this chapter we discuss finite and infinite sets. N denotes the set of
natural numbers and R denotes the set of real numbers.

Definition 1

a) The empty set ∅ is said to have 0 elements.

b) If n ∈ N, a set S is said to have n elements if there exists a bijection


from the set Nn = {1, 2, ..., n} onto S.

c) A set S is said to be finite if it is either empty or it has n elements


for some n ∈ N.

d) A set S is said to be infinite if it is not finite.

Theorem 1 A set S has n elements if and only if there is a bijection


from S onto the set {1, 2, ..., n}.

Proof

1
2 Chapter 1. Finite and Infinite Sets

To prove this result, we use the fact that inverse of a bijection is a


bijection.

Using Definition 1(b), S has n elements if and only if there exists a


bijection f from the set Nn = {1, 2, ..., n} onto S. i.e., if and only if
there exists bijection f −1 from the set S onto Nn = {1, 2, ..., n}.

1.1 Enumerating a Finite Nonempty Set

If S is a nonempty finite set having n elements, then by Definition 1(b)


there is a bijection f : Nn = {1, 2, ..., n} → S so that every element
s in S is of the unique form s = f (m) for some m ∈ Nn . Hence the
elements in a nonempty finite set S can be displayed by the following
enumeration:
f (1), f (2), ..., f (n).

Theorem 2 A set S1 has n elements if and only if there is a bijection


from S1 onto a set S2 that has n elements.

Proof By Theorem 1, S1 has n elements if and only if there is a bijection


from S1 onto the set{1, 2, ..., n}.

Hence if S1 has n elements, then there exists a bijection of S1 onto


{1, 2, ..., n}. Now take S2 = {1, 2, ..., n}. Noting that S2 has n ele-
ments, we have proved that there exists a bijection from S1 onto a set
S2 that has n elements.

Conversely, suppose f is a bijection of S1 onto S2 that has n elements.


Since S2 has n elements, by Theorem 1, there is a bijection, say, g from
S2 onto Nn = {1, 2, ..., n}. Being the composition of bijections, g ◦ f
is a bijection of S1 onto Nn , so (by Theorem 1, it follows) that S1 has n
1.1 Enumerating a Finite Nonempty Set 3

elements. This completes the proof.

Proof of the following result is left as an exercise.

Theorem 3 A set T1 is finite if and only if there is a bijection from T1


onto a set T2 that is finite.

Theorem 4 (a) Let m, n ∈ N with m ≤ n. Then there exists an


injection from Nm into Nn .

(b) (Pigeonhole Principle) Let m, n ∈ N with m > n. Then there


does not exist an injection from Nm into Nn .

Proof (a) Let f : Nm → Nn be defined by f (k) = k for k ∈ Nm ⊆ Nn .


Then f is injective as f (k1 ) = f (k2 ) ⇒ k1 = k2 .

(b) The proof is applying Mathematical Induction on the value of n.


First let n = 1. If g is any map of Nm (m > 1) into N1 , then it is clear
that g(1) = · · · = g(m) = 1, so that g is not injective.

Now as induction hypothesis, assume the result is true for n = k. i.e.,


assume that Nk (k > 1) is such that no mapping of Nm into Nk (m > k)
is an injection; we will show that no mapping from Nm into Nk+1 (m >
k + 1) is an injection.

Two cases arise: either h(Nm ) ⊆ Nk or h(Nm ) 6⊆ Nk

Case 1) If the range h(Nm ) of h is contained in Nk ⊆ Nk+1 , then, by


the induction hypothesis, the mapping h is not injective as a map into
Nk , and hence h is not injective as a map into Nk+1 .

Case 2) Suppose the range h(Nm ) of h is not contained in Nk .

Case 2a) If more than one element in Nm is mapped into the natural
number k + 1, then h is certainly not injective.
4 Chapter 1. Finite and Infinite Sets

Case 2b) Suppose one element in Nm , say p ∈ Nm , is mapped into


the natural number k + 1 by h. We now define h1 : Nm−1 → Nk by
(
h(q) if q = 1, . . . , p − 1
h1 (q) =
h(q + 1) if q = p, . . . , m − 1.

It follows from the induction hypothesis (noting m − 1 > k and h1


is a mapping of Nm−1 into Nk (m − 1 > k) ) that h1 is not injective.
Hence at least one element of {1, . . . , p − 1, p + 1, m} is mapped by h
into more than one element in Nk ⊆ Nk+1 . Hence h : Nm → Nk+1 is not
injective.

We have shown that no mapping h of Nm into Nk+1 (m > k + 1)


is an injection. Hence by the Principle of Mathematical Induction, the
result follows.

Remark The assertion (b) of Theorem 4 is the Pigeonhole Principle. It


may be interpreted as saying that if m pigeons are put into n pigeonholes
and if m > n, then at least two pigeons must share one of the pigeonholes.

The next result states that a finite set might not have n elements
for more than one value of n.

Theorem 5 (Uniqueness Theorem) If S is a finite set, then the


number of elements in S is a unique number in N.

Proof If the set S has m elements, then by Definition 1(b), there exists
a bijection f1 from Nm onto S. If S also has n elements, there exists a
bijection f2 from Nn onto S.

Case 1) If m > n, then f2−1 ◦ f1 is a bijection of Nm onto Nn , which


contradicts Pigeonhole Principle.

Case 2) If n > m, then f1−1 ◦ f2 is a bijection of Nn onto Nm which


1.1 Enumerating a Finite Nonempty Set 5

contradicts Pigeonhole Principle.

Therefore m = n. This completes the proof.

Theorem 6 (a) If n ∈ N, then there is an injection from Nn into N.

(b) If n ∈ N, then there does not exist an injection from N into Nn .

Proof

(a) It can be seen that the function f : Nn → N defined by

f (m) = m, m ∈ Nn

is an injection from Nn intoN.

(b) Assume that f : N → Nn is an injection, and let m = n + 1. Then


the restriction of f to Nm ⊂ N is also an injection into Nn . But this
contradicts Pigeonhole Principle. Hence there does not exist an injection
from N into Nn .

We now prove a familiar result.

Corollary The set N of natural numbers is an infinite set.

Proof Assume that N is a finite set. Obviously N is nonempty, so the


remaining possibility is that it has m elements for some m ∈ N. Then,
by the definition, there is a bijection f from Nm onto N. But this implies
that the inverse function f −1 : N → Nm is a bijection and in particular
f −1 is an injection, contrary to part (b) of Theorem 6.

Theorem 7 If A is a set with m elements and B is a set with n elements


and if A ∩ B = ∅, then A ∪ B has m + n elements.

Proof Suppose A is a set with m elements and B is a set with n elements.


Then, by the Definition 1(b), there are bijections f : Nm → A and
6 Chapter 1. Finite and Infinite Sets

g : Nn → B. We define h on Nm+n by
(
f (i) for i = 1, ..., m
h(i) =
g(i − m) for i = m + 1, ..., m + n

Claim: h is a bijection from Nm+n onto A ∪ B.

h is an injection: h(i) = h(j) implies either f (i) = f (j) or g(i − m) =


g(j − m) which implies (since both f and g are injective) that i = j or
i − m = j − m; which implies i = j.

h is an surjection: c ∈ A ∪ B implies either c ∈ A or c ∈ B. If c ∈ A


then there exists i ∈ Nm such that f (i) = c. Hence, h(i) = c. If c ∈ B
then there exists j ∈ Nn such that g(j) = c. Taking i = m + j, we have
g(i − m) = c . Hence, h(i) = c.

This completes the proof of the theorem.

Theorem 8 If A is a set with m ∈ N elements and C ⊆ A is a set with


1 element then A\C is a set with m − 1 elements.

Proof Suppose A is a set with m ∈ N elements. Then, by Definition


1(b), there is a bijection f : Nm → A.

Suppose C is a set with 1 element and C ⊆ A = f (Nm ). Then,


C = {f (k)} for some k ∈ Nm . Define g on Nm−1 by
(
f (i) for i = 1, · · · , k − 1,
g(i) =
f (i + 1) for i = k, · · · , m − 1.

Then (it can be seen that) g is a bijection of Nm−1 onto A\C. Hence by
Definition 1(b), A\C is a set with m − 1 elements.

Theorem 9 If C is an infinite set and B is a finite set, and then C\B


is an infinite set.
1.1 Enumerating a Finite Nonempty Set 7

Proof Suppose C is an infinite set and B is a finite set. To show that


C\B is an infinite set, we apply contradiction method.

If C\B is a finite set, say having m elements, and B is a finite set


with n elements, then by Theorem 7, (C\B) ∪ B has m + n elements.
i.e., C has m + n elements, which is a contradiction to the fact the C is
not finite.

Theorem 10 Suppose that S and T are sets and that T ⊆ S.

a) If S is a finite set, then T is a finite set.

b) If T is an infinite set, then S is an infinite set.

Proof

Case 1) If T = ∅, then by Definition 1 (c), T is a finite set.

Case 2) Suppose that T 6= ∅. We prove the result using Principle of


Mathematical Induction on the number of elements in S.

If S has one element, then the only nonempty subset T of S must


coincide with S, so T is a finite set. Hence the result is true for a set
having one element.

As induction hypothesis, suppose that every nonempty subset of a


set with k elements is finite. Now let S be a set having k + 1 elements
(so there exists a bijection f of Nk+1 onto S ), and let T ⊆ S.

Case 2b) If f (k + 1) ∈
/ T, we can consider T to be a subset of
S1 = S\{f (k + 1)}, which has k elements by Theorem 8. Hence, by the
induction hypothesis, T is a finite set.

Case 2c) If f (k + 1) ∈ T , then T1 = T \{f (k + 1)} is a subset of


S1 = S\{f (k + 1)}. Since S1 has k elements, the induction hypothesis
implies that T1 is a finite set. But this implies (using Theorem 7) that
8 Chapter 1. Finite and Infinite Sets

T = T1 ∪ {f (k + 1)} is also a finite set.

We have shown that every nonempty subset of a set with k + 1 ele-


ments is also finite.

We conclude that every subset of a finite set is finite.

1. This assertion is the contrapositive of the assertion in (a). [In


contrapositive method, we use the fact that p ⇒ q and ¬q ⇒ ¬p
are logically equivalent.] We take

p : S is a finite set

q : T is a finite set.

We have proved in (a) that p ⇒ q is true. Hence, by contrapositive


method ¬q ⇒ ¬p is also true. i.e., (b) is also true. This completes the
proof.

1.2 Countable Sets

We now introduce an important type of infinite set.

Definition 2

a) A set S is said to be denumerable (or countably infinite) if there


exists a bijection from N onto S.

b) A set S is said to be countable if it is either finite or denumerable.

c) A set S is said to be uncountable if it is not countable.


1.3 Enumerating a Denumerable Set 9

1.3 Enumerating a Denumerable Set

If S is denumerable, then by Definition 2(a), there is bijection f : N → S


so that every element s in S is of the unique form s = f (n) for some
n ∈ N. Hence the elements in a denumerable set S can be displayed by
the enumeration

S = {f (1), f (2), f (3), . . .}.

Theorem 11 S is denumerable if and only if there exists a bijection of


S onto N.

Proof To prove this result, we use the fact that inverse of a bijection is
a bijection.

By the definition, S is denumerable if and only if there exists a bi-


jection, say, f of N onto S. i.e., if and only if there exists bijection f −1
of S onto N. This completes the proof.

Theorem 12 N is denumerable.

Proof The identity map i : N → N is a bijection of N onto N. Hence,


by Definition 2(a), N is denumerable.

Corollary N is countable.

Proof By Theorem 12, N is denumerable. Hence N is countable.

Example 1 Show that the set of even natural numbers is denumerable.

Solution The set of even natural numbers is E = {2n : n ∈ N}. Define


the mapping f : N → E by

f (n) = 2n for n ∈ N.
10 Chapter 1. Finite and Infinite Sets

f is injective: f (n1 ) = f (n2 ) ⇒ 2n1 = 2n2 ⇒ n1 = n2 .

f is surjective: m ∈ E ⇒ m = 2n for some n ∈ N. This implies

m = f (n).

Hence f is a bijection of N onto E and hence by Definition 2(a), E is


denumerable.

Example 2 Show that the set of odd natural numbers is denumerable.

Solution

The set O = {2n−1 : n ∈ N} of odd natural numbers is denumerable,


because the mapping g : N → O defined by

g(n) = 2n − 1, for n ∈ N,

is a bijection of N onto O .

Example 3 Show that the set Z of all integers is denumerable.

Solution

To construct a bijection of N = { 1, 2, 3, ...} onto


Z = { ... − 3, −2, −1, 0, 1, 2, 3, ...}, we map 1 onto 0, we map the
set of even natural numbers onto the set N of positive integers, and we
map the set of odd natural numbers onto the negative integers. This
mapping can be displayed by the enumeration:

Z = {0, 1, −1, 2, −2, 3, −3, ...}


1.3 Enumerating a Denumerable Set 11

and the mapping is f : N → Z defined by


(
n
2, when n is even
f (n) =
− (n−1)
2 , when n is odd

The mapping f : N → Z is a bijection (Details are left as an exercise)


and hence Z is denumerable.

Example 4 Show that the union of two disjoint denumerable sets is


denumerable.

Solution

If A and B are denumerable we can display the elements of the sets as


follows:
A = {a1 , a2 , a3 , ...}

and
B = {b1 , b2 , b3 , ...}.

Then we can enumerate the elements of A ∪ B as:

a1 , b1 , a2 , b2 , a3 , b3, ...

The corresponding bijective mapping f : N → A ∪ B is defined by


(
bn/2 , when n is even
f (n) =
a(n+1)/2 , when n is odd

Theorem 13 The set N × N is denumerable.

Proof Note that N×N = {(m, n) where m, n ∈ N}. We can enumerate


the pairs (m, n) as
12 Chapter 1. Finite and Infinite Sets

(1, 1), (1, 2), (2, 1), (1, 3), (2, 2), (3, 1), (1, 4), . . . ,

according to increasing sum m + n and increasing m and the same is


displayed in Fig. 1.1 by following the arrows in each diagonal.

Figure 1.1: The set N × N

To define a bijection from N × N → N we proceed as follows:

We note that the first diagonal has one point, the second has two
points, . . . , and the k th diagonal has k points. Hence the total number
of points in diagonal 1 through k is given by

1
ψ(k) = 1 + 2 + · · · + k = k(k + 1).
2

Also,
ψ(k + 1) = ψ(k) + (k + 1)

and ψ(k + 1) ≥ ψ(k) for k ∈ N.

Thus ψ is strictly increasing.

The point (m, n) in N×N lies in the k th diagonal when k = m+n−1,


1.3 Enumerating a Denumerable Set 13

and it is the mth point in that diagonal as we move downward from left to
right. Hence, in the counting scheme (or enumeration process), we count
the point (m, n) by first counting the points in the first k−1 = m+ n−2
diagonals and then adding m; and the corresponding counting function
h : N × N → N is given by

h(m, n) = ψ(m + n − 2) + m for (m, n) ∈ N × N .

Claim: h is a bijection from N × N onto N.

h is injective: If (m, n) 6= (m0 , n0 ), then either m + n 6= m0 + n0 , or


m + n = m0 + n0 , and m 6= m0 .

Case 1) In the case m + n 6= m0 + n0 , suppose m + n < m0 + n0 ,


then

h(m, n) = ψ(m + n − 2) + m
≤ ψ(m + n − 2) + (m + n − 1)
= ψ(m + n − 1)
≤ ψ(m0 + n0 − 2), since m + n − 1 < m0 + n0 − 1
and ψ is strictly increasing.
< ψ((m0 + n0 − 2) + m0 ), since m0 > 0
= h(m0 , n0 ).

i.e., h(m, n) 6= h(m0 , n0 ). The case of m0 + n0 < m + n can be handled


similarly, or by symmetry.

Case 2) If m + n = m0 + n0 , and m 6= m0 , then

h(m, n) − m = ψ(m + n − 2) = ψ(m0 + n0 − 2) = h(m0 , n0 ) − m0 ,

and hence h(m, n) − h(m0 , n0 ) = m − m0 6= 0 and so h(m, n) 6=


14 Chapter 1. Finite and Infinite Sets

h(m0 , n0 ).

We conclude that h is injective.

h is surjective: Clearly 1 ∈ N has a pre-image, since h(1, 1) = 1.

If p ∈ N with p ≥ 2, we have to find an element (mp , np ) ∈ N × N


with h(mp , np ) = p. Since p < ψ(p), the set

Ep = {k ∈ N : p ≤ ψ(k)}

is nonempty. Being a non empty subset of N, by Well-Ordering Property,


Ep has a least element kp > 1. (This means that p lies in the kp th
diagonal). Since p ≥ 2, it follows that

ψ(kp − 1) < p ≤ ψ(kp ) = ψ(kp − 1) + kp .

Let mp = p − ψ(kp − 1) so that 1 ≤ mp ≤ kp , and let np = kp − mp + 1


so that 1 ≤ np ≤ kp and mp + np − 1 = kp . Hence

h(mp , np ) = ψ(mp + np − 2) + mp = ψ(kp − 1) + mp = p.

Thus h is surjective.

This completes the proof of the claim. i.e., h is a bijection from N × N


onto N. Hence N × N is denumerable. This completes the proof of the
theorem.

Theorem 14 (a) A set S1 is denumerable if and only if there exists a


bijection from S1 onto a set S2 that is denumerable.

(b) A set T1 is countable if and only if there exists a bijection from


T1 onto a set T2 that is countable.
1.3 Enumerating a Denumerable Set 15

Proof

(a) By Theorem 11, S is denumerable if and only if there exists a bijection


of S onto N.

Hence if S1 is denumerable, then there exists a bijection of S1 onto


N. Now take S2 = N. Noting that N is denumerable, we have proved
that there exists a bijection from S1 onto a set S2 that is denumerable.

Conversely, suppose f is a bijection of S1 onto S2 that is denumerable.


Since S2 is denumerable, by Theorem 11, there is a bijection, say, g from
S2 onto N. Being the composition of bijections, g ◦ f is a bijection of
S1 onto N, so (by Theorem 11 it follows) that S1 is denumerable.

(b) Proof of (b) is left as an exercise.

Next Theorem says that subset of a countable set is countable. Su-


perset of an uncountable set is uncountable.

Theorem 15 Suppose that S and T are sets and that T ⊆ S.

a) If S is a countable set, then T is a countable set.

b) If T is an uncountable set, then S is an uncountable set.

Proof We prove (a), leaving the proof (b) as an exercise. If S is finite,


we may apply Part (a) of Theorem 9. Hence we may assume that S is
denumerable. If T is finite, then we are through; hence we may assume
that T is also infinite.

Now let f be a bijection of N onto S (such a bijection exists as S is


denumerable) and let

P = {n ∈ N : f (n) ∈ T } = f −1 (T ).

Since T is not empty, it follows that P is not empty, and we can apply
16 Chapter 1. Finite and Infinite Sets

the Well-Ordering Property of N to obtain the least element p1 of P . We


then let p2 be the least element of the nonempty set P \{p1 } (P \{p1 }
is nonempty, for otherwise P contains only p1 and hence P is finite,
a contradiction to the fact that T is infinite). Continuing in this way,
having selected p1 , p2 , . . . , pk in P , we let pk+1 be the least element in
the nonempty set P \{p1 , . . . , pk }. By construction, we have 1 ≤ p1 <
p2 < · · · < pk < pk+1 < · · · , and hence it follows (by induction) that
r ≤ pr for all r ∈ N.

We now define g : N → N by

g(r) = pr for all r ∈ N.

From the fact that pk+1 ∈


/ {p1 , . . . , pk }, we infer that g is injective.
Therefore the composition f ◦ g is an injection of N into S. Since the
values of f ◦ g are contained in T ⊆ S, we can consider f ◦ g to be an
injection from N to T . To see that f ◦ g is a surjective map of N onto T ,
recall that f is a surjective map onto S so that every t ∈ T ⊆ S has the
form t = f (nt ) for some nt ∈ P ⊆ N. However, since r ≤ pr = g(r) for
r ∈ N, we have nt = g(mt ) for some mt ∈ N with mt ≤ nt . Therefore
t = f ◦ g(mt ), which shows that f ◦ g is a surjection of N onto T .
Therefore, T is denumerable.

Theorem 16 The following statements are equivalent:

a) S is a countable set.

b) There exists a surjection of N onto S.

c) There exists an injection of S into N.

Proof

(a) ⇒ (b) If S is finite, say having n elements, then there exists a


1.3 Enumerating a Denumerable Set 17

bijection h from Nn onto S and we define H on N by


(
h(k) for k = 1, ..., n,
H(k) =
h(n) for k > n.

Then H is a surjection of N onto S.

If S is denumerable, there exists a bijection H of N onto S, which is


also a surjection of N onto S.

(b) ⇒ (c) If H is a surjection of N onto S we define H1 : S → N by letting


H1 (s) be the least element in the non empty set H −1 (s) = {n ∈ N :
H(n) = s}. To see that H1 is an injection of S intoN, note that if
s, t ∈ S and H1 (s) = H1 (t), say equal to m, then m ∈ H −1 (s) and
m ∈ H −1 (t), so that H(m) = s and H(m) = t .Thus s = t, and hence
H1 is an injection.

(c) ⇒ (a) If H1 is an injection of S into N, then it is a bijection of S


onto H1 (S) ⊆ N. By Part (a) of Theorem 15(a), H1 (S) is countable,
and hence the set S is countable.

Remark In view of (a) ⇔ (b) part of Theorem 16, we note that a


countable set is a set S whose elements can be displayed as the range of
a sequence
S = {x1 , x2 , . . . , xn , . . .}

where repetitions are allowed. Consequently the requirement that the


mapping be injective can be relaxed.

Example 5 Prove that if A and B are denumerable, then A ∪ B is


denumerable.

Solution
18 Chapter 1. Finite and Infinite Sets

We first invite the attention of the reader that here A and B need
not be disjoint. So we are not attempting to get a bijective mapping
from N → A ∪ B as in Example 3. Since A is denumerable and since
A ⊂ A ∪ B, the set A ∪ B is not finite. Hence to show that A ∪ B is
denumerable it is enough to show that A ∪ B is countable. By Theorem
16, it is enough to exhibit a surjection of N onto A ∪ B. Since A is
denumerable, there will be an enumeration

a1 , a2 , a3 , . . .

Similarly, B has an enumeration

b1 , b2 , b3 , . . .

Now we display elements of A ∪ B in an array as shown below.

Figure 1.2:

The surjection from N onto A∪B is defined as indicated by the diagonal


lines. Hence A ∪ B is denumerable.

Theorem 17 The set Q of all rational numbers is denumerable.

Proof We first show that Q+ is countable. In view of (a) ⇔ (b) part of


Theorem 16, it is enough to find a surjection from N onto Q+ . Since
N × N is countable, it follows [from (a) ⇔ (b) part of Theorem 16] that
there exists a surjection f from N onto N × N. If g : N × N → Q+ is
1.3 Enumerating a Denumerable Set 19

the mapping that sends the ordered pair (m, n) into the rational number
having a representation m/n then g is a surjection onto Q+ . Therefore
the composition g ◦f is a surjection from N onto Q+ , and hence Theorem
16 implies that Q+ is a countable set.

Similarly, the set Q− of all negative rational numbers is countable.


Hence, the set Q = Q− ∪ {0} ∪ Q+ is countable. Since Q contains the
denumerable set N, the set Q cannot be finite. Hence Q is a denumerable
set. This completes the proof.

Remark A less formal way to show that Q+ is countable is to enumerate


m
the elements of Q+ as follows. We display the set F of all fractions n
with m, n ∈ N in an array as shown in Fig. 2.1, where the nth row
consists of all fractions with denominator n.

Figure 1.3: The set Q+

The function from N onto the set F of fractions is defined as indicated


by the diagonal lines. We have
 
1 1 2 1 2 3
F = , , , , , , ··· .
1 2 1 3 2 1
20 Chapter 1. Finite and Infinite Sets

Therefore, by Theorem 16, the set F is countable. Each rational number


1
is represented by many different members of F (for example, 1 = 1 =
2 +
2 = · · · ). For each r ∈ Q , we can select the representing fraction with
the smallest denominator, and thus we can regard Q+ as a subset of F .
Hence, being the subset of the countable set F , the set Q+ is countable.

Theorem 18 If Am is a countable set for each m ∈ N, then the union


∪∞
m=1 Am is countable. [Here Am ’s may be overlapping].

Proof Let A = ∪∞
m=1 Am . For each m ∈ N, let ϕm be a surjection of
N onto Am (such a surjection exists as Am is countable). We define
ψ : N × N → A by
ψ(m, n) = ϕm (n)

Claim: ψ is a surjection.

If a ∈ A, then a ∈ ∪∞
m=1 Am which implies there exists a least m ∈ N
such that a ∈ Am ; and since ϕm : N → Am is a surjection, a ∈ Am
implies there exists a least n ∈ N such that a = ϕm (n). Therefore,
a = ψ(m, n). Hence ψ is a surjection.

Since N × N is countable, it follows from Theorem 16 that exists a


surjection f : N → N × N and hence here ψ ◦ f is a surjection of N onto
A. Now apply Theorem 16 again to conclude that A is countable.

Remark: A less formal (but more intuitive) way to see the truth of
theorem is to enumerate the elements of Am , m ∈ N as:

A1 = {a11 , a12 , a13 , ...}


A2 = {a21 , a22 , a23 , ...}
A3 = {a31 , a32 , a33 , ...}
1.3 Enumerating a Denumerable Set 21

We then enumerate this array using the “diagonal procedure”:

a11 , a12 , a21 , a13 , a22 , a31 , a14 , ...,

as was displayed in Fig. 1.

Theorem 19 (Cantor’s Theorem) If A is any set, then there is no


surjection of A onto the set P (A) of all subsets of A.

Proof Suppose that ϕ : A → P (A) is a surjection. As ϕ(a) ∈ P (A) for


a ∈ A, we have ϕ(a) is a subset of A. Hence either a belongs to ϕ(a) or
it does not belong to this set. We let

D = {a ∈ A : a ∈
/ ϕ(a)}.

Since D is a subset of A, D ∈ P (A) and hence if ϕ is a surjection from


A onto P (A), then D = ϕ(a0 ) for some a0 ∈ A.

We must have either a0 ∈ D or a0 ∈


/ D. If a0 ∈ D, then since
D = ϕ(a0 ), we must havea0 ∈ ϕ(a0 ), contrary to the definition of D.
Similarly, if a0 ∈
/ D, then a0 ∈
/ ϕ(a0 ) so that a0 ∈ D, which is also a
contradiction.

Therefore, ϕ cannot be a surjection. This completes the proof.

Remark Using Cantor’s Theorem, starting with a set it is possible to


obtain larger and larger sets. A particular case is dealt in the following
example.

Corollary The collection P (N) of all subsets of the set N of natural


numbers is uncountable.

Proof By Cantors’s theorem, there is no surjection of N onto the set


P (N) of all subsets of N. Hence, by Theorem 16, P (N) is not countable.
22 Chapter 1. Finite and Infinite Sets

Exercises

1. Prove that a nonempty set T1 is infinite if and only if there is a


bijection from T1 onto a finite set T2 .

2. Let S = {1, 2} and T = {a, b, c}

a) Determine the number of different injections from S into T

b) Determine the number of different surjections from T onto S.

3. Exhibit a bijection between N and the set of all odd integers greater
than 13.

4. Exhibit a bijection between N and a proper subset of itself.

5. Give an example of a countable collection of finite sets whose union


is not finite.

6. Determine the number of elements in P (S), the collection of all subsets


of S, for each of the following sets:

a) S = {1, 2},

b) S = {1, 2, 3},

c) S = {1, 2, 3, 4}.

Be sure to include the empty set and the set S itself in P (S).

7. Prove that the collection P (N) of all finite subsets of N is countable.

Answers

2. (a) There are 6 = 3 · 2 · 1 different injections of S into T .

(b) There are 3 surjections that map a into 1, and there are 3 other
surjections that map a into 2. Hence there are 6 surjections.

6. (a) P ({1, 2}) = {Φ, {1}, {2}, {1, 2} has 22 = 4 elements.


1.3 Enumerating a Denumerable Set 23

(c) P ({1, 2, 3, 4}) has 24 = 16 elements.

7. For each m ∈ N, the collection of all subsets of Nm is finite. Note


S∞
that P (N) = m=1 P (Nm )
Chapter 2
The Algebraic Properties of R

Introduction

The rational number system is inadequate for many purposes, both


as a field and as an ordered set. For example, there is no rational number
r such that r2 = 2. This leads to the introduction of so-called irrational
numbers which are often written as infinite decimal expansions and are
considered to be approximated by the corresponding finite decimals. For

example, the irrational number 2 is approximated by the sequence

1, 1.4, 1.41, 1.414, 1.4142, . . .

The set consisting of all rational and irrational numbers is the set of
real numbers R = (−∞, ∞). The set of real numbers is also known as
the real line for the following reason: the real numbers has a one-to-one
correspondence with the points of the straight line. In this text book
we shall consider the axioms that characterize the real numbers. They

24
2.1 Binary Operation 25

consist of the field axioms (content of this chapter), the order axioms
(content of the next chapter) and the completeness axiom (content of
chapter 5). In algebraic terminology, the set of real numbers is referred
to as the one and only complete ordered field.

2.1 Binary Operation

A binary operation (binary composition) on a nonempty set S is


a function, usually denoted by ∗, which associates each ordered pair
(a, b) of elements a, b in S to a unique element c ∈ S. Usually c is
denoted by a ∗ b. Thus a binary operation ∗ on S is a function such
that
∗:S×S →S

where
∗(a, b) = a ∗ b .

If ∗ is a binary operation on a set S, we say that S is closed under the


operation ∗.

In R, the set of real numbers, both usual addition and usual mul-
tiplication are binary operations. We use the conventional notations of
a + b and a · b (or merely ab) when discussing the properties of addition
and multiplication.

We begin by giving a list of basic properties of addition and multipli-


cation. This list includes all the essential algebraic properties of R in
the sense that all other such properties can be deduced as theorems. In
the terminology of abstract algebra, algebraic properties of R are often
26 Chapter 2. The Algebraic Properties of R

called field properties of R.1

Algebraic Properties (Field Axioms) of R

On the set R of real numbers there are two binary operations, denoted by
+ and · and called addition and multiplication, respectively. These
operations satisfy the following properties.

(A1) (Commutative property of addition)

a + b = b + a for all a, b in R;

(A2) (Associative property of addition)

(a + b) + c = a + (b + c) for all a, b, c in R;

(A3) (Existence of a zero element)

There exists an element 0 in R such that

0 + a = a and a + 0 = a for all a in R;

(A4) (Existence of negative elements)

For each a in R there exists an element −a in R such that a + (−a) = 0


and (−a) + a = 0;

(M1) (Commutative property of multiplication)

a · b = b · a for all a, b in R;

(M2) (Associative property of multiplication)

(a · b) · c = a · (b · c) for all a, b, c in R;

(M3) (Existence of a unit element)

There exists an element 1 in R distinct from 0 such that 1 · a = a and


1 A detailed discussion of the concept of field is done in ‘Abstract Algebra’ (Fifth

semester core course)


2.1 Binary Operation 27

a · 1 = a for all a in R;

(M4) (Existence of reciprocals)


1 1

For each a 6= 0 in R there exists an element a in R such that a · a =1
and a1 · a = 1;


(D) (Distributive property of multiplication over addition)

For all a, b, c in R,

a · (b + c) = (a · b) + (a · c)

and
(b + c) · a = (b · a) + (c · a).

The above are the field properties. Hence R is a field.

Remark The above list is not “minimal” since, for convenience, we


have included a few redundancies. For example, the second assertions in
(A3) and (A4) follow from the first assertions by using the commutative
property in (A1). Also, the second assertions in (M3) and (M4) follow
from the first assertions by using the commutative property in (M1).

Example 1 Solve the equation

2x + 5 = 8

justifying each step by referring to an appropriate field property of R.

Solution

2x + 5 = 8 ⇒ (2x + 5) + (−5) = 8 + (−5), adding −5 to both sides

⇒ 2x + [5 + (−5)] = 3, using associative property of addition


28 Chapter 2. The Algebraic Properties of R

⇒ 2x + 0 = 3, as −5 is the additive inverse of 5

⇒ 2x = 3, since 0 is the zero element


1
⇒ (2x) = 12 · 3, multiplying both sides by 12
2

⇒ 12 · 2 x = 12 · 3, using associative property of multiplication




⇒ 1 · x = 32 , as 1
2 is the multiplicative inverse of 2

⇒ x = 32 , since 1 is the unit element.

Theorem 1 (a) [Uniqueness of zero element] If z and a are elements of


R such that z + a = a, then z = 0.

(b) [Uniqueness of unit element] If u and b 6= 0 are elements of R such


that u · b = b, then u = 1.

(c) [Multiplication by 0 gives 0] If a ∈ R, then a · 0 = 0.

Proof

(a) Supposez + a = a. Then

z = z + 0, using (A3),

= z + (a + (−a)), using (A4)

= (z + a) + (−a), using (A2)

= a + (−a), using the the hypothesis z + a = a

= 0, using (A4)

(b) Using (M3), (M4), (M2), the assumed equality u · b = b, and (M4)
again, we get

u = u · 1 = u · (b · (1/b)) = (u · b) · (1/b) = b · (1/b) = 1.


2.1 Binary Operation 29

(c) From (M3) we know that a · 1 = a. Then adding a · 0 and using (D)
and (A3) we get

a+a·0 = a·1+a·0
= a · (1 + 0) = a · 1 = a.

Thus, we conclude from (a) that a · 0 = 0.

We next show that for a given element a in R, the element −a and


1
the element a (when a 6= 0) are uniquely determined.

Theorem 2

(a) If a and b are elements of R such that a + b = 0, then b = −a.


1
(b) If a 6= 0 and b are elements of R such that a · b = 1, then b = a

(c) If a · b = 0, then either a = 0 or b = 0.

Proof

(a) Using (A3), (A4), (A2), the hypothesis a + b = 0 and (A3) we have

b = 0 + b = ((−a) + a) + b = (−a) + (a + b) = (−a) + 0 = −a.

(b) Using (M3), (M4), (M2), the hypothesis a · b = 1, and (M3) we have

b = 1 · b = ((1/a) · a) · b = (1/a) · (a · b) = (1/a) · 1 = 1/a.

(c) Suppose a · b = 0 and a 6= 0. We prove that b = 0. We multiply a · b


by 1/a and apply (M2), (M4) and (M3) to get

(1/a) · (a · b) = ((1/a) · a) · b = 1 · b = b.
30 Chapter 2. The Algebraic Properties of R

Since a · b = 0, by Part (c) of the previous theorem this gives

(1/a) · (a · b) = (1/a) · 0 = 0.

Thus we have b = 0.

The verification that a · b = 0 and b 6= 0 implies a = 0 is left as an


exercise.

Theorem 3 Let a, b be arbitrary elements of R. Then:

(a) the equation a + x = b has the unique solution x = (−a) + b;


1

(b) if a 6= 0, the equation a · x = b has the unique solution x = a ·b

Proof Using properties (A2), (A4), and (A3) in succession, we obtain

a + ((−a) + b) = (a + (−a)) + b = 0 + b = b,

which implies that x = (−a) + b is a solution of the equation a + x = b.


To show that it is the only solution, suppose that x1 is any solution of
the equation then a + x1 = b, and if we add −a to both sides, we get

(−a) + (a + x1 ) = (−a) + b. (2.1)

If we now use (A2), (A4), and (A3) on the left side of (2.1), we obtain

(−a) + (a + x1 ) = (−a + a) + x1 = 0 + x1 = x1

Hence from (2.1), we conclude that x1 = (−a) + b.

The proof of part (b) is left as an exercise.

Theorem 4 If a is any element of R, then:


2.1 Binary Operation 31

(a) (−1) · a = −a, (b) −(−a) = a,

(c) (−1) · (−1) = 1 (d) (−a) · (−a) = a2

Proof (a) We use (D), in conjunction with (M3), (A4), and to obtain

a + (−1) · a = 1 · a + (−1) · a, using (M3)

= [1 + (−1)] · a, using (D)

= 0 · a, using (A4)

= 0, using the resulta · 0 = 0.

Thus, from part (a) of Theorem 2, we conclude that (−1) · a = −a.

(b) By (A4) we have(−a) + a = 0. Thus from part (a) of Theorem 2 it


follows that a = −(−a).

(c) In part (a), substitute a = −1. Then

(−1) · (−1) = −(−1)

Hence, the assertion follows from part (b) with a = 1.

(d) (−a)(−a) = [(−1)a][(−1)a], since (−1)a = −a

= (−1)a (−1)a, using the associative law

= (−1)(−1) · a2 , using the commutative law

= 1 · a2 , since (−1) · (−1) = 1

= a2 , since 1 is the unit element.

Example 2 Prove that if a, b ∈ R, then

−(a + b) = (−a) + (−b),


32 Chapter 2. The Algebraic Properties of R

Solution

−(a + b) = (−1)(a + b), using Part (a) of Theorem 4

= (−1)a + (−1)b, using Distributive property

= (−a) + (−b), again using Part (a) of Theorem 4

Example 3 Prove that if a ∈ R and a 6= 0, then


 
1 1
=− .
(−a) a

Solution
1
= (−1)(a) (−1) a1 , using Part (a) of Theorem 4
 
(−a) − a

= (−1)(−1)(a) a1 , using associative and commutative properties of




multiplication

= a a1 , since (−1) · (−1) = 1




= 1.

Hence by Part (b) of Theorem 2,


 
1 1
− = .
a (−a)

Theorem 5 Let a, b, c, be elements of R.


1 1
(a) If a 6= 0, then a 6= 0 and 1 = a.
a

(b) If a · b = a · c and a 6= 0, then b = c.

Proof

(a) We are given a 6= 0, so by (M4), a1 exists. a1 6= 0; for otherwise a1 = 0,


and then 1 = a · a1 = a · 0 = 0, contrary to (M3). Thus a1 6= 0, and

2.1 Binary Operation 33

1

by (M4), we have a · a = 1, and Part (b) of Theorem 2 implies that
1 = a.
1
a
1
(b) If we multiply both sides of the equation a · b = a · c by a and apply
the associative property (M2), we get

1 1
   
a ·a ·b= a · a · c.

Thus, 1 · b = 1 · c which is the same as b = c.

From now on we shall generally drop the use of the dot to denote multi-
plication and write ab for a · b. As usual, we shall write a2 for aa, a3 for
(a2 )a; more generally, for n ∈ N, we define an+1 = (an ) · a. We agree to
adopt the convention that a0 = 1 and a1 = a for any a ∈ R.

Also, if a ∈ R, then
am+n = am an

for all m, n ∈ N. If a 6= 0, we will use the notation a−1 for a1 , and if


n
n ∈ N, we will write a−n for a1 , when it is convenient to do so.

More Operations using the Field Axioms

The operation of subtraction is defined by

a − b = a + (−b)

for a, b in R.

Similarly, division is defined for a, b in R, b 6= 0, by


 
a 1
=a· .
b b
34 Chapter 2. The Algebraic Properties of R

2.2 Rational and Irrational Numbers

We regard the set N of natural numbers as being a subset of R by


identifying the natural number n ∈ N with the n-fold sum of the unit
element 1 ∈ R. Similarly, we identify 0 ∈ Z with the zero element of R,
and we identify the n-fold sum of −1 with the integer −n. Consequently,
we regard N and Z as subsets of R.
b
Elements of R that can be written in the form a where a, b ∈ Z and
a 6= 0 are called rational numbers. The set of all rational numbers in
R will be denoted by the standard notation Q.

Example 4 Show that the sum and product of two rational numbers is
again a rational number.

Solution
m1 m2
If a = n1 , b= n2 are elements in Q, then n1 6= 0, n2 6= 0 and

m1 m2 m1 n2 + m2 n1
a+b= + = (2.2)
n1 n2 n1 n2

and since m1 n2 + m2 n1 ∈ Z and n1 n2 6= 0 , Equation (2.2) implies that


a + b is a rational number. Hence the sum of two rational numbers is a
rational number. Also,

m1 m2 m1 m2
ab = = (2.3)
n1 n2 n1 n2

and since m1 m2 ∈ Z and n1 n2 6= 0 , equation (2.3) implies that ab is a


rational number. Hence the product of two rational numbers is a rational
number.

Q is a Field
2.2 Rational and Irrational Numbers 35

We have seen in Example that the sum and product of two rational
numbers is again a rational number. It can be seen that the field prop-
erties listed at the beginning of this chapter can be shown to hold for Q.
Hence Q is a field.

Irrational Numbers

In the later sections of this chapter we shall show that there exists
elements in R that are not in Q. Elements of R that are not in Q became
known as irrational numbers, meaning that they are not ratios of
integers. Although the word “irrational” in modern English usage has a
quite different meaning, we shall adopt the standard mathematical usage
of this term.

The fact that there does not exist a rational number whose square is
2 is the content of the next theorem. In the proof we use the notions of
even and odd numbers. Recall that a natural number is even if it has
the form 2n for some n ∈ N, and it is odd if it has the form 2n − 1 for
some n ∈ N. The natural number 1 is not even as it cannot be written
in the form 1 = 2n for some n ∈ N. However, 1 is odd, since 1 = 2 · 1 − 1.

Example 5 Prove that a natural number cannot be both even and odd.

Solution

Let p be a natural number that is both even and odd. Then p = 2m


for some m ∈ N and p = 2n−1 for some n ∈ N, so that 2m = 2n−1; this
implies 1 = 2(n − m). This implies that 1 is an even natural number, a
contradiction. Hence there does not exist a natural number that is both
even and odd.

Every natural number is either even or odd, and no natural number


is both even and odd.
36 Chapter 2. The Algebraic Properties of R

Theorem 6 There does not exist a rational number r such that r2 = 2.



In other words 2 is an irrational number.

Proof

Suppose, on the contrary, that there exists a rational number r such


that r2 = 2. Then there are integers p and q such that r = pq and
 2
hence pq = 2. Also assume that p and q are positive and have no
common integer factors other than 1 (This choice is possible for the
following reason: If one (or both) of p and q is negative, then squaring it
nullify the sign. Hence we can choose both p and q as positive. If p and
q have common factors other than 1, then all the common factors will
be cancelled out with numerator and denominator leaving 1 as the only
common factor. Now take the new numerator as p and the denominator
as q.)

Since p2 = 2q 2 , we see that p2 is even. This implies that p is also


even (because if p is odd, then p = 2n − 1 for somen ∈ N, then its square
p2 = 4n2 − 4n + 1 = 2(2n2 − 2n + 1) − 1 is also odd). Therefore, since
p and q do not have 2 as a common factor, q must be an odd natural
number.

Since p is even, then p = 2m for some m ∈ N, and hence 4m2 = 2q 2 , so


that 2m2 = q 2 . Therefore, q 2 is even, and it follows from the argument
in the preceding paragraph that q is also an even natural number.

Hence q is both odd and even. That is, we have arrived at a con-
tradiction to the fact that no natural number can be both even and
odd. This contradiction makes us to conclude
2 that there does not exist
p
positive integers p and q such that q = 2. This completes the proof.

Remark The fact that there does not exist a rational number r such
2.2 Rational and Irrational Numbers 37

that r2 = 2 was observed in the sixth century B.C. by the ancient Greek
society of Pythagoreans. They discovered that the diagonal of a square
with unit sides could not be expressed as a ratio of integers (Fig. 2.1).
In view of the Pythagorean Theorem for right triangles, this implies that
the square of no rational number can equal 2.

Figure 2.1:

Example 6 Show that if ξ ∈ R is irrational and r 6= 0 is rational, then



r + ξ and rξ are irrational. In particular − 2 is irrational.

Solution

If r 6= 0 is a rational number, then using the field axioms in Q, −r


1
and r both belong to Q.

Now suppose to the contrary that, both r + ξ and rξ are rational. Again
using the field axioms in Q, we have the following:

1. −r ∈ Q, r+ξ ∈ Q implies −r+(r+ξ) ∈ Q implies (−r+r)+ξ ∈ Q


implies 0 + ξ ∈ Q implies ξ ∈ Q, a contradiction to the given
assumption that ξ is an irrational number.
38 Chapter 2. The Algebraic Properties of R

1 1 1

2. r ∈ Q, rξ ∈ Q implies r (rξ) ∈ Q implies rr ξ ∈ Q implies
1 · ξ ∈ Q implies ξ ∈ Q, a contradiction to the given assumption
that ξ is an irrational number.


Hence r + ξ and rξ are irrational. Also, by Theorem 6, 2 is irrational.

Hence, by what we have observed, −1 ∈ Q and 2 ∈ R\Q implies
√ √
(−1) 2 ∈ R\Q . i.e, − 2 is irrational.

Example 7 If x and y are irrational numbers, show that it does not


follow that x + y and xy are irrational.

Solution

We show this by considering examples. If we take x = 2 and

y = − 2, then both are irrational numbers, but their sum x + y =
√ √
2 + (− 2) = 0 is not irrational. For the same x and y,
√ √
xy = ( 2)(− 2) = −2,

which is again not irrational.

Example 8 Show that there does not exist a rational number t such
that t2 = 6.

Solution

Proof Suppose, on the contrary, that there exists a rational number t


such that t2 = 6.Then
 there are integers p and q are integers such that
2
p p
t= q and hence q = 6. Also assume that p and q are positive and
have no common integer factors other than 1.

Since p2 = 6q 2 = 2 · 3q 2 , we see that p2 is even. This implies that p


is also even. Therefore, since p and q do not have 2 as a common factor,
q must be an odd natural number.
2.2 Rational and Irrational Numbers 39

Since p is even, then p = 2m for some m ∈ N, and hence 4m2 = 6q 2 ,


so that 2m2 = 3q 2 . Therefore, 3q 2 is even, and hence q 2 is even, and it
follows that q is also an even natural number.

Hence q is both odd and even. That is, we have arrived at a con-
tradiction to the fact that no natural number can be both even and
odd. This contradiction makes us  to conclude
2 that there does not exist
p
positive integers p and q such that q = 6. This completes the proof.

Example 9 Show that there does not exist a rational number t such
that t2 = 3.

Proof Suppose, on the contrary, that there exists a rational number t


such that t2 = 3.Then
 there are integers p and q are integers such that
2
p p
t= q and hence q = 3. Also assume that p and q are positive and
have no common integer factors other than 1.
S
There are three possibilities for p ∈ N. For some m ∈ N {0}, (i) p = 3m
(ii) p = 3m + 1 or (iii) p = 3m + 2 . We shall show that each of these
cases gives a contradiction.

Case (i) p = 3m: Then p2 = 3q 2 implies 9m2 = 3q 2 implies 3m2 = q 2


implies q 2 is also a multiple of 3, implies q is also a multiple of 3 (because
if q is not a multiple of 3, then q = 3n + 1 or q = 3n + 2 implies
q 2 = (multiple of 3) + 1, a contradiction to the fact that q 2 is also a
multiple of 3). This shows that p and q have a common divisor 3 other
than 1, which is a contradiction.

Case (ii) p = 3m + 1: Now p2 = 3q 2 implies p2 is a multiple of 3, and


hence p is a multiple of 3, a contradiction to the fact that p = 3m + 1.

Case (iii) can be treated as in Case (ii ).

As each of the cases gives a contradiction, we conclude that there does


40 Chapter 2. The Algebraic Properties of R

not exist a rational number t such that t2 = 3.

Exercises

1. Prove that the additive inverse of an element in R is unique.

2. Prove that the multiplicative inverse of an element in R is unique.

3. Solve the following equations, justifying each step by referring to an


appropriate property or theorem.

(a) 3x + 7 = 8, (b) x2 = 3x,

(c) x2 − 1 = 8, (d) (x − 3)(x + 7) = 0.

(e) 2x + 6 = 3x + 2

4. Prove that if a, b ∈ R, then


a
 (−a)
(a) (−a) · (−b) = a · b (b) − b = b if b 6= 0.

5. If a ∈ R satisfiesa · a = a, prove that either a = 0 or a = 1.


1
= a1 · 1b .
 
6. If a 6= 0 and b 6= 0, show that ab

7. Show that 5 is not a rational number.

8. Prove that for any x ∈ Q ,x2 6= 3 , x2 6= 5 ,and x2 6= 6 .



9. Show that 8 is not a rational number.

10. Use Principle of Mathematical Induction to show that if a ∈ R and


m, n ∈ N, then am+n = am an and (am )n = amn .

Answers/Hints

3. (a) 1/3 (b) 0, 3


2.2 Rational and Irrational Numbers 41

(c) 3, −3 (d) 3, −7

5.a · a = a ⇒ a · a − a = 0 ⇒ a(a − 1) = 0 ⇒ a = 0 or a − 1 = 0 .

9. This will be discussed as an example in the next chapter.

10. Fix m ∈ N and use Mathematical Induction to prove that am+n =


am an and (am )n = amn for alln ∈ N. Then, for a givenn ∈ N, prove
that the equalities are valid for allm ∈ N.
Chapter 3
The Order Properties of R

Introduction

Real numbers possess an ordering relation. The ‘order’ properties of R


refer to the notions of positivity and inequalities between real numbers.
The relation between two distinct real numbers is denoted by the symbol
> which is read as ‘greater than’. The symbol < is read as ‘less than’.
In order to include the equality sign, the symbol ≥ read as ‘greater than
or equal to’ and the symbol ≤ read as ‘less than or equal to’ are also
considered.

Attention! Ordinarily the order relation does not exist between the
members of a general field. For example, no two complex numbers can
be related by an order; and hence the field of complex numbers doesn’t
possess an order relation. But, R is an ordered field. Q is also an
ordered field. We also note that R\Q is not even a field.

42
3.1 The Order Properties of R 43

It is elementary to know that a real number a is positive or negative


according as a > 0, or a < 0 . In the following discussion we shall define
these using the order properties of R .

3.1 The Order Properties of R

There is a nonempty subset P of R, called the set of positive real


numbers, that satisfies the following properties:

(i) If a, b belong to P, then a + b belongs to P.

(ii) If a, b belong to P, then ab belongs to P.

(iii) If a belongs to R, then exactly one of the following holds:

a ∈ P, a = 0, −a ∈ P.

Remarks

1. The order property (i) ensures the compatibility of order with the
operation of addition, and (ii) ensures the compatibility of order with
the operation of multiplication.

2. The set {−a : a ∈ P} is called the set of negative real numbers.

3. Condition (iii) is usually called the Trichotomy Property, since


it divides R into three distinct types of elements. It states that the
set {−a : a ∈ P} of negative real numbers has no element in common
with the set P of positive real numbers.

4. The set R is the union of the following three disjoint sets: P, {0} ,and
the set of negative real numbers.
44 Chapter 3. The Order Properties of R

Definition 1 If a ∈ P, we say that a is a positive (or a strictly


S
positive) real number and we write a > 0. If a ∈ P {0}, we say that
a is a nonnegative real number and we write a ≥ 0.

If −a ∈ P, we say thata is a negative (or a strictly negative) real


S
number and write a < 0. If −a ∈ P {0}, we say that a is a nonpositive
real number and write a ≤ 0.

We now introduce the notion of inequality between elements of R in


terms of the set P of positive elements.

Definition 2 Let a, b be elements of R.

(i) If a − b ∈ P, then we write a > b or b < a.


S
(ii) If a − b ∈ P {0}, then we write a ≥ b or b ≤ a.

For notational convenience, we shall write

a<b<c

to mean that both a < b and b < c are satisfied. Similarly, if a ≤ b and
b ≤ c, we shall write
a≤b≤c

Also, if a ≤ b and b < d, we shall write

a ≤ b < d.

Rules of Inequalities

We shall now establish some of the basic properties of the order relation
on R; these are usually called rules of inequalities.

Theorem 1 Let a, b, c be elements of R.


3.1 The Order Properties of R 45

(a) If a > b and b > c, thena > c.

(b) Exactly one of the following holds: a > b, a = b, a < b

(c) If a ≥ b and b ≥ a, then a = b.

Proof

(a) If a > b and b > c, then a − b ∈ P and b − c ∈ P, then the order


property (i) implies that (a − b) + (b − c) belongs to P. i.e., a − c ∈ P .
Hence, by (i) of Definition 2, it follows that a > c.

(b) By the field property of R, a, b ∈ R implies a − b ∈ R. Now by the


Trichotomy Property exactly one of the following possibilities occurs:
a − b ∈ P, a − b = 0, −(a − b) = b − a ∈ P.

i.e., exactly one of the following possibilities occurs:

a > b, a = b, a<b

(c) Suppose a ≥ b and b ≥ a. We have to show that a = b . If a 6= b,


then from part (b) we have either a > b or b > a. In either case, one of
the hypotheses is contradicted. Therefore we must have a = b.

It is natural to expect that the natural numbers are positive. We


show next how this property is derived from the basic properties. The
key observation is that the square of any nonzero real number is positive.

Theorem 2 (a) If a ∈ R and a 6= 0, then a2 > 0.

(b) 1 > 0.

(c) If n ∈ N, then n > 0. In particular, 2 > 0.

Proof

(a) By the Trichotomy Property if a 6= 0, then either a ∈ P or −a ∈ P. If


46 Chapter 3. The Order Properties of R

a ∈ P, then by the order property (ii), we have a2 = a · a ∈ P. Similarly,


if −a ∈ P, then again by (ii), we have (−a)(−a) ∈ P. Now, using the
fact that (−1)a = −a and using the associative and commutative laws,
we have

(−a)(−a) = ((−1)a)((−1)a) = (−1)(−1) · a2 = a2 .

Hence (−a) (−a) ∈ P implies that a2 ∈ P.

We conclude that if a 6= 0, then a2 > 0.

(b) Since 1 6= 0, part (a) implies that 1 > 0, since 1 = (1)2 .

(c) We use mathematical induction. The validity of the assertion with


n = 1 is just part (b). If we suppose the assertion is true for the natural
numberk, then k ∈ P. Since 1 ∈ P, we then have k + 1 ∈ P by the order
property (i). Hence, the assertion is true for all natural numbers.

Example 1 Prove that there is no n ∈ N such that 0 < n < 1.

Solution

Let S = {n ∈ N : 0 < n < 1}. We have to show that S is empty,


so that there is no n ∈ N such that 0 < n < 1. If S is not empty, the
Well-Ordering Property of N implies there is a least element m in S.
However, 0 < m < 1 implies that 0 < m2 < m, and since m2 is also in
S we obtain a contradiction to the fact that m is the least element of S.
Hence S is empty.

The following properties relate the order in R to addition and multi-


plication. They provide some of the tools with which we work in dealing
with inequalities.

Theorem 3 Let a, b, c, d be elements of R.


3.1 The Order Properties of R 47

(a) (i) If a > b, then a + c > b + c.

(ii) If a < b, then a + c < b + c.

(b) (i) If a > b and c > d, then a + c > b + d.

(ii) If a < b and c < d, then a + c < b + d.

(c) (i) If a > b and c > 0, then ca > cb

(ii) If a < b and c > 0, then ca < cb

(iii) If a > b and c < 0, then ca < cb

(iv) If a < b and c < 0, then ca > cb


1
(d) (i) If a > 0, then a > 0.
1
(ii) If a < 0, then a < 0.

Proof

(a) (i) If a > b, thena − b ∈ P. Since (a + c) − (b + c) = a − b , this implies


that (a + c) − (b + c) ∈ P. Thus, by (i) of Definition 2, a + c > b + c.

(ii) can be proved similarly.

(b) (i) If a > b and c > d, then a − b ∈ P and c − d ∈ P, then (a + c) −


(b + d) = (a − b) + (c − d) also belongs to P by the order property (i).
Thus, a + c > b + d.

(ii) can be proved similarly.

(c) (i) If a > b and c > 0, then a−b ∈ P and c ∈ P, then ca−cb = c(a−b)
is in P by the order property (ii). Thus ca > cb when c > 0.

(ii) can be proved similarly.

(iii) If c < 0, then −c ∈ P, so that cb − ca =(−c)(a − b) is in P. Thus


cb > ca when c < 0.
48 Chapter 3. The Order Properties of R

(iv) can be proved similarly.


1
(d) (i) If a > 0, then a 6= 0 (by the Trichotomy Property), so that a 6= 0,
1
by Part (a) of Theorem 5 of chapter 2. If < 0, then part (c) of the
a
1
and b = 0 implies that 1 = a a1 < 0,

present theorem with c = a
1
contradicting Part (b) of Theorem 2. Therefore, we have a > 0 since
the other two possibilities have been excluded.
1
(ii) Similarly, if a < 0, then the possibility a > 0 again leads to the
contradiction 1 = a a1 < 0.


Remark Combining Part (c) of Theorem 2 and Part (d) of Theorem


1
3, we see that the reciprocal of any natural number n is positive.
n
Consequently, rational numbers that have the form m 1

n = m n , where
m and n are natural numbers, are positive.

Example 2 If a < b and c ≤ d, then prove that a + c < b + d.

Solution

Case 1) If c = d. Part (a)(ii) of Theorem 3 gives a < b implies a+c < b+c.
Hence in this case a + c < b + d .

Case 2) If c < d. By Part (a)(ii) of Theorem 3, a < b implies a + c <


b + c and c < d implies b + c < b + d. Combining the inequalities we
obtaina + c < b + c < b + d.

Theorem 4 If a and b are in R and if a < b, then a < 21 (a + b) < b.

Proof Since a < b, it follows from Part (a)(ii) of Theorem 3 that

a + a < a + b and a + b < b + b.

i.e., 2a < a + b and a + b < 2b.


3.1 The Order Properties of R 49

Therefore we have
2a < a + b < 2b.

By Part (c) of Theorem 2, we have 2 > 0, and therefore, by Part (d) of


1
Theorem 3, we have 2 > 0. Thus we infer from Part (c) of Theorem 3
that
1
2 (2a) < 12 (a + b) < 12 (2b)

i.e.,
a < 12 (a + b) < b.

Now the result that no smallest positive real number can exist
follows.
1
Corollary If b ∈ R and b > 0, then0 < 2b < b. Hence no smallest
positive real number exists.

Proof Take a = 0 in Theorem 4. This together with the assumption


that 0 < b implies that 0 < 12 (0 + b) < b which implies that 0 < 12 b < b .

Theorem 5 If a ∈ R is such that 0 ≤ a < ε for every positive ε, then


a = 0.

Proof Suppose to the contrary that a > 0. Then it follows from Corol-
1
lary to Theorem 4 (taking b = a) that 0 < 2a < a. Now, if we take
1
ε0 = 2 a, then we have 0 < ε0 < a. This is a contradiction to the fact
that a < ε for every ε > 0. We conclude that a = 0.

Example 3 If a ∈ R is such that 0 ≤ a ≤ ε for every ε > 0, then show


that a = 0.

Solution

Suppose to the contrary that a > 0. Then it follows from Corollary


50 Chapter 3. The Order Properties of R

to Theorem 4 (taking b = a) that 0 < 12 a < a. Now, if we take ε0 = 21 a,


then we have 0 < ε0 < a. This is a contradiction to the fact that a ≤ ε
for every ε > 0. We conclude that a = 0.

Theorem 6 Let a, b ∈ R, and suppose that a − ε < b for every ε > 0.


Then a ≤ b.

Proof Suppose to the contrary that b < a and let ε0 = 21 (a − b). Then
ε0 > 0, and 2ε0 = a − b or b = a − 2ε0 or b < a − ε0 , contrary to the
hypothesis. This completes the proof.

The product of two positive real numbers is again positive. However,


the positivity of a product of two real numbers does not imply that each
factor is positive. The correct conclusion is that the factors must have
the same sign (both positive or both negative), as we show next.

Theorem 7 If ab > 0, then

either (i) a
| > 0 and
{z b > 0} or (ii) a
| < 0 and
{z b < 0} .

Proof First we note that ab > 0 implies that a 6= 0 and b 6= 0 (since


if either a or b is 0, then the product ab is 0). From the Trichotomy
Property, either a > 0 or a < 0.
1
Case (i) If a > 0, then, by Part (d) of Theorem 3, a > 0 and therefore
    
1 1
b=1·b= a b= (ab) > 0.
a a

(The last inequality follows using Part (c) of Theorem 3).

Case (ii) Similarly, if a < 0, then a1 < 0 so that b = a1 (ab) < 0.




Theorem 8 If ab < 0, then


3.1 The Order Properties of R 51

either (i) a
| > 0 and
{z b < 0} or (ii) a
| < 0 and
{z b > 0}

Proof (We use Part (c) of Theorem 3)

Assume that ab < 0. Clearly a 6= 0. Using the Trichotomy Property,


either a > 0 or a < 0.
1
Case (i) If a > 0, then a > 0 and therefore b = 1 · b = a1 (ab) < 0.
1
< 0 and therefore b = 1 · b = a1 (ab) > 0.
Case (ii) If a < 0, then a

Example 4 Show that 8 is not a rational number.

Solution
√ √
If possible, suppose 8 is rational. Then 8 = p/q where p and q
are positive integers that have no common factor other than 1. This

together with the fact that 2 < 8 < 3 implies 2 < pq < 3 implies
2q < p < 3q implies 0 < p − 2q < q . Thus p − 2q is a positive integer
less than q . Then, being the multiple of a non-integer number with an

integer, 8(p − 2q) is not an integer. But
√ p
8(p − 2q) = (p − 2q)
q

p2
= − 2p
q
p2
= q − 2p
q2
= 8q − 2p.

Being the difference of two integers, 8q − 2p is an integer. Hence 8(p −

2q) is an integer. This is a contradiction to the observation that 8(p −

2q) is not an intger. We conclude that 8 is irrational.
52 Chapter 3. The Order Properties of R

3.2 Solving Inequalities

We now show use the order properties established so far to solve certain
inequalities.

Example 5 Determine the set A of all real numbers x such that 3x+2 ≤
6.

Solution

Here
A = {x ∈ R : 3x + 2 ≤ 6}.

We note that

x ∈ A ⇔ 3x + 2 ≤ 6 ⇔ 3x ≤ 4 ⇔ x ≤ 43 .

Therefore, we have A = {x ∈ R : x ≤ 43 }.

[Details of justification of one of the steps:

Using Part (a)(ii) of Theorem 3,

3x + 2 ≤ 6 ⇒ (3x + 2) − 2 ≤ 6 − 2,

Again using Part (a)(ii) of Theorem 3,

2x ≤ 3 ⇒ 2x + 3 ≤ 3 + 3.]

Example 6 Determine the set S = {x ∈ R : x2 + 2x > 3}.

Solution
3.2 Solving Inequalities 53

Note that

x ∈ S ⇔ x2 + 2x > 3 ⇔ x2 + 2x − 3 > 0 ⇔ (x − 1)(x + 3) > 0.

Therefore, by Theorem 7, either we have (i) x − 1 > 0 and x + 3 > 0,


or we have (ii) x − 1 < 0 and x + 3 < 0.

In case (i) we must have both x > 1 and x > −3, which is satisfied if
and only if x > 1.

In case (ii) we must have both x < 1 and x < −3, which is satisfied if
and only if x < −3.

We conclude that
[
S = {x ∈ R : x > 1} {x ∈ R : x < −3}.
n o
2x+1
Example 7 Determine the set T = x ∈ R : x+2 <1 .

Solution

We note that

2x + 1 2x + 1 − (x + 2) x−1
x∈T ⇔ −1<0⇔ <0⇔ < 0.
x+2 x+2 x+2
x−1 1
x+2 < 0 implies (x−1) x+2 < 0 implies (by Theorem 8) either x−1 < 0
1 1
and x+2 > 0 or x − 1 > 0 and x+2 < 0.

Therefore we have either (i) x−1 < 0 and x+2 > 0, or (ii) x−1 > 0
and x + 2 < 0.

In case (i) we must have both x < 1 and x > −2, which is satisfied if
and only if −2 < x < 1.

In case (ii), we must have both x > 1 and x < −2, which is never
54 Chapter 3. The Order Properties of R

satisfied.

Hence we conclude that

T = {x ∈ R : −2 < x < 1}.

Inequalities Involving Square Roots of Postive Numbers

It should be noted that the existence of square roots of strictly pos-


itive real numbers has not been formally established yet; however, we
assume their existence here for the purpose of discussing the examples.
(The existence of square roots will be discussed in the chapter ‘Monotone
sequences’).

Example 8 Let a ≥ 0 and b ≥ 0. Then show that

√ √
a < b ⇔ a2 < b2 ⇔ a< b (3.1)

Solution

When a = 0, the result is obvious. We consider the case where a > 0


and b > 0.

a > 0 and b > 0 implies (from order property (1) of R) that a+b > 0.
i.e., b + a > 0 .

If a < b then b − a > 0; this together with b + a > 0 implies the


product (b − a)(b + a) > 0 . Since b2 − a2 = (b − a)(b + a), it follows
thatb2 − a2 > 0. i.e., b2 > a2 .

Hence a < b ⇒ a2 < b2 is proved.

Now assume b2 − a2 > 0. Then (b − a)(b + a) > 0. Then by Theorem


7, either (b − a) > 0 and (b + a) > 0 or (b − a) < 0 and (b + a) < 0 .
3.2 Solving Inequalities 55

But since a ≥ 0 and b ≥ 0, the possibility of b + a < 0 is ruled out.


The remaining possibility is (b − a) > 0 and (b + a) > 0; in particular,
b − a > 0 implies that a < b.

Hence a2 < b2 ⇒ a < b is proved.


√ √ √ 2
If a > 0 and b > 0, then a > 0 and b > 0. Since a = ( a) and
√ 2 √ √
b= b , the biconditional a < b ⇔ a < b is a consequence of the
√ √
biconditional a < b ⇔ a2 < b2 when a and b are replaced by a and b,
respectively.

Proceeding as in the above example, we obtain the following result:

Result: Show that if a ≥ 0 and b ≥ 0, then

√ √
a ≤ b ⇔ a2 < b2 ⇔ a≤ b (3.2)

Definition If a and b are positive real numbers, then their arithmetic



mean is 12 (a + b) and their geometric mean is ab.

Example 9 The Arithmetic-Geometric Mean Inequality for a, b


is

ab ≤ 21 (a + b) (3.3)

with equality occurring if and only if a = b. Prove this.

Solution

To prove this, note that if a > 0, b > 0, and a 6= b, then1 a > 0,
√ √ √ √ √
b > 0 and a 6= b. Therefore it follows that a − b 6= 0 so that
√ √ √ √
either a − b > 0 or a − b < 0. In either case, using Part (a) of
√ √ 2
Theorem 2, it follows that a − b > 0. Expanding this square, we
obtain

a − 2 ab + b > 0,
56 Chapter 3. The Order Properties of R

and hence it follows that



ab < 12 (a + b).

√ 1
Therefore ab < + b) holds (with strict inequality) when a 6= b.
2 (a
√ √
Moreover, if a = b(> 0), then ab = a2 = a = 12 (a + a) = 21 (a + a),
so that both sides of (3.3) are equal to a, so (3.3) becomes an equality.
This proves that (3.3) holds for a > 0, b > 0.

On the other hand, suppose that a > 0, b > 0 and that ab =
1
2 (a + b). Then, squaring both sides and multiplying by 4, we obtain

4ab = (a + b)2 = a2 + 2ab + b2 ,

and hence it follows that

0 = a2 − 2ab + b2 = (a − b)2 .

But this equality implies that a − b = 0 (for if a − b > 0 then (a − b)2 >
0and if a − b < 0 then(a − b)2 < 0, both are contradiction to the fact
that(a − b)2 = 0). Hence a = b. Thus, equality in (3.3) implies that
1
a = b.

Remark The general Arithmetic-Geometric Mean Inequality for the


positive real numbers a1 , a2 . . . , an is

1 a1 + a2 + · · · + an
(a1 a2 . . . an ) n ≤ (3.4)
n

with equality occurring if and only if a1 = a2 = · · · = an .

1 a 6= b⇒(by the Trichotomy property) either a < b or b < a ⇒(using the previous
√ √ √ √ √ √
example) a < b or b < a ⇒ a 6= b.
3.2 Solving Inequalities 57

Theorem 9 (Bernoulli’s Inequality) If x > −1, then

(1 + x)n ≥ 1 + nx (3.5)

for all n ∈ N.

Proof We apply Principle of Mathematical Induction. The case n = 1


yields equality so that the assertion is valid in this case. Thus, we assume
the validity of the inequality (3.5) for a positive integer k, and shall
deduce it for k + 1. The assumption (1 + x)k ≥ 1 + kx and the fact that
1 + x > 0 imply (using the order property: a ≥ b and c > 0 implies that
ac ≥ bc) that

(1 + x)k+1 = (1 + x)k (1 + x)
≥ (1 + kx)(1 + x)
= 1 + (k + 1)x + kx2
≥ 1 + (k + 1)x.

Thus, the inequality (3.5) holds for n = k + 1. Hence, the inequality


is true for all n ∈ N.

Example 10 If c > 1, show that cn ≥ c for all n ∈ N, and that cn > c


for n > 1.

Solution

Take c = 1 + x. Then, since c > 1, we have 1 + x > 1 and this implies


x > 0 > −1, so by Bernoulli’s Inequality,

(1 + x)n ≥ 1 + nx for all n ∈ N.


58 Chapter 3. The Order Properties of R

For n = 1,
1+x=1+x

i.e., cn = c for n = 1.

For n > 1,

(1 + x)n ≥ 1 + x + (n − 1)x > 1 + x, since (n − 1)x > 0.

i.e., cn > c for n > 1.

Cauchy’s Inequality and The Triangle Inequality

We conclude this chapter by stating two important inequalities. Cauchy’s


Inequality and The Triangle Inequality.

Theorem 10 (Cauchy’s Inequality) If n ∈ N and a1 , . . . , an and


b1 , . . . , bn are real numbers, then:

(a1 b1 + · · · + an bn )2 ≤ a21 + · · · + a2n b21 + · · · + b2n .


 
(3.6)

Moreover, if not all of the bj = 0, then equality holds in (3.6) if and only
if there exists a number s ∈ R such that a1 = sb1 , . . . , an = sbn .

Proof To prove the result, we define a function F : R → R for t ∈ R by

F (t) = (a1 − tb1 )2 + · · · + (an − tbn )2 .

It follows from Part (a) of Theorem 2 that (ai − tbi )2 ≥ 0 for i =


1, 2, . . . , n and using order property (i) of R that F (t) ≥ 0 for all
t ∈ R. If we expand the squares, we obtain

F (t) = A − 2Bt + Ct2 ≥ 0,


3.2 Solving Inequalities 59

where we have set

A = a21 + · · · + a2n , B = a1 b1 + · · · + an bn , C = b21 + · · · + b2n .

Since the quadratic function F (t) is nonnegative for allt ∈ R, it cannot


have two distinct real roots. Therefore its discriminant

∆ = (−2B)2 − 4AC = 4(B 2 − AC)

must satisfy∆ ≤ 0. Consequently we must haveB 2 ≤ AC, which is


precisely (3.6).

If bj = 0 for all j = 1, . . . , n, then equality holds in (3.6) for any choice


of the aj . Suppose now that not all bj = 0 (j = 1, . . . , n). It is readily
seen that if aj = sbj for some s ∈ R and allj = 1, . . . , n, then both
sides of (3.6) equal to s2 (b21 + · · · + b2n )2 . On the other hand, if equality
holds in (3.3), then we must have ∆ = 0 so that there exists a unique
real root s of the quadratic equation F (t) = 0. This implies F (s) = 0
which implies (a1 − sb1 )2 + . . . + ( an − sbn )2 = 0 which implies

a1 − sb1 = 0, . . . , an − sbn = 0

Hence it follows that aj = sbj for all j = 1 , . . . , n.

Theorem 11 (The Triangle Inequality) If n ∈ N and a1 , . . . , an and


b1 , . . . , bn are real numbers, then

1 1 1
(a1 + b1 )2 + · · · + (an + bn )2 2 ≤ a21 + · · · + a2n 2 + b21 + · · · + b2n 2
  

(3.7)
Moreover, if not all of the bj = 0, then equality holds in (3.7) if and only
if there exists a number s such thata1 = sb1 , . . . , an = sbn .
60 Chapter 3. The Order Properties of R

Proof Since (aj + bj )2 = a2j + 2aj bj + b21 forj = 1, . . . , n, it follows from


Cauchy’s Inequality that [if A, B, and C are as in the preceding proof]

B 2 ≤ AC and hence B ≤ AC we have

(a1 + b1 )2 + · · · + (an + bn )2 = A + 2B + C

√ √ √ 2
≤ A + 2 AC + C = A+ C

√ √
Hence using the fact that a ≤ b ⇒ a≤ b, we have

1 √ √
(a1 + b1 )2 + · · · + (an + bn )2 2 ≤ A + C


which is (3.7).

If equality holds in (3.7), then we must have B = AC, so that
equality holds in Cauchy’s Inequality and so that there exists a number
s such that a1 = sb1 , . . . , an = sbn .

Exercises

1. Prove that a2 ≥ 0 for a ∈ R .

2. R+ denotes the set of positive real numbers. Show that there is no


real number x such that x2 + a = 0 for any a ∈ R+ .

3. Show that a ∈ R+ implies a−1 ∈ R+ .

4. If 0 < a < b and 0 ≤ c ≤ d, prove that 0 ≤ ac ≤ bd. Also show by an


example that it does not follow that ac < bd.

5. If a < b andc < d, prove that ad + bc < ac + bd.

6. Find numbers a, b, c, d in R satisfying 0 < a < b and c < d < 0


andac < bd.
3.2 Solving Inequalities 61

7. Find numbers a, b, c, d in R satisfying 0 < a < b and c < d < 0 and


bd < ac.

8. If a, b ∈ R show that a2 + b2 = 0 if and only if a = 0 and b = 0.

9. If 0 ≤ a < b, prove that a2 ≤ ab < b2 . Also show by example that it


does not follow that a2 < ab < b2 .
√ 1
10. If 0 < a < b, then show that a < ab < b and 0 < b < a1 .
1 1
11. If n ∈ N, show that n2 ≥ n and hence n2 ≤ n

12. Find all real numbers x such that

(a) x2 > 1, (b) 1 < x2 < 2

(c) x2 > 3x + 4, (d) 1 < x2 < 4


1 1
(e) x < x (f) x < x2

13. Let a, b ∈ R, and suppose that for every ε > 0 we have a ≤ b + ε.

(a) Show that a ≤ b.

(b) Show that it does not follow that a < b.

14. Prove that [ 21 (a + b)]2 ≤ 1 2


2 (a + b2 ) for all a, b ∈ R. Show that
equality holds if and only if a = b.

15. (a) If 0 < c < 1, show that 0 < c2 < c < 1.

(b) If 1 < c, show that 1 < c < c2 .

16. If c > 1 and m, n ∈ N, show that cm > cn if and only if m > n.

17. If 0 < c < 1, show that cn ≤ c for alln ∈ N.

18. If 0 < c < 1 and m, n ∈ N, show that cm < cn if and only if m > n.

19. If a > 0, b > 0 andn ∈ N, show that a < b if and only if an < bn .
62 Chapter 3. The Order Properties of R

[Hint: Use Mathematical Induction]


1 1
20. Assuming the existence of roots, show that if c > 1, then c m < c n
if and only if m > n.

Answers

2. Hint: Ref. Exercise 1

8. If a 6= 0, then a2 > 0 : since b2 ≥ 0, it follows that a2 + b2 > 0.


√ √ √
10. If 0 < a < b, then 0 < a2 < ab < b2 . Then a = a2 < ab < b2 =
b.

12. (a) {x : x > 1 or x < −1}


√ √
(b) {x : 1 < x < 2 or − 2 < x < −1}

(c) {x : x > 4 or x < −1}

(d) {x : 1 < x < 2 or −2 < x < −1}

(e) {x : −1 < x < 0 or x > 1}

(f) {x : x < 0 or x > 1}.

14. The inequality is equivalent to 0 ≤ a2 − 2ab + b2 = (a − b)2

15. Use the Results: “If a > b and c > 0, then ca > cb”

“ If a > b and c < 0, then ca < cb ”

16. Ifm > n, then k = m − n ∈ N and ck ≥ c > 1 which implies that


cm > cn . Conversely, the hypotheses that cm > cn and m ≤ n lead to a
contradiction.

20. Let b = c1/mn and show that b > 1. This implies that c1/n = bn =
c1/m if and only if m > n.
Chapter 4
Absolute Value and the Real
Line

Introduction

Sometimes, it is useful to restrict our attention over non-negative real


numbers only. For this purpose, we define non-negative value of a real
number which we call an absolute value or modulus of the real number.
From the Trichotomy Property1 we are assured that if a ∈ R and a 6= 0,
then exactly one of the numbers a and −a is positive. The absolute
value of a 6= 0is defined to be the positive one of these two numbers.
The absolute value of 0 is defined to be 0.

1 By the Trichotomy Property, for any two real numbers a and b either one of

the following holds: a < b, a = b, or a > b. Now take b = 0 . Then a < 0, a =


0, or a > 0.

63
64 Chapter 4. Absolute Value and the Real Line

4.1 Absolute Value and the Real Line

Definition If a ∈ R, the absolute value of a, denoted by |a|, is defined


by 


 a if a>0
|a| = 0 if a=0

 −a

if a<0

For example, |3| = 3 and | − 2| = 2. From the definition it is obvious


that |a| ≥ 0 for all a ∈ R. Also |a| = a if a ≥ 0, and |a| = −a if a ≤ 0.
We note that:

|a| ≥ a for any a ∈ R .

Theorem 1

(a) |a| = 0 if and only if a = 0.

(b) | − a| = |a| for all a ∈ R.

(c) |ab| = |a| |b| for all a, b ∈ R.


2
(d) |a| = a2 for all a ∈ R

(e) If c ≥ 0, then |a| ≤ c if and only if −c ≤ a ≤ c.

(f) −|a| ≤ a ≤ |a| for all a ∈ R.

Proof

(a) If a = 0, then by the definition of absolute value of a real number,


|a| = 0. Next to show that |a| = 0 ⇒ a = 0 , we prove that a 6= 0 ⇒
|a| 6= 0. [This is enough as p ⇒ q ≡ ¬q ⇒ ¬p, by the contrapositive
4.1 Absolute Value and the Real Line 65

method of proving theorems]. Now, if a 6= 0, then −a 6= 0, so that2


|a | =
6 0

Thus if |a| = 0, thena = 0..

(b) If a = 0, then |0| = 0 = | − 0|. If a > 0, then −a < 0


so that |a| = a = −(−a) = | − a|. If a < 0, then −a > 0 so that
|a| = −a = | − a|.

(c) If either a or b is 0, then both |ab| and |a| |b| are equal to 0.

If a > 0 and b > 0, then ab > 0, and |ab| = ab = |a| |b| .

If a > 0 and b < 0, then ab < 0, so that

|ab| = −ab = a(−b) = |a| |b|.

The case of a < 0 and b > 0 can be treated similarly.



(d) a2 = a2 , since a2 ≥ 0,

= |aa| = |a| |a| , using part (c) above,


2
= |a| . (e) Suppose that|a| ≤ c.

Case 1) If a ≥ 0, then |a| = a. Hence |a| ≤ c implies a ≤ c· This


together with −a ≤ a implies −a ≤ a ≤ c.

Case 2) If a < 0, then −a > 0 and a < −a, and |a| = −a. Hence |a| ≤ c
implies −a ≤ c. This together with a < −a implies a ≤ −a ≤ c.

Hence in any case we have both a ≤ c and −a ≤ c. The latter inequality

2 Details:From the definition, if a 6= 0 the possibility of |a| is as follows:



a if a>0
|a| =
−a if a < 0
66 Chapter 4. Absolute Value and the Real Line

is −a ≤ c ⇒ (−1)(−a) ≥ (−1)c ⇒ a ≥ −c implies −c ≤ a, we have


−c ≤ a ≤ c. Conversely, if −c ≤ a ≤ c, then we have both a ≤ c and
−c ≤ a. That is, both a ≤ c and −a ≤ c so that |a| ≤ c.

(f) Take c = |a| in part (e).

The following inequality will be used frequently.

Theorem 2 (Triangle Inequality) For any a and b in R

|a + b| ≤ |a| + |b|

Proof From (f) above, we have −|a| ≤ a ≤ |a| and−|b| ≤ b ≤ |b|.


Then, adding and using the result “if p > q and r > t, then p+r > q +t”,
we obtain
−(|a| + |b|) ≤ a + b ≤ |a| + |b|.

Hence, with c = |a| + |b| ,we have |a + b| ≤ |a| + |b| by (e) above.

Corollary 1 For any a and b in R

(a)| |a| − |b| | ≤ | a − b | .

(b) |a − b| ≤ |a| + |b|.

Proof

(a) We write a = a − b + b. Then

|a| = |(a − b) + b|

≤ |a − b| + |b|,

applying the Triangle Inequality.


4.1 Absolute Value and the Real Line 67

Adding − |b| to both sides of the inequality yields,

|a| − |b| ≤ |a − b|. (4.1)

Similarly, from |b| = |b − a + a| ≤ |b − a| + |a|, we have

|b| − |a| ≤ |b − a|

which gives
− |b − a| ≤ |a| − |b| ,

and hence

− |a − b| = −| − (b − a)|
= −|b − a|, using Part (b) of Theorem 1
≤ |a| − |b|. (4.2)

If we combine inequalities (4.1) and (4.2), using (e) of Theorem 1 with


c = |a − b|, we obtain the inequality

| |a| − |b| | ≤ | a − b | .
68 Chapter 4. Absolute Value and the Real Line

Aliter Proof of (a):

2
|a − b| = (a − b)2
= a2 + b2 − 2ab
≥ a2 + b2 − 2 |ab| , since |ab| ≥ ab
and hence − |ab| ≤ −ab
= a2 + b2 − 2 |a| |b| , since |ab| = |a| |b|
2 2
= |a| + |b| − 2 |a| |b|
2
= (|a| − |b|)
2
= | |a| − |b| | . (4.3)

√ √
Now using the result “if a ≥ 0 and b ≥ 0, then a ≥ b ⇔ a≥ b ” [Ref.
Chapter Order Properties of R] and noting that, here |a − b| ≥ 0 and
| |a| − |b| | ≥ 0, inequality (4.3) gives

| a − b | ≥ | |a| − |b| | ,

(b) Replacing b in the Triangle Inequality |a + b| ≤ |a| + |b| by −b we


get |a − b| ≤ |a| + | − b|. Since | − b| = |b| by Part (b) of Theorem 1,
we obtain the inequality in (b).

Corollary 2 [Triangle Inequality to any finite number of ele-


ments of R] For any a1 , a2 , . . . , an in R, we have

|a1 + a2 + · · · + an | ≤ |a1 | + |a2 | + · · · + |an |.

Proof We prove by mathematical induction with base value n0 = 2.


By the Triangle Inequality (Theorem 2), the result is true for n = 2. As
4.1 Absolute Value and the Real Line 69

induction hypothesis, suppose the inequality is true for k ≥ 2. i.e.,

|a1 + a2 + · · · + ak | ≤ |a1 | + |a2 | + · · · + |ak |

Now



|a1 + a2 + · · · + ak + ak+1 | = a1 + a2 + · · · + ak + ak+1

| {z } | {z }

≤ |a1 + a2 + · · · + ak | + |ak+1 |, using Theorem 2

≤ |a1 | + |a2 | + · · · + |ak | + |ak+1 |, using the induction hypothesis.

Hence the inequality is true for k + 1. Hence by mathematical induction,


the inequality is true for n ∈ N .

The following examples illustrate how the preceding properties of abso-


lute value can be used.

Corollary 3 For any a and b in R we have

|a| + |b| ≥ |a + b| .

Proof
2 2 2
(|a| + |b| ) = |a| + |b| + 2 |a| · |b|

= a2 + b2 + 2 |ab|

≥ a2 + b2 + 2ab, since |ab| ≥ ab

= (a + b)2

2
= |a + b|
70 Chapter 4. Absolute Value and the Real Line

√ √
Now using the result “if a ≥ 0 and b ≥ 0, then a ≥ b ⇔ a≥ b ” the
above gives
|a| + |b| ≥ |a + b| .

Example 1 Determine the set A of all real numbers x that satisfy


|2x + 3| < 6.

Solution

Here A = {x ∈ R : |2x + 3| < 6}.

x ∈ A if and only if |2x + 3| < 6 if and only if [by Part (e) of Theorem
1] −6 < 2x + 3 < 6 if and only if −9 < 2x < 3, which is satisfied if and
only if (Dividing by 2) − 29 < x < 32 . We conclude that

A = {x ∈ R : − 92 < x < 32 }.

Example 2 Determine the set B = {x ∈ R : |x − 1| < |x|}

Solution

We have the inequality

|x − 1| < |x |. (4.4)

There are three cases where the absolute value symbols can be removed,
and they are (i) x ≥ 1, (ii) 0 ≤ x < 1, (iii) x < 0.

Case (i) x ≥ 1 implies x − 1 ≥ 0 and x > 0, so the inequality (4.4)


becomes x − 1 < x, which is satisfied without any further restriction.
Therefore all x satisfying x ≥ 1 belong to the setB.

Case (ii): 0 ≤ x < 1 implies x − 1 < 0, so |x − 1| = −(x − 1), and


hence the inequality (4.4) becomes −(x − 1) < x, or −x + 1 < x, or
1 < 2x which imposes the further restriction that x > 21 . Thus, case (ii)
4.1 Absolute Value and the Real Line 71

1
contributes all x satisfying 2 < x < 1 to the set B.

Case (iii): |x − 1| = −(x − 1) and |x| = −x, hence the inequality (4.4)
becomes−(x−1) < −x, which is equivalent to1 < 0. Since this statement
is always false, no value of x covered by case (iii) satisfies the inequality.
Combining the three cases, we conclude that

B = {x ∈ R : x > 12 }.

Aliter: There is an alternative method of determining the set B based


on the fact that a < b if and only if a2 < b2 when a ≥ 0 and b ≥ 0. Thus,
the inequality |x − 1| < |x| is satisfied if and only if |x − 1|2 < |x|2 .
2
Hence we have (x − 1)2 < x2 and hence x2 − 2x + 1 < x2 , which on
simplification becomes x > 12 . Thus, we again find that B = {x ∈ R :
x > 12 }. 3

Example 3 Suppose the function f is defined by


2x2 −3x+1
f (x) = 2x−1 for 2 ≤ x ≤ 3.

Find a constant M such that |f (x)| ≤ M for all x satisfying 2 ≤ x ≤ 3.

Solution

We consider separately the numerator and denominator of

|2x2 − 3x + 1|
|f (x)| = .
|2x − 1|

3 Noting that |a|2 = a2 for any a, as by the definition


 2
a if a≥0
|a|2 = .
(−a)2 if a < 0
72 Chapter 4. Absolute Value and the Real Line

From the Triangle Inequality, we obtain

|2x2 −3x+1| ≤ 2x2 +|−3x|+1 = 2|x|2 +3|x|+1 ≤ 2·32 +3·3+1 = 28


since |x| ≤ 3 for the x under consideration. Also,

|2x − 1| ≥ |2x | − |1 | = 2|x| − 1 ≥ 2 · 2 − 1 = 3

1 1
since |x| ≥ 2 for the x under consideration. Thus, |2x−1| ≤ 3 for
28
x ≥ 2. Therefore, for 2 ≤ x ≤ 3 we have |f (x)| ≤ 3 . Hence we can
28
take M = 3 . (Note that we have found one such constant M ; evidently
28
any number M > 3 will also satisfy |f (x)| ≤ M . It is also possible
28
that 3 is not the smallest possible choice for M .)

The Real Line

The real number system has a convenient geometric representation and


it is the real line. In this interpretation, the absolute value |a| of an
element a in R is regarded as the distance from a to the origin 0. More
generally, the distance between elements a and b in R is |a − b|.

If a is a given real number, then saying that a real number x is “close


to” a should mean that the distance |x−a| between them is “small”. This
idea can be discussed by the terminology of neighborhoods. Definition
follows.

Figure 4.1:
4.1 Absolute Value and the Real Line 73

Definition Let a ∈ R and ε > 0. Then the ε-neighborhood of a is the


set Vε (a) = {x ∈ R : |x − a| < ε}. [Fig. 4.1].

For a ∈ R, the statement that x belongs to Vε (a) is equivalent to the


statement
−ε < x − a < ε.

It is also equivalent to the statement

a−ε<x<a+ε

Theorem 3 Let a ∈ R. If x belongs to the neighborhood Vε (a) for every


ε > 0, then x = a.

Proof If x belongs to the neighborhood Vε (a) for every ε > 0, then


|x − a| < ε for every ε > 0, and hence using the result “ if a ∈ R such
that 0 ≤ a < ε ∀ ε > 0, then a = 0” we obtain |x − a| = 0, and hence
x − a = 0, that is, x = a.

Example 4 Let A = {x : 0 < x < 1}. Show that each element of A has
some ε-neighborhood of it contained inA.

Solution

Leta ∈ A. We have to show that there exists ε > 0 (depending on


the choice of a) such that Vε (a) ⊆ A. Now let

ε = min{a, 1 − a}.

Claim: Vε (a) ⊆ A.
1 1
We first note that a ∈ A implies 0 < a ≤ 2 (Fig. 4.2) or 2 ≤ a < 1
(Fig. 4.3). In the former case we have ε = min{a, 1 − a} = a and in
the latter case ε = min{a, 1 − a} = 1 − a.
74 Chapter 4. Absolute Value and the Real Line

Figure 4.2:

Figure 4.3:

Case 1) If ε = a, then

x ∈ Vε (a) ⇒ |x − a| < ε ⇒ |x − a| < a

⇒ −a < x − a < a ⇒ 0 < x < 2a

⇒ 0 < x < 1(as in this case a ≤ 12 )

⇒x∈ A

Case 2) If ε = 1 − a, then

x ∈ Vε (a) ⇒ |x − a| < ε ⇒ |x − a| < 1 − a

⇒ −(1 − a) < x − a < 1 − a ⇒ a − 1 < x − a < 1 − a

⇒ 2a − 1 < x < 1
1
⇒ (since 2 < a, 2a − 1 > 0, so) 0 < x < 1 ⇒ x ∈ A.
4.1 Absolute Value and the Real Line 75

Thus Vε (a) is contained inA. Since the choice of a ∈ A was arbitrary, we


conclude that each element of A has some ε-neighborhood of it contained
in A.

Example 5 If I = {x : 0 ≤ x ≤ 1}, then for anyε > 0, the ε-


neighborhood Vε (0) of 0 contains points not in I, and so Vε (0) is not
contained inI. For example, the number xε = − 2ε is in Vε (0) but not
inI.

Figure 4.4:

Example 6 Show that if x, y belong to the ε-neighborhoods of a, b


respectively, then x + y belongs to the 2ε-neighborhood of a + b.

Solution

If x, y belong to the ε-neighborhoods of a, b, then |x − a| < ε and


|y − b| < ε. Now

|(x + y) − (a + b)| = |(x − a) + (y − b)|


≤ |x − a| + |y − b| by the Triangle Inequality
< ε + ε = 2ε.

Thus x + y belongs to the 2ε-neighborhood of a + b

Attention! If x, y belong to the ε-neighborhoods of a, b respectively,


then x + y not necessarily belong to the ε-neighborhood of a + b. For
example, both 0.7 and 0.5 belong to the 1- neighborhood of 0, but 0.7 +
76 Chapter 4. Absolute Value and the Real Line

0.5 = 1.2 doesn’t belong to the 1- neighborhood of 0 + 0 = 0. (Fig.5).

Figure 4.5:

Example 7 Show that if a, b ∈ R, then

max {a, b} = 12 (a + b + |a − b|) and min {a, b} = 21 (a + b − |a − b|)

Solution

Case 1) Suppose that a ≤ b. Then

1. max{a, b} = b. Also,

1 1
(a + b + |a − b|) = [a + b − (a − b)] = b.
2 2

2. min {a, b} = a. Also,

1 1 1
(a + b − |a − b|) = {a + b − [−(a − b)]} = {a + b + a − b = a.
2 2 2

Case 2) Suppose that b ≤ a. Then

1. max{a, b} = a. Also,

1 1
(a + b + |a − b|) = [a + b + (a − b)] = a.
2 2
4.1 Absolute Value and the Real Line 77

2. min {a, b} = b. Also,

1 1
(a + b − |a − b|) = [a + b − (a − b)] = b.
2 2

This proves the result.

Example 8 Show that if a, b, c ∈ R,, then the “middle number” is


mid {a, b, c} = min {max {a, b} , max {b, c} , max {c, a}} .

Solution

Case 1) If a ≤ b ≤ c, then mid{a, b, c} = b.

Also, b = min {b, c, c}

= min {max{a, b}, max{b, c}, max{c, a}}

Hence in this case

mid {a, b, c} = min {max {a, b} , max {b, c} , max {c, a}} .

The other cases are similar.

Exercises

1. Let a ∈ R. Show that



(a) |a| = a2 , (b)|a2 | = a2 .
|a|

2. If a, b ∈ R and b 6= 0, show that ab = |b| .

3. If a, b ∈ R, show that |a + b| = |a| + |b| if and only if ab ≥ 0.

4. If x, y, z ∈ R, x ≤ z, show that x ≤ y ≤ z if and only if


|x − y| + |y − z| = |x − z|. Interpret this geometrically.

5. Find all x ∈ R that satisfy the equation |x + 1| + |x − 2| = 7.


78 Chapter 4. Absolute Value and the Real Line

6. Find all x ∈ R that satisfy the following inequalities:

(a). |x − 1| < 1

(b). |x − 1| ≤ 2

(c). |4x − 5| ≤ 13,

(d). |x2 − 1| ≤ 3,

(e). |x − 1| > |x + 1|,

(f). |x| + |x + 1| < 2

(g). |x − 1| + |x + 2| ≤ 1.

7. Show that |x − a| < ε if and only if a − ε < x < a + ε.

8. If a < x < b anda < y < b, show that|x − y| < b − a. Interpret this
geometrically.

9. Determine and sketch the set of pairs (x, y) in R × R that satisfy:

(a) |x| = |y|, (b) |x| + |y| = 1

(c) |xy| = 2, (d) |x| − |y| = 2

10. If x, y refer to rectangular coordinates, show that the equation


|x| + |y| = 4 represents sides of a square of area 32.

11. Determine and sketch the set of pairs (x, y) in R × R that satisfy
the inequalities:

(a) |x| ≤ |y|, (b) |x| + |y| ≤ 1

(c) |xy| ≤ 2, (d) |x| − |y| ≤ 1

12. Find all x ∈ R the satisfy the inequality 4 < |x + 2| + |x − 1| < 5.


4.1 Absolute Value and the Real Line 79

13. Find all x ∈ R the satisfy both |2x − 3| < 5 and |x + 1| > 2
simultaneously.

14. Let ε > 0 and δ > 0, and let a ∈ R. Show that Vε (a) ∩ Vδ (a) and
Vε (a) ∪ Vδ (a) are γ-neighborhoods of a for appropriate values of γ.

15. Show that ifa, b ∈ R, and a 6= b, then there exist ε-neighborhoods


U of a and V of b such that U ∩ V = Φ.

16. Show that if a, b, c ∈ R, then

min {a, b, c} = min {min {a, b} , c} .

17. For all real numbers x and y, show that

|x + y| |x| |y|
≤ +
1 + |x + y| 1 + |x| 1 + |y|

Answers
√ √
1. (a) If a ≥ 0, then |a| = a = a2 ; if a < 0, then |a| = −a = a2 .

2. It suffices to show that |1/b| = 1/|b| for b 6= 0. Consider the cases


b > 0 and b < 0.

3. If x ≤ y ≤ z, then

|x − y| + |y − z| = (y − x) + (z − y) = z − x = |z − x|. To establish
the converse, show that y < x and y > z are impossible. For example, if
y < x ≤ z, it follows from what we have shown and the given relationship
that |x − y| = 0, so that y = x, a contradiction.

5. x = 4 or x = −3

6. (a) 0 < x < 2 (b) −1 ≤ x ≤ 3

(c) −2 ≤ x ≤ 9/2 (d) −2 ≤ x ≤ 2


80 Chapter 4. Absolute Value and the Real Line

(e) x < 0 (f) −3/2 < x < 1/2

7. |x−a| < ε if and only if −ε < x−a < ε if and only if a−ε < x < a+ε

9. (a) {(x, y) : y = ±x} (c) The hyperbolas y = 2/x andy = −2/x.

11. (a) If y ≥ 0, then −y ≤ x ≤ y and we get the region in the upper


half-plane on or between the lines y = x and y = −x.

12. {x : −3 < x < −5/2 or 3/2 < x < 2}

13. {x : 1 < x < 4}

17.

|x + y| 1
= 1−
1 + |x + y| 1 + |x + y|
1
≤ 1−
1 + |x| + |y|
|x| + |y|
=
1 + |x| + |y|
|x| |y|
≤ +
1 + |x| 1 + |y|
Chapter 5
The Completeness Property of
R

In this chapter we consider the completeness property of R; this


property guarantees the existence of elements in R under certain
hypotheses. The system Q of rational numbers satisfies both the al-
gebraic properties and the order properties of R but we have seen that
√ √
2 cannot be represented as a rational number; therefore 2 does not
belong to Q. This observation shows the necessity of an additional prop-
erty to characterize the real number system. This additional property,
the Completeness (or the Supremum) Property, is an essential feature of
R. R is a complete ordered field.

81
82 Chapter 5. The Completeness Property of R

5.1 Suprema and Infima

For the discussion of suprema and infima we need the notions of upper
bound and lower bound for a set of real numbers.

Definition Let S be a subset of R.

(i) A number u ∈ R is said to be an upper bound of S if s ≤ u for all


s ∈ S.

(ii) A number w ∈ R is said to be a lower bound of S if w ≤ s for all


s ∈ S.

From the definition of upper bound, the following two statements about
a number u and a set S are equivalent:

(i) u is an upper bound of S,

(ii) s ≤ u for all s ∈ S.

Criteria for a Number not to be an Upper/Lower Bound

A number v ∈ R is not an upper bound of S if and only if there exists


some s0 ∈ S such thatv < s0 . A number z ∈ R is not a lower bound of
S if and only if there exists some s00 ∈ S such that s00 < z.

Remarks

1. A subset S of R may not have an upper bound (for example, take


S = R). [See Exercise 1].

2. If S has one upper bound, then it will have infinitely many upper
bounds because if u is an upper bound of S, then any v such that
5.1 Suprema and Infima 83

u < v is also an upperbound of S. For example, A = {x ∈ R : x ≤


1} has 1 an upper bound. Any real number greater than 1 is also
an upper bound of A.

3. It is possible for a set to have lower bound but no upper bound


(and vice versa). For example, the set A = {x ∈ R : x ≥ 0} has
a lower bound (actually infinitely many lower bounds) but has no
upper bound; the set B = {x ∈ R : x < 0} has no lower bound but
infinitely many upper bounds.

4. If u is not an upper bound a set S, then v < u cannot be an upper


bound of S.

5. If w is not a lower bound a set S, then v > w cannot be a lower


bound of S.

6. A number u ∈ R is not an upper bound of a set S if there is an


element s0 ∈ S such that u < s0 .

The Upper and Lower Bounds of the Empty Set

1. Every real number is an upper bound of the empty set Φ.

2. Every real number is a lower bound of the empty set.

Example 1 Show that every real number is an upper bound of the


empty set Φ.

Solution

We first note that if a number u ∈ R is not an upper bound of a set


S, then an element s0 ∈ S must exist such that u < s0 .
84 Chapter 5. The Completeness Property of R

Let u ∈ R. As there is no element in S = Φ, it is impossible to find


s ∈ S such that u < s0 . Hence u ∈ R is an upper bound of S = Φ .
0

Definitions

1. A set in R is bounded above if it has an upper bound.

2. If a set in R has a lower bound, we say it is bounded below.

3. If a set in R has both an upper bound and a lower bound, we say


it is bounded.

4. We say that a set in R is unbounded if it lacks either an upper


bound or a lower bound. For example, the set {x ∈ R : x ≤ 5}
is unbounded (even though it is bounded above) since it is not
bounded below.

Definition Let S be a subset of R.

(i) If S is bounded above, then an upper bound u is said to be a supre-


mum (or a least upper bound) of S if no number smaller than u
is an upper bound of S. i.e., u is a supremum of S if it satisfies the
conditions:

(a) u is an upper bound of S, and

(b) if v is any upper bound of S, then u ≤ v.

(ii) If S is bounded below, then a lower bound w is said to be an infimum


(or a greatest lower bound) of S if no number larger than w is a lower
bound of S. i.e., w is an infimum of S if it satisfies the conditions:

(a) w is a lower bound of S, and


5.1 Suprema and Infima 85

(b) if t is any lower bound of S, then t ≤ w.

Figure 5.1:

There are four possibilities for a nonempty subset S of R: it can

(i) have both a supremum and an infimum [for example, S = {x ∈ R :


3 ≤ x ≤ 5}]

(ii) have a supremum but no infimum [for example, {x ∈ R : x ≤ 5}]

(iii) have an infimum but no supremum [for example, {x ∈ R : x ≥ 5}]

(iv) have neither a supremum nor an infimum. [for example, R itself.


Two more examples are, {x ∈ R : x ∈
/ N} and Z , the set of integers].

Attention! As noted earlier, every real number is an upper bound for


the empty set, so that the empty set has no supremum. Similarly it has
no infimum.

Theorem 1 [Uniqueness of Supremum] There can be only one supre-


mum of a given subset S of R, if it exists. (Then we can refer to the
supremum of a set instead of a supremum.)

Proof Suppose that u1 and u2 are suprema of S; then they are both
upper bounds of S. If u1 < u2 , then the hypothesis that u2 is a supre-
mum implies that u1 cannot be an upper bound of S. Therefore, we
86 Chapter 5. The Completeness Property of R

must have u1 = u2 . This completes the proof.

Proceeding in a similar manner, we obtain the following result.

Theorem 2 There can be only one infimum of a given subset of R.

Notation When the supremum of a set S exist, we shall denote them


by
sup S

and the infimum by


inf S.

Attention! sup S and inf S are real numbers, while S is set of real
numbers.

Remark If u0 is an upper bound of a set S, then

sup S ≤ u0 .

This is because sup S is the least of the upper bounds of S.

The criterions in the following two Lemmas are often useful in establish-
ing that a certain upper bound of a given set is actually the supremum
of the set.

Lemma 1 A number u is the supremum of a nonempty subset S of R if


and only if u satisfies the two conditions:

1. s ≤ u for all s ∈ S;

2. if v < u, then there exists an s0 ∈ S such that v < s0 .

Proof Condition (1) says that u is an upper bound, and condition (2)
says that no number less than u is an upper bound. Hence the result
follows immediately from the definition of supremum.
5.1 Suprema and Infima 87

Lemma 2 An upper bound u of a nonempty set S in R is the supremum


of S if and only if for each ε > 0 there exists an sε ∈ S such that
u − ε < sε .

Proof Suppose that u is an upper bound of S that satisfies the stated


condition that for each ε > 0 there exists an sε ∈ S such that u − ε < sε .
If v < u and we takeε = u − v, then ε > 0, and the stated condition
implies there exists a number sε ∈ S such thatv = u − ε < sε . Therefore,
v is not an upper bound of S. Since v is an arbitrary number smaller
thanu, we have no number smaller than u is an upperbound of S and so
we conclude that u = sup S.

Conversely, suppose that u = sup S and let ε > 0. Since u − ε < u,


then u − ε is not an upper bound of S. Therefore, some element sε of S
must be greater than u − ε; that is, u − ε < sε . (See Fig. 5.2).

Figure 5.2:

Now we summarise the above discussion.

The following statements about an upper bound u of a set S are


equivalent:

(1) if v is any upper bound of S, then u ≤ v (i.e., u is the supremum of


S)

(2) if z < u, then z is not an upper bound of S


88 Chapter 5. The Completeness Property of R

(3) if z < u, then there exists sz ∈ S such that z < sz

(4) if ε > 0, then there exists sε ∈ S such that u − ε < sε

Remark The supremum of a set may or may not be an element of the set
under consideration. Sometimes it is and sometimes it is not, depending
on the particular set. This is illustrated in the following examples.

Example 2 Show that if a nonempty set S has finite number of elements,


then sup S and inf S exist and both belong to the set S.

Solution

Using mathematical induction, we show that S has a largest element


and a least element.

If S is a set having only one element, say, u, then u is the largest and
least element of S.

As induction hypothesis, assume that every nonempty subset having


k elements has largest element and least element.

Let S = {x1 , x2 , . . . , xk , xk+1 } be a set having k + 1elements.


Then, by induction hypothesis, max{x1 , x2 , . . . , xk } = a(say) and
min{x1 , x2 , . . . , xk } = b(say) exist and both belong to S. Hence

max{x1 , x2 , . . . , xk , xk+1 } = max{a, xk+1 }


| {z }

and
min{x1 , x2 , . . . , xk , xk+1 } = min{b, xk+1 }
| {z }

and both belong to S.


5.1 Suprema and Infima 89

Hence a nonempty set S having n number of elements (n ∈ N) has sup S


and inf S exist and both belong to the set S.

Example 3 If I = {x : 0 ≤ x ≤ 1}. Then prove that sup I = 1 and


inf I = 0.

Solution

Clearly 1 is an upper bound and 0 is a lower bound for I.

1. If v < 1, there exists an element s0 ∈ I such that v < s0 . Hence


v is not an upper bound for I and, since v is an arbitrary number
that is less than 1, we have that no number less than 1 is an upper
bound of I and hence conclude that sup I = 1.

2. Proving inf I = 0 is left as an exercise.

Note that both sup I and inf I are elements of the set I.

Example 4 Give an example of a set having infimum and supremum


but not belong to the set.

Solution

The set B = {x : 0 < x < 1} clearly has 1 for an upper bound.


Using the same argument given in Example 2 for the setA, we see that
sup B = 1. In this case, the set B does not contain sup B = 1. Similarly,
inf B = 0, which is not contained in B.

The Completeness Property of R

Now we are ready to observe that every nonempty subset of R that


is bounded above has a supremum. The following statement, which is
also called the completeness Property of R, is our final assumption
aboutR. Thus we say that R is a complete ordered field.
90 Chapter 5. The Completeness Property of R

The Supremum Property (or Completeness Property) of R Ev-


ery nonempty set of real numbers that has an upper bound has a supre-
mum in R.

The analogous property of infima can be deduced from the Supremum


Property.

The Infimum Property of R Every nonempty set of real numbers


that has a lower bound has an infimum in R.

Proof (Using the Supremum property of R) Suppose that S is a nonempty


subset of R that is bounded below. Then the set S 0 = {−s : s ∈ S} is
bounded above, and the Supremum Property (Completeness Property)
implies that u = sup S 0 exists. It then follows that −u is the infimum
of S. [Details: u = sup S 0 implies −s ≤ u for s ∈ S, implies −u ≤ s
for s ∈ S, implies −u is a lower bound of S. There is no −v such that
−u < −v ≤ s for s ∈ S as then −s ≤ v < u, a contradiction to the fact
u = sup S 0 . Hence −u− = inf S.

From the proof of the infimum property of R the following result


follows:

Proposition Let S be a nonempty subset of R that is bounded below.


Then inf S = − sup{−s : s ∈ S}.

Exercises

1. Give three subsets of R that have no upper bounds.

2. Give three subsets of R that have no lower bounds.

3. Let S = {x ∈ R : x ≥ 0}. Show in detail that the set S has lower


bounds, but no upper bounds. Show that inf S = 0.

4. Let A = {x ∈ R : x > 0}. Does A have lower bounds? Does A


5.1 Suprema and Infima 91

have upper bounds? Does inf A exist? Does sup A exist? Prove your
statements.
1
5. Let B = n : n ∈ N . Show that sup B = 1 andinf B ≥ 0. [It will
follow easily from the Archimedean Property that will be discussed in
chapter 7 thatinf B = 0.]
n n
o
6. Let S = 1 − (−1)n : n ∈ N . Find inf S and sup S.

7. If a set S ⊆ R contains one of its upper bounds, show that this upper
bound is the supremum of S.

8. Let S ⊆ R be nonempty. Show that u ∈ R is an upper bound of S if


and only if the conditions t ∈ R and t > u imply that t ∈
/ S.

9. Let S ⊆ R be nonempty. Show that if u = sup S,then for every


1
number n ∈ N the number u − n is not an upper bound of S, but the
1
number u + is an upper bound of S. (The converse is also true.
n
S
10. Show that if A and B are bounded subsets of R, then A B is a
S
bounded set. Show that sup(A B) = sup{sup A, sup B}.

11. Let S be a bounded set in R and let S0 be a nonempty subset of S.


Show that inf S ≤ inf S0 ≤ sup S0 ≤ sup S.

12. Let S ⊆ R and suppose that s∗ = sup S belongs to S. If u ∈


/ S,
S
show thatsup(S {u}) = sup{s∗, u}.

13. Show that a nonempty finite set S ⊆ R contains its supremum.

Answers

1. (i) S = R is a subset of R that has no upper bound. [If possible,


suppose there is a number u ∈ R that is an upper bound of S = R;
then s ≤ u for alls ∈ S = R. But u + 1 ∈ S = R, and u + 1 > u , a
contradiction]
92 Chapter 5. The Completeness Property of R

(ii)T = N , the set of natural numbers, is a subset of R that has no upper


bound.

(iii)U = Q , the set of rational numbers, is a subset of R that has no


upper bound.

2. (i) S = R is a subset of R that has no lower bound. [If possible,


suppose there is a number w ∈ R that is a lower bound of S = R; then
w ≤ s for all s ∈ S = R. But w − 1 ∈ S = R, and w − 1 < w , a
contradiction]

(ii) V = { . . . , −3, −2, −1} , the set of rational numbers, is a subset


of R that has no lower bound.

(iii) T = Q , the set of rational numbers, is a subset of R that has no


lower bound.

3. Since 0 ≤ x for all x ∈ S, then u = 0 is a lower bound of S. If


v > 0, then v is not a lower bound of S because v/2 ∈ S and v/2 < v.
Therefore inf S = 0.

5. Since 1/n ≤ 1 for all n ∈ N, then 1 is an upper bound forB.

6. sup S = 2 andinf S = 1/2.

8. Let u ∈ S be an upper bound of S. If v is another upper bound of S,


then u ≤ v. Hence u = sup S.

10. Let u = sup A, v = sup B and w = sup{u, v}. Then w is an upper


S
bound of A B, because if x ∈ A, thenx ≤ u ≤ w, and if x ∈ B,
S
thenx ≤ u ≤ w. If z is any upper bound A B, then z is an upper
bound of A and of B, so that u ≤ z and v ≤ z. Hence w ≤ z. Therefore,
S
w = sup (A B).

12. Consider two cases u ≥ s∗ and u < s∗


5.1 Suprema and Infima 93

13. Hint: Use mathematical induction and Exercise 12.


Chapter 6
Applications of the Supremum
Property

We shall now illustrate how to work with suprema and infima. The
following examples show how the definitions are used to prove results.
We shall also give some very important applications of these concepts
to derive fundamental properties of the real number system that will be
used often.

6.1 Applications of the Supremum Prop-


erty

Example 1 [Suprema and infima of sets be compatible with the Algebraic


Properties of R] Let S be a nonempty subset of R that is bounded above

94
6.1 Applications of the Supremum Property 95

and let a ∈ R. Define the set

a + S = {a + s : s ∈ S}.

Show that
sup(a + S) = a + sup S.

and
inf(a + S) = a + inf S.

Solution

Let u = sup S.

Claim: sup(a + S) = a + u.

As u = sup S, we have s ≤ u for any s ∈ S. Hence we have

a + s ≤ a + u for any s ∈ S.

Therefore, a + u is an upper bound for the set a + S; hence, we have

sup(a + S) ≤ a + u. (6.1)

If v is any upper bound of the set a + S, then a + s ≤ v for all s ∈ S.


Then s ≤ v − a for all s ∈ S, which implies that v − a is an upper bound
of S and hence sup S ≤ v − a, i.e., u ≤ v − a, so that a + u ≤ v. Since v is
any upper bound of a + S, in particular, we can replace v by sup(a + S)
to get
a + u ≤ sup(a + S) (6.2)

Combining the inequalities (6.1) and (6.2), we conclude that

sup(a + S) = a + u
96 Chapter 6. Applications of the Supremum Property

i.e., sup(a + S) = a + sup S .

The proof of inf(a + S) = a + inf S is left as an exercise.

If the suprema or infima of two sets are involved, it is often necessary


to establish results in two stages, working with one set at a time. An
example follows:

Example 2 Suppose that A and B are nonempty subsets of R that


satisfy the property:

a ≤ b for all a ∈ A and all b ∈ B.

Prove that sup A ≤ inf B.

Solution

Fix b ∈ B. Then by assumption, we have

a ≤ b for all a ∈ A.

This means that b is an upper bound of A, so that

sup A ≤ b.

As b ∈ B is arbitrary, the above shows

sup A ≤ b for all b ∈ B.

Hence we see that the number sup A is a lower bound for the set B.
Therefore, we conclude that

sup A ≤ inf B.

Example 3 Let A and B be bounded nonempty subsets of R, and let


A + B = {a + b : a ∈ A, b ∈ B}. Prove that sup(A + B) = sup A + sup B
6.1 Applications of the Supremum Property 97

Solution

Let α = sup A and β = sup B. Then

a ≤ α for a ∈ A and b ≤ β for b ∈ B.

Thus
a+b≤α+β for a ∈ A and b ∈ B.

Hence α + β is an upper bound of A + B . Hence α + β ≤ sup(A + B).

To show that α + β = sup(A + B), it remains to show that v < α + β


is not an upper bound of A + B. Now v < α + β implies v − β < α
implies v − β is not an upper bound of A, so there exists a ∈ A such
that v − β < a. Then v − a < β, and hence v − a is not an upper bound
B, so there exists b ∈ B such that v − a < b. This implies v < a + b.
showing that v is not an upper bound of A + B. Hence

sup A + sup B = α + β = sup(A + B).

The Boundedness of Functions

The idea of upper bound and lower bound is applied to functions by


considering the range of a function. Given a function f : D → R, we
say that f is bounded above if the set

f (D) = {f (x) : x ∈ D}

is bounded above in R; that is, there exists B ∈ R such that

f (x) ≤ B for all x ∈ D.

Similarly, the function f is bounded below if the set f (D) is bounded


98 Chapter 6. Applications of the Supremum Property

below. We say that f is bounded if it is bounded above and below; this


is equivalent to saying that there exists M ∈ R such that

|f (x)| ≤ M for all x ∈ D.

The following example illustrates how to work with suprema and


infima of functions.

Example 4 Suppose f and g are real-valued functions with common


domain D ⊆ R and that f and g are bounded, that is, their ranges
f (D) = {f (x) : x ∈ D} and g(D) = {g(x) : x ∈ D} are bounded sets in
R.

If f (x) ≤ g(x) for all x ∈ D,

then show that


sup f (D) ≤ sup g(D) (6.3)

Notation: Another notation for the inequality (6.3) is

sup f (x) ≤ sup g(x).


x∈D x∈D

Solution to Example 4

To prove this we note that the number sup g(D) is an upper bound
for the set f (D) because for anyx ∈ D, we have

f (x) ≤ g(x) ≤ sup g(D).

As sup f (D) is less than or equal to any upperbound of f (D), we


conclude that
sup f (D) ≤ sup g(D).

Attention! It should be noted that the hypothesis f (x) ≤ g(x) for all
6.1 Applications of the Supremum Property 99

x ∈ D in the previous example does not imply any relation between


sup f (D) andinf g(D). For example, if f (x) = x2 and g(x) = x with
D = {x ∈ R : 0 < x < 1}, then f (x) ≤ g(x) for all x ∈ D but
sup f (D) = 1 andinf g(D) = 0. However, sup g(D) = 1, so that the
conclusion in the previous example holds.

Example 5 Suppose f and g are real-valued functions with common


domain D ⊆ R and that f and g are bounded, that is, their ranges
f (D) = {f (x) : x ∈ D} and g(D) = {g(x) : x ∈ D} are bounded sets in
R.

If f (x) ≤ g(y) for all x, y ∈ D,

then show that


sup f (D) ≤ inf g(D). (6.4)

Notation: Another notation for the inequality (6.4) is

sup f (x) ≤ inf g(y).


x∈D y∈D

Solution to Example 5

The proof is in two stages. First, for a particular y ∈ D, we see that


g(y) is an upper bound for the setf (D), since

f (x) ≤ g(y) for all x ∈ D.

Hence
sup f (D) ≤ g(y).

Since the choice of y ∈ D is arbitrary, we have

sup f (D) ≤ g(y) for all y ∈ D.

Hence the number sup f (D) is a lower bound for the set g(D).
100 Chapter 6. Applications of the Supremum Property

We conclude that
sup f (D) ≤ inf g(D).

Example 6 Let X be a nonempty set and let f : X → R have bounded


range in R. If a ∈ R, show that

sup{a + f (x) : x ∈ X} = a + sup{f (x) : x ∈ X}.

Solution

We note that f (X) = {f (x) : x ∈ X}. Let u = sup f (X). Then


f (x) ≤ u for all x ∈ X, so that

a + f (x) ≤ a + u for all x ∈ X,

and hence
sup{a + f (x) : x ∈ X} ≤ a + u.

If w < a + u, then w − a < u, so w − a is an upper bound of the set f (X),


hence there exists xw ∈ X with w−a < f (xw ), and hence w < a+f (xw ),
and thus w is not an upper bound for {a + f (x) : x ∈ X}. Hence

a + u = sup{a + f (x) : x ∈ X}.

The Archimedean Property

The Archimedean property, named after the ancient Greek mathemati-


cian Archimedes (287 BC – 212 BC) of Syracuse, is a property of having
no infinitely large or infinitely small elements.

Theorem 1 (Archimedean Property)

If x ∈ R, then there exists nx ∈ N such that x < nx .

Proof Letx ∈ R. Suppose on the contrary that there doesn’t exist


6.1 Applications of the Supremum Property 101

nx ∈ N such that x < nx . Then

n ≤ x for all n ∈ N,

and hence x is an upper bound of N. Therefore, by the Supremum


Property, the nonempty set N has a supremum u ∈ R. Since u − 1 < u,
it follows that u − 1 is not an upper bound for N and hence there exists
m ∈ N such thatu − 1 < m. But then u < m + 1, and since m + 1 ∈ N,
this contradicts the assumption that u is an upper bound of N. Hence
there exists nx ∈ N such that x < nx .

Remark: The Archimedean Property ensures that corresponding to the


positive real number z, the subset Ez = {m ∈ N : z < m} of N is not
empty.

Result: The subset N of natural numbers is not bounded above in R.

Proof This follows from Theorem 1. Suppose on the contrary that N is


bounded above in R. Then it has an upper bound, say, x in R. Now, by
Archimedean Property, there exists nx ∈ N such that x < nx . This is a
contradiction to the fact that x is a upper bound of N. Hence N is not
bounded above inR.

The Archimedean Property can be stated in a variety of ways. We


present some of these variations.

Corollary 1 [Archimedean Property ] Let y and z be positive real


numbers. Then:

(a) There exists n ∈ N such that z < ny.

(b) There exists nz ∈ N such that nz − 1 ≤ z < nz .

Proof
z
(a) Since y and z are positive real numbers. y > 0, by the Archimedean
102 Chapter 6. Applications of the Supremum Property

z z
property (Theorem 1, withx = y ), there exists n ∈ N such that y <n
and hence z < ny.

(b) The Archimedean Property ensures that corresponding to the posi-


tive real number z, the subset Ez = {m ∈ N : z < m} of N is not empty.
By the well-ordering property1 of N, the nonempty subset Ez of N has
a least element, which we denote by nz . Then nz − 1 does not belong to
this subset; hence we have nz − 1 ≤ z < nz .
1
Well-Ordering Property of N: Every nonempty subset of N has a
least element.

Example 7 Given any x ∈ R show that there exists a unique n ∈ Z


such that n − 1 ≤ x < n.

Solution

Letx ∈ R. Suppose there are elements m and n in Z such that


m − 1 ≤ x < m and n − 1 ≤ x < n. We have to show that m = n.
Suppose to the contrary that m 6= n . Two cases arise:

Case 1) m < n . Then x < m ≤ n−1 ≤ x, so that x < x, a contradiction.

Case 2) n < m . Then x < n ≤ m−1 ≤ x, so that x < x, a contradiction.

Hence m = n. That is, there exists a unique n ∈ Z such that n − 1 ≤


x < n.
1
Corollary 2 [Archimedean Property] If S = n : n ∈ N , then
inf S = 0.

Proof 1 ∈ S and henceS 6= Φ. Also S is bounded below by 0. Hence,


being a non-empty bounded below subset of R, by the Infimum Property
6.1 Applications of the Supremum Property 103

S has an infimum and we let

w = inf S.

Since 0 is a lower bound of S, we have w ≥ 0. For any ε > 0, we have


1
ε > 0, and hence the Archimedean Property implies that there exists
n ∈ N such that
1
< n,
ε
1
which implies n < ε. Therefore we have

1
0≤w≤ < ε.
n

i.e., 0 ≤ w < ε. But since ε > 0 is arbitrary, it follows, from Theorem 5


of chapter 4, thatw = 0.

Corollary 3 [Archimedean Property ] If y > 0, there exists ny ∈ N


1
such that 0 < ny < y.

Proof Setting z = 1 in Part (a) of Corollary 1 gives (with n = ny

1 < ny y

1
which implies ny < y.
1 1
Already, 0 < ny . Hence 0 < ny < y.

Aliter Proof Since  


1
inf : n ∈ N = 0,
n
1
the assumption y > 0 implies y is not a lower bound for the set n :n∈N .
104 Chapter 6. Applications of the Supremum Property

Thus there exists ny ∈ N such that

1
0< < y.
ny

1
Example 8 Show that sup{1 − n : n ∈ N} = 1.

Solution

Since
1
1− n < 1 for all n ∈ N,

1 is an upper bound. To show that 1 is the supremum, we have to show


1
that for each ε > 0, 1 − ε is not an upper bound of {1 − n : n ∈ N};
1
i.e., to show that there exists m ∈ N such that 1 − ε < 1 − m, which
1
is equivalent to m < ε. By Corollary 3 to the Archimedean Property,
1
corresponding to ε > 0 it is possible to find m ∈ N such that 0 < m < ε.
This completes the work.

Example 9 Let S ⊆ R be nonempty. Prove that if a number u in R


has the properties: (i) for every n ∈ N the number u − n1 is not an upper
1
bound of S, and (ii) for every number n ∈ N the number u + n is an
upper bound of S, then u = sup S.

Solution
1
Fix s ∈ S. Since for every number n ∈ N the number u + n is an upper
bound of S, we have

1
s≤u+ for n ∈ N.
n

Hence s is a lower bound of the set u + n1 : n ∈ N , and thus



6.1 Applications of the Supremum Property 105

   
1 1
s ≤ inf u + : n ∈ N = u + inf : n ∈ N = u + 0 = u..
n n

Since the choice of s ∈ S is arbitrary, the above implies

s ≤ u for s ∈ S.

Thus u is an upper bound of the set S. It remains to show that if v < u,


then v is not an upper bound of S. Now v < u implies u − v > 0 implies
(by Corollary 3 to the Archimedean Property) there exists n ∈ N such
1
that n < u − v. This implies v < u − n1 . Hence v is not an upper bound
1
of S, since u − n is not an upper bound ofS. Hence u = sup S.

The Existence of 2

The importance of the Supremum Property lies in the fact that it guar-
antees the existence of real numbers under certain hypotheses. At the
moment, we shall illustrate this use by proving the existence of a positive
real number x such that x2 = 2; that is, the positive square root of 2. It
was shown earlier (Theorem 6 of Chapter 2 in Page 2.2) that such an x
cannot be a rational number; thus, we will be deriving the existence of
at least one irrational number.

Theorem 2 There exists a positive real number x such that x2 = 2.

Proof Let
S = {s ∈ R : 0 ≤ s and s2 < 2}.

Since 0 ≤ 1 and 12 < 2, we have 1 ∈ S; hence the set S is not empty.


Also, S is bounded above by 2, because if t > 2, then t2 > 4 so that
t ∈
/ S. Therefore the Supremum Property implies that the set S has
106 Chapter 6. Applications of the Supremum Property

a supremum in R, and we let x = sup S. Note that x > 1. [Reason:


x = sup S = 1 implies s ≤ 1 for all s ∈ S. But this is not true because of
the following reason: 0 ≤ 1.1 and (1.1)2 = 1.21 < 2, so that 1.1 ∈ S and
1.1 > 1.]

We shall prove that x2 = 2 by ruling out the other two possibilities:


x2 < 2 and x2 > 2.

First assume that x2 < 2. We shall show that this assumption con-
tradicts the fact that x = sup S by finding an n ∈ N such that x+ n1 ∈ S,
thus implying that x is not an upper bound for S. To see how to choose
1 1
n, note that for n ∈ N, n2 ≤ n so that
 2
1 2x 1 2x 1 1
x+ = x2 + + 2 ≤ x2 + + = x2 + (2x + 1).
n n n n n n

Hence if we can choose n so that

1
(2x + 1) < 2 − x2 ,
n

1 2

then we get x + n < x2 + (2 − x2 ) = 2. By assumption x2 < 2, so we
1 2−x2
have 2 − x2 > 0; this together with 2x+1 > 0 implies 2x+1 > 0. Hence
Corollary 3 to the Archimedean Property can be used to obtain n ∈ N
such that
1 2 − x2
< .
n 2x + 1
1
These steps can be reversed to show that for the choice of n with n <
2
2−x 1
2x+1 , we have x + n ∈ S, which contradicts the fact that x is an upper
bound of S. Therefore we cannot have x2 < 2.

Now assume that x2 > 2. We shall show that it is then possible to


1
find m ∈ N such that x − m is also an upper bound of S, contradicting
6.1 Applications of the Supremum Property 107

the fact that x = sup S. To do this, note that


 2
1 2x 1 2x
x− = x2 − + 2 > x2 − .
m m m m

Hence if we can choose m so that

2x
< x2 − 2,
m

then  2
1
x− > x2 − (x2 − 2) = 2.
m

1
Now by assumption x2 > 2, hence we have x2 − 2 > 0, so (with 2x > 0)
we have
x2 − 2
> 0.
2x
Hence, by Corollary 3 to the Archimedean Property, there exists m ∈ N
such that
1 x2 − 2
<
m 2x
These steps can be reversed to show that for this choice of m we have
 2
1
x− > 2.
m

Now if s ∈ S, then
 2
1
s2 < 2 < x− ,
m
1 2

so s2 < x − m , and hence using the result “For a ≥ 0, b ≥ 0, a <
√ √
b ⇔ a < b”, we have
1
s<x− .
m
108 Chapter 6. Applications of the Supremum Property

1
This implies that x − m is an upper bound for S, which contradicts the
fact that x = sup S. Therefore we cannot have x2 > 2.

Since the possibilities x2 < 2 and x2 > 2 have been excluded, by


Trichotomy property, we must have x2 = 2. This completes the proof.

Existence of Positive n th Root of a Positive Real Number

By slightly modifying the argument in Theorem 2, it can be shown that


if a > 0, then there is a unique b > 0 such that b2 = a. We call b the
√ 1
positive square root of a and denote it by b = a or b = a 2 . A
slightly more complicated argument involving the binomial theorem can
be formulated to establish the existence of a unique positive nth root
√ 1
of a, denoted by n a or a n , for each n ∈ N.

Q Doesn’t Possess Completeness Property

Result: The ordered field Q of rational numbers does not possess the
Completeness Property.

Proof If in the proof of Theorem 2 we replace the set S by the set of


rational numbers T = {r ∈ Q : 0 ≤ r, r2 < 2}, the argument then gives
the conclusion that y = sup T satisfies y 2 = 2. As there is no rational
number such that y 2 = 2, y cannot be a rational number, and hence it
follows that the set T that consists of rational numbers does not have a
supremum belonging to the set Q. Thus the ordered field Q of rational
numbers does not possess the Completeness Property.

Density of Rational Numbers in R

We have just seen that there exists at least one irrational real number,

namely 2. Actually there are more irrational numbers than rational
numbers in the sense that the set of rational numbers is countable, while
the set of irrational numbers is uncountable. We next show that in spite
6.1 Applications of the Supremum Property 109

of this apparent disparity, the set of rational numbers is dense in R in


the sense that a rational number (in fact, infinitely many of them) can
be found between any two distinct real numbers.

Theorem 3 (The Density Theorem) If x and y are real numbers


with x < y, then there exists a rational number rsuch that x < r < y.

Proof It is no loss of generality to assume that x > 0. Then 0 < x < y


and this implies y−x > 0. Hence by Corollary 3 to Archimedean Property
there exists n ∈ N such that

1
< y − x.
n

i.e.,
1 < n(y − x).

Therefore, we have
nx + 1 < ny.

Applying Part (b) of Corollary 1 of the Archimedean Property to nx > 0,


we obtain m ∈ N such that

m − 1 ≤ nx < m.

This m also satisfies


m < ny,

since m ≤ nx + 1 < ny.

Thus we have
nx < m < ny
m
implies x < n < y.
110 Chapter 6. Applications of the Supremum Property

m
Hence r = n is a rational number satisfying

x < r < y.

Corollary If x and y are real numbers withx < y, then there exists an
irrational number z such that x < z < y.

Proof Applying the Density Theorem (Theorem 3 above) to the real


numbers x
√ and √y , we obtain a rational number1 r 6= 0 such that
2 2

x y
√ <r< √ .
2 2

i.e.,

x < r 2 < y.


Then z = r 2 is irrational2 and satisfies x < z < y.
1
If r = 0, then again applying Theorem 3 it is possible to find a rational
number between 0 and √y . Now choose this new rational number as r,
2
so that r 6= 0.
2
√ z
If z = r 2 is rational, then r ,√being the ratio of two rational num-
z r 2

bers, is rational and hence r = r = 2 must be rational, which is a

contradiction to the fact that 2 is irrational.

Example 10 If u > 0 is any real number and x < y, show that there
exists a rational number r such that x < ru < y.

Solution

If u > 0 is any real number and x < y. Since u > 0, we have


x<y⇒ x
u < uy . Hence, by the Density Theorem, there exists a rational
number rsuch that x
u < r < uy . i.e., x < ru < y.
6.1 Applications of the Supremum Property 111

Example 11 If u > 0 is any real number, then show that the set
{ru : r ∈ Q} is dense in R.

Solution follows immediately from the above Example.

Exercises

1. Show that sup{2 − n1 : n ∈ N} = 2.

2. If S = n1 − m
1

: n, m ∈ N , find inf S and sup S.

3. Let S ⊆ R be nonempty. Prove that if a number w in R has the


properties: (i) for every n ∈ N the number w + n1 is not a lower bound of
1
S, and (ii) for every number n ∈ N the number w − n is a lower bound
of S, then w = inf S.

4. Let S be a nonempty bounded set in R.

(a) Let a > 0, and let aS = {as : s ∈ S}. Prove that

inf(aS) = a inf S, sup(aS) = a sup S.

(b) Let b < 0 and let bS = {bs : s ∈ S}. Prove that

inf(bS) = b sup S, sup(bS) = b inf S.

Hence in particular, taking b = −1,

inf(−S) = − sup S,

and
sup(−S) = − inf S.

5. Let X be a nonempty set and let f : X → R have bounded range in


112 Chapter 6. Applications of the Supremum Property

R. If a ∈ R, show that

inf{a + f (x) : x ∈ X} = a + inf{f (x) : x ∈ X}.

6. Let A and B be bounded nonempty subsets of R, and let A + B =


{a + b : a ∈ A, b ∈ B}. Prove that inf(A + B) = inf A + inf B.

7. Let X be a nonempty set, and let f and g be defined on X and have


bounded ranges in R. Show that

sup{f (x) + g(x) : x ∈ X} ≤ sup{f (x) : x ∈ X} + sup{g(x) : x ∈ X}.

and

inf{f (x) : x ∈ X} + inf{g(x) : x ∈ X} ≤ inf{f (x) + g(x) : x ∈ X}.

Give examples to show that each of these inequalities can be either


equalities or strict inequalities.

8. Let X = Y = {x ∈ R : 0 < x < 1}. Define h : X × Y → R by


h(x, y) = 2x + y.

(a) For each x ∈ X, find f (x) = sup{h(x, y) : y ∈ Y }; then find


inf{f (x) : x ∈ X}.

(b) For eachy ∈ Y , find g(y) := inf{h(x, y) : x ∈ X}; then find


sup{g(y) : y ∈ Y }. Compare with the result found in part (a).

9. Perform the computations in (a) and (b) of the preceding exercise for
the function h : X × Y → R defined by
(
0 if x < y
h(x, y) =
1 if x < y
6.1 Applications of the Supremum Property 113

10. Let X and Y be nonempty sets and let h : X × Y → R have bounded


range in R. Let f : X → R and g : Y → R be defined by

f (x) = sup{h(x, y) : y ∈ Y }, g(y) = inf{h(x, y) : x ∈ X}

Prove that

sup{g(y) : y ∈ Y } ≤ inf{f (x) : x ∈ X}.

Notation We sometimes express this by writing

sup inf h(x, y) ≤ inf sup h(x, y).


y x x y

11. Let X and Y be nonempty sets and let h : X × Y → R have bounded


range in R. Let F : X → R and G : Y → R be defined by

F (x) = sup{h(x, y) : y ∈ Y }, G(y) := sup{h(x, y) : x ∈ X}.

Establish the Principle of the Iterated Suprema:

sup{h(x, y) : x ∈ X, y ∈ Y } = sup{F (x) : x ∈ X} = sup{G(y) : y ∈ Y }.

Notation We sometimes express this in symbols by

suph(x, y) = supsuph(x, y) = supsuph(x, y).


x, y x y y x

1
12. If y > 0 show that there exists n ∈ N such that 2n < y.

13. Show that there exists a positive real number y such that y 2 = 3.
114 Chapter 6. Applications of the Supremum Property

14. Show that if a > 0, then there exists a positive real number z such
that z 2 = a.

15. Show that there exists a positive real number u such that u3 = 2.

Answers

2. inf S = −1 and sup S = 1.

4. (a) Let u = sup S and a > 0. Then x ≤ u for all x ∈ S, hence ax ≤ au


for all x ∈ S, hence it follows that au is an upper bound of aS. If v is
another upper bound of aS, then ax ≤ v for all x ∈ S, hence x ≤ v/a
for all x ∈ S, showing that v/a is an upper bound for S so that u ≤ v/a,
from which we conclude that au ≤ v. Therefore au = sup (aS).

(b) Let u = sup S andb < 0. Then x ≤ u for all x ∈ S, hence


bx ≥ bu for all x ∈ S, [Attention! Note that the inequality get affected
by multiplication by a negative number] hence it follows that bu is a
lower bound of bS. If v is another lower bound of bS, then bx ≥ v for all
x ∈ S, hence x ≤ v/b for all x ∈ S, showing that v/b is an upper bound
for S so that u ≤ v/b, from which we conclude that bu ≥ v. Therefore
bu = inf (bS).

7. If u = sup f (X) and v = sup g(X), then f (x) ≤ u and g(x) ≤ v for
all x ∈ X hence f (x) + g(x) ≤ u + v for all x ∈ X.

8. (a) f (x) = 2x + sup{y} = 2x + 1.

9. (a) f (x) = 1 for x ∈ X.

(b) g(y) = 0 for y ∈ Y.

11. Let S = {h(x, y) : x ∈ X, y ∈ Y }. We have h(x, y) ≤ F (x) for all


x ∈ X, y ∈ Y so that sup S ≤ sup{F (x) : x ∈ X}.

If w < sup{F (x) : x ∈ X}, then there exists x0 ∈ X with w < F (x0 )
6.1 Applications of the Supremum Property 115

= sup{h(x0 , y) : y ∈ Y }, hence there exists y0 ∈ Y with w < h(x0 , y0 ).


Thus w is not an upper bound of S, and so w < sup S. Since this is true
for any w such that w < sup{F (x) : x ∈ X}, we have sup{F (x) : x ∈
X} ≤ sup S

12. Note that n < 2n (hence 1/2n < 1/n) for any n ∈ N

13. Let S = {s ∈ R : 0 ≤ s, s2 < 3}. Show that S3 is nonempty and


bounded by 3 and let y = sup S. If y 2 < 3 and 1/n < (3 − y 2 )/2y + 1)
show that y + 1/n ∈ S. If y 2 > 3 and 1/m < (y 2 − 3)/2y show that
y − 1 · m ∈ S. Therefore y 2 = 3.
Chapter 7
Intervals

7.1 Intervals

The order relation on R determines a natural collection of subsets known


as “intervals.” The notations and terminology for these special sets are
as follows.

Definition/Remarks If a, b ∈ R and a < b, then the open interval


determined by a and b is the set

(a, b) = {x ∈ R : a < x < b}. (7.1)

The points a and b are called the end points of the open interval (a, b),
but the end points are not included in the open interval. If both end

116
7.1 Intervals 117

points are adjoined to the open interval, we have the closed interval

[a, b] = {x ∈ R : a ≤ x ≤ b}. (7.2)

The sets
[a, b) = {x ∈ R : a ≤ x < b}. (7.3)

and
(a, b] = {x ∈ R : a < x ≤ b}. (7.4)

are half-open (or half-closed) intervals determined by the end points


a and b. Each of the above intervals [ (7.1)-(7.4)] have length defined
by b − a. If a = b, note that the corresponding open interval is the empty
set
(a, a) = Φ, (7.5)

whereas the corresponding closed interval is the singleton set

[a, a] = {a}. (7.6)

If a ∈ R then the sets defined by

(a, ∞) = {x ∈ R : x > a}, (7.7)

(−∞, a) = {x ∈ R : x < a}, (7.8)

are called open rays (or infinite open intervals). Similarly, the sets
defined by
[a, ∞) = {x ∈ R : x ≥ a}, (7.9)

and
(−∞, a] = {x ∈ R : x ≤ a}, (7.10)
118 Chapter 7. Intervals

are called closed rays (or infinite closed intervals). In these cases the
point a is called the end point of these intervals. It is often convenient
to think of the entire set R as an infinite interval. In this case we write

(−∞, ∞) = R, (7.11)

and we do not consider any point to be an end point of (−∞, ∞).

It will be noted that intervals of the forms (7.1)-(7.6) are bounded


intervals. Intervals of the forms (7.7)-(7.11) are unbounded. [More
precisely, intervals (7.1)-(7.4) has infimum = a and supremum = b.
Interval in (7.5), being the empty set, has upper bounds and lower
bounds, but neither infimum nor supremum exists. Interval in (7.6)
has inf = sup = a. Intervals in (7.7) and (7.9) have lower bounds and
inf = a, but have no upper bound. Intervals in (7.8) and (7.10) have no
lower bounds; but upper bounds exist and sup = a. Interval in (7.11)
has neither a lower bound nor an upper bound].

Attention! In denoting the unbounded intervals in (7.7)-(7.11) we


have used the symbols −∞ and ∞. These symbols should be considered
merely for notational conveniences; they are not elements of R.

The unit interval is the closed interval

[0, 1] = {x ∈ R : 0 ≤ x ≤ 1}. (7.12)

It will often be denoted by the standard notationI. (The unit interval


is also called the continuum.)

Characterization of Intervals

An obvious property of intervals is that if two points x, y with x < y


belong to an interval I, then any point lying between them also belongs
7.1 Intervals 119

to I. That is, if x < t < y, then the point t belongs to the same interval
as x and y. In other words, if x and y belong to an interval I, then
the interval [x, y] is contained in I. We now show that a subset of R
possessing this property must be an interval.

Theorem 1 (Characterization Theorem) If S is a subset of R that


contains at least two points and has the property

if x, y ∈ S and x < y then [x, y] ⊆ S, (7.13)

then S is an interval.

Proof There are four cases to consider: (i) S is bounded, (ii) S is


bounded above but not below, (iii) S is bounded below but not above,
and (iv) S is unbounded,

Case (i) : Suppose S is bounded. Then it is bounded above and


below and hence by the Completeness Property of R, as S 6=Φ, both
inf S and sup S exist. Let

a = inf S and b = sup S.

Then S ⊆ [a, b]

and we will show that (a, b) ⊆ S.

Let z ∈ (a, b). Then a < z < b, and hence z is not a lower bound of
S. So there exists x ∈ S with x < z. Also, z is not an upper bound of
S, so there exists y ∈ S with z < y. Therefore z ∈ [x, y]. By property
(7.13), [x, y] ⊆ S. Hence z ∈ [x, y] implies that z ∈ S.

Since z is an arbitrary element of (a, b), we conclude that

(a, b) ⊆ S.
120 Chapter 7. Intervals

1. Now if a ∈ S and b ∈ S, then S = [a, b].

2. If a ∈
/ S and b ∈
/ S, then S = (a, b).

3. If a ∈
/ S and b ∈ S, then S = (a, b].

4. If a ∈ S and b ∈
/ S, then S = [a, b).

Hence S is an interval.

Case (ii): Let b = sup S. Then S ⊆ (−∞, b] and we will show that
(−∞, b) ⊆ S. For, if z ∈ (−∞, b], then z < b, so there exist x, y ∈ S
such that z ∈ [x, y] ⊆ S. Therefore (−∞, b) ⊆ S. If b ∈ S, then S =
(∞, b], and if b ∈
/ S, then S = (−∞, b). Hence S is an interval.

Case (iii) is left as an exercise.

Case (iv) Suppose S is unbounded. We claim that S = (−∞, ∞).


Already S ⊆ (−∞, ∞). To prove that (−∞, ∞) ⊆ S, we let z ∈
(−∞, ∞). Then there are x, y ∈ S such that z ∈ [x, y]. By prop-
erty (7.13), [x, y] ⊆ S. Hence z ∈ S. Thus (−∞, ∞) ⊆ S, and hence
S = (−∞, ∞). Hence S is an interval in this case also.

This completes the proof.

Nested Intervals

Definition A sequence of intervals In , n ∈ N, is nested (Fig. 7.1) if


the following chain of inclusions holds:

I1 ⊇ I2 ⊇ I3 ⊇ · · · ⊇ In ⊇ In+1 ⊇ · · · .
7.1 Intervals 121

Figure 7.1: Nested intervals I1 ⊇ I2 ⊇ I3 ⊇ · · · ⊇ In ⊇ In+1 ⊇ · · · .

1
 
Example 1 If In = 0, n , n ∈ N, then In ⊇ In+1 for each n ∈ N so
that the sequence of closed intervals is nested. In this case, the element 0
belongs to all In . We claim that 0 is the only such element, for otherwise
suppose there exists y > 0 such that y ∈ In for all n ∈ N. Then we must
1
have for all n ∈ N, y < n. This is not possible as it is a contradiction
to the result [Corollary 3 to the Archimedean Property] “If y > 0, there
1
exists ny ∈ N such that 0 < ny < y.”.

We conclude that 0 is the only element that belongs to all In . We


T∞
denote this by writing n=1 In = {0}.

Attention! In general, a nested sequence of intervals need not have a


common point. The following example illustrates this.

Example 2 If Jn = 0, n1 for n ∈ N, then this sequence of open




intervals is nested, but the intervals have no point in common. This


is true because for a given y > 0, (using Corollary to Archimedean
1
Property) there exists my ∈ N such that my < y so that y ∈
/ (0, 1/my ).

i.e., y ∈
/ Jmy .

Example 3 Proceeding as in the above example, the sequence of intervalsKn =


122 Chapter 7. Intervals

(n, ∞), n ∈ N, is nested but has no common point.

Example 4 Proceeding as in the above example, the sequence of inter-


vals Kn = [n, ∞), n ∈ N, is nested but has no common point.

However, it is a very important property of R that every nested


sequence of closed and bounded intervals does have a common point. The
completeness of R plays an essential role in establishing this property.

Theorem 2 (Nested Intervals Property) If In = [an , bn ], n ∈ N,


is a nested sequence of closed and bounded intervals, then there exists a
number ξ ∈ R such that ξ ∈ In for all n ∈ N.

Proof Since the intervals are nested, we have

I1 ⊇ I2 ⊇ I3 ⊇ · · · ⊇ In ⊇ In+1 ⊇ · · · .

In particular, In ⊆ I1 for all n ∈ N, so that

[an , bn ] ⊆ [a1 , b1 ] for n ∈ N

and hence an ∈ [a1 , b1 ] for n ∈ N; so an ≤ b1 for all n ∈ N. Hence the


nonempty set {an : n ∈ N} is bounded above, hence by the Completeness
Property of R supremum of the set exists. We let ξ be the supremum.
i.e., ξ = sup{an : n ∈ N}.

Claim: ξ ∈ In for all n ∈ N. i.e., an ≤ ξ ≤ bn for all n ∈ N.

Clearly

an ≤ ξ for all n ∈ N.

It remains to show that ξ ≤ bn for all n. This is established by showing


that for any particular n, the number bn is an upper bound for the set
{ak : k ∈ N}. We consider two cases.
7.1 Intervals 123

Figure 7.2: If k < n, then In ⊆ Ik

(i) If n ≤ k, then since In ⊇ Ik , we have [ak , bk ] ⊆ [an , bn ], so that


ak ≤ bk ≤ bn .

(ii) If k < n, then since Ik ⊇ In , we have [an , bn ] ⊆ [ak , bk ] , so that


ak ≤ an ≤ bn . (See Fig. 7.2.) Thus, we conclude that

ak ≤ bn for all k,

so that bn is an upper bound of the set {ak : k ∈ N}. Since the choice
of n is arbitrary, this shows that

ak ≤ bn for all k ∈ N, for all n ∈ N.

Hence,

ξ ≤ bn for each n ∈ N.

This together with an ≤ ξ for all n ∈ N implies

an ≤ ξ ≤ bn for all n.

Hence we have ξ ∈ In for all n ∈ N.

Theorem 3 If In = [an , bn ], n ∈ N, is a nested sequence of closed,


bounded intervals such that the lengths bn − an of In satisfy

inf{bn − an : n ∈ N} = 0,
124 Chapter 7. Intervals

then the number ξ contained in In for all n ∈ N is unique.

Proof If η = inf{bn : n ∈ N} and ξ = sup{an : n ∈ N} then using an


argument similar to the proof of Theorem 2 (Nested Intervals Property
Theorem) shows that ak ≤ η for all k, and hence that ξ ≤ η.

[Details: Fix k ∈ N .

Case (i) If n ≤ k, then since In ⊇ Ik , we have that ak ≤ bk ≤ bn .

Case (ii) If k < n, then since Ik ⊇ In , we have ak ≤ an ≤ bn .

Hence, we have ak ≤ bn for n ∈ N . Thus ak ≤ inf{bn : n ∈ N}. i.e.,


ak ≤ η. Since the choice of k ∈ N is arbitrary, we have ak ≤ η for k ∈ N .
Thus sup{ak : k ∈ N} ≤ η . i.e., ξ ≤ η .]

Now x ∈ In for all n ∈ N if and only if an ≤ x ≤ bn for all n ∈ N


if and only if sup{an : n ∈ N} ≤ x ≤ inf{bn : n ∈ N} if and only if
ξ ≤ x ≤ η.

If we have inf{bn − an : n ∈ N} = 0, then for any ε > 0, the number


ε is not a lower bound of the set {bn − an : n ∈ N}, so that there exists
m ∈ N such that bm − am < ε. Also, η ≤ bm and am ≤ ξ, so that η ≤ bm
and −ξ ≤ −am ; hence η − ξ ≤ bm − am . Thus 0 ≤ η − ξ ≤ bm − am < ε.
Since this holds for all ε > 0, it follows from the result “If a ∈ R is such
that 0 ≤ a < ε for every ε > 0, then a = 0” that η − ξ = 0. Therefore,
we conclude that ξ = η is the only point that belongs to In for every
n ∈ N.

The Uncountability of R

Theorem 4 The set R of real numbers is not countable.

Proof We will prove that the unit interval I = [0, 1] is an uncountable


set. This implies that the set R is an uncountable set, for if R were
7.1 Intervals 125

countable, then the subset I would also be countable. (Recall the result:
“Subset of a countable set is countable”).

The proof is by contradiction. If we assume that I is countable,


then we can enumerate the set as I = {x1 , x2 , · · · , xn · · · }. We first
select a closed subinterval I1 of I such that x1 ∈
/ I1 , then select a closed
subinterval I2 of I1 such that x2 ∈
/ I2 , and so on. In this way, we obtain
nonempty closed intervals

I1 ⊇ I2 ⊇ · · · ⊇ In ⊇ · · · .

Since I1 ⊆ I, the above implies In ⊆ I and xn ∈


/ In for all n.

The Nested Intervals Property (Theorem 2) implies that there exists a


point ξ ∈ I such that ξ ∈ In for all n. Therefore

ξ 6= xn for all n ∈ N .

This shows that there exists ξ ∈ I such that ξ is not included in the
enumeration. So the enumeration of I is not a complete listing of the
elements of I, as claimed. Hence, I is an uncountable set. This completes
the proof.

The fact that the set R of real numbers is uncountable can be com-
bined with the fact that the set Q of rational numbers is countable to
conclude that the set R\Q is uncountable. We have the Corollary:

Corollary The set of irrational numbers is uncountable.

Proof Assume to the contrary that R\Q is countable. Since the union
of two countable sets is a countable set, the assumed countability of
R\Q would imply that the union of Q and R\Q would be a countable
set. That is, R is countable. This is a contradiction. Hence we conclude
126 Chapter 7. Intervals

that the set of irrational numbers R\Q is an uncountable set.

The set R of real numbers is uncountable. The set of irrational


numbers is also uncountable.

Cantors Proof on the Uncountability of [0, 1]

We now give a second proof of the result: “the set I of real numbers
x that satisfy 0 ≤ x ≤ 1 is not countable”. The proof is the diagonal
argument of Cantor and uses the concept of decimal representation.

Theorem 6 The unit interval [0, 1]= {x ∈ R : 0 ≤ x ≤ 1} is not


countable.

Proof The proof is by contradiction. Suppose [0, 1] is countable. Then


there is an enumeration x1 , x2 , x3 , . . . of all real numbers in [0, 1]. We
will use the fact that every real number x in [0, 1] has a decimal repre-
sentation in the form x = 0.a1 a2 a3 . . . where each ai represents a digit
from 0 to 9. It must be kept in mind that certain numbers have two deci-
mal representations, one terminating in 0’s and the other in 9’s. Suppose
[0, 1] is countable. Then there is an enumeration x1 , x2 , x3 , . . . of all
real numbers in [0, 1] given by

x1 = 0.a11 a12 a13 ... a1n ...


x2 = 0.a21 a22 a23 ... a2n ...
x3 = 0.a31 a32 a33 ... a3n ...
..
.
xn = 0.an1 an2 an3 ... ann ...
..
.

We now define a new real number y = 0.y1 y2 y3 . . . yn . . . by setting yn as


7.1 Intervals 127

follows. We let y1 = 2 if a11 ≥ 5 and y1 = 7 if a11 ≤ 4; in general we let


(
2 if ann ≥ 5
yn = .
7 if ann ≤ 4

Then we have 0 ≤ y ≤ 1. The number y is not equal to any of the


numbers with two decimal representations, since yn 6= 0, 9. Also we
have yn 6= xn for all n ∈ N because y and xn are different in the nth
decimal place. (Thus the choice of 2 and 7 is not important, just some
choice to assure that the nth decimal places differ.) Therefore, y is not
included in the above enumeration of [0, 1], contradicting the hypothesis.
Therefore, [0, 1] is not a countable set. This completes the proof.

Exercises

1. If I = [a, b] and I 0 = [a0 , b0 ] are closed intervals in R, show that


I ⊆ I 0 if and only if a0 ≤ a and b ≤ b0 .

2. If S ⊆ R is nonempty, show that S is bounded if and only if there is


some closed bounded interval I ⊆ R such that S ⊆ I.

3. If S ⊆ R is a nonempty bounded set, and IS is the interval IS =


[inf S, sup S], show that S ⊆ IS . Moreover, if J is any closed bounded
interval of R such that S ⊆ J, show that IS ⊆ J.

4. Show that if I1 ⊇ I2 ⊇ · · · ⊇ In ⊇ · · · is a nested sequence of closed


intervals in R, and if In = [an , bn ], then a1 ≤ a2 ≤ · · · ≤ an ≤ · · ·
and b1 ≥ b2 ≥ · · · ≥ bn ≥ · · · .
T∞
5. Let In = 0, n1 for n ∈ N. Prove that if x > 0, then x ∈
 
/ n=1 In .
T∞
6. Let In = 0, n1 for n ∈ N. Prove that n=1 In = {0}.
 

T∞
7. Prove that if Jn = 0, n1 for n ∈ N, then n=1 Jn = ∅.

128 Chapter 7. Intervals

T∞
8. Prove that if Kn = (n, +∞) for n ∈ N, then n=1 Kn = ∅.

9. Show that if

a1 a2 an b1 b2 bm
+ + ··· + = + + · · · + m 6= 0.
10 102 10n 10 102 10

where the ak and bk belong to {0, 1, . . . , 9}, then n = m and ak = bk


for k = 1, . . . , n.
3 7
10. Give two binary representations of 8 and 16 as periodic decimals.

11. Give the first four digits in the binary representation of 13 .

12. Give the complete binary representation of 13 .

13. Find the decimal representation of − 27 .


1 2
14. Express 7 and 19 as periodic decimals.

15. Find the rational number represented by the periodic decimals


1.25137 · · · 137 · · · , 35.14653 · · · 653 · · · and 7.31414 · · · 14 · · ·

16. Using the notation in the proofs of Theorems 2 and 3, show that we
T∞ T∞
have η ∈ n=1 In . Also show that [ξ, η] = n=1 In .

17. Show that the intervals obtained from the inequalities

a1 a2 an a1 a2 an 1
+ 2 + ··· + n ≤ x ≤ + 2 + ··· + n + n
2 2 2 2 2 2 2

form a nested sequence.

Answers

2. S has an upper bound b and a lower bound a if any only if S is


contained in the interval [a,b]

7. If x > 0, then there exists n ∈ N with 1/n < x, so that x ∈


/ Jn . If
7.1 Intervals 129

y ≤ 0, then y ∈
/ J1 .

10. 38 = (.011000 · · · )2 = (.010111 · · · )2 ,

7
16 = (.0111000 · · · )2 = (.0110111 · · · )2

11. 13 ≈ (.0101)2
1
12. 3 ≈ (.010101 · · · )2 , the block 01 repeats.

14. 1/7 = 142857 · · · , the block repeats

2/19 = .105263 157894736842 · · · the block repeats.

15.1.25 137 · · · 137 · · · = 31253/24975 · · · 35.14653 · · · 653 · · · = 3511139/99900

17. Let η = inf{bn : n ∈ N}; we claim that an ≤ η for all n. n ∈ N; we


will show that an is a lower bound for the set {bk : k ∈ N}. We consider
two cases (i) If n ≤ k, then since In ⊇ Ik , we have an ≤ ak ≤ bk (ii) If
k < n, then since Ik ⊇ In , we have an ≤ bn ≤ bk . Therefore an ≤ bk for
all k ∈ N, so that an is a lower bound for {bk : k ∈ N} and so an ≤ η.
T
In particular, this shows that η ∈ [an , bn ] for all n, so that η ∈ In .
Chapter 8
Sequences of Real Numbers

In this chapter we shall familiarize the concept of sequence of real num-


bers and convergence of sequences. From the definition of limit of
a sequence we will observe that the convergence (or divergence) of a
sequence X = (xn ) depends only on the ultimate behavior of the terms
of the sequence. Hence for any natural number m, if we drop the first m
terms of the sequence, then the resulting sequence Xm converges if and
only if the original sequence converges, and in this case, the limits are
the same. We will state this formally as a theorem after introducing the
concept of a tail of a sequence.

8.1 Sequences and their Limits

Definition A function f from the set N = {1, 2, . . .} of natural numbers


to a nonempty set S is called a sequence in the set S.

130
8.1 Sequences and their Limits 131

In this chapter we are interested in a particular, but important, case


when S = R. Definition follows:

Definition A sequence of real numbers (or a sequence in R) is a


function on the set N = {1, 2, 3 · · · } of natural numbers whose range
is contained in the set R of real numbers.

Remark Being a function from the set of natural numbers to the set
of real numbers, a sequence in R assigns to each natural number n =
1, 2, . . . a uniquely determined real number. The real numbers so
obtained are called the elements of the sequence, or the values of the
sequence, or the terms of the sequence. It is customary to denote the
element of R assigned to n ∈ N by a symbol such as xn (or an , or zn ).
Thus, if X : N → R is a sequence, we shall ordinarily denote the value
of X at n by xn , rather than by X(n). We will denote this sequence by
the notations
X, (xn ), (xn : n ∈ N).

We use the parentheses to indicate that the ordering induced by


the natural order of N is a matter of importance. Thus, we distin-
guish notationally between the sequenceX = (xn : n ∈ N), whose terms
have an ordering, and the set {xn : n ∈ N} of values of this sequence
which are not ordered. For example, the sequence ((−1)n : n ∈ N) is
(−1, 1, −1, 1, · · · ) and alternates between −1 and 1, whereas {(−1)n :
n ∈ N} is equal to the set {−1, 1}.

In defining sequences it is often convenient to list the terms of a


sequence in order, stopping when the rule of formation seems evident.
Thus we may write
X = (2, 4, 6, 8, . . .)
132 Chapter 8. Sequences of Real Numbers

for the sequence of even natural numbers, or


 
1 1 1 1
Y = , , , , ···
1 2 3 4

for the sequence of reciprocals of the natural numbers, or


 
1 1 1 1
Z= , , , , ···
12 22 32 42

for the sequence of reciprocals of the squares of the natural numbers. A


more satisfactory method is to specify a formula for the general term of
the sequence, such as
   
1 1
X = (2n : n ∈ N), Y = :m∈N , Z= :s∈N .
m s2

In practice, it is often convenient to specify the value x1 and a formula for


obtaining xn+1 (n ≥ 1) when xn is known. Still more generally we may
specify x1 and a formula for obtaining xn+1 (n ≥ 1) from x1 , x2 , . . . , xn .
We refer to either of these methods as an inductive or recursive def-
inition of the sequence. In this way, the sequence X of even natural
numbers can be defined by

x1 = 2, xn+1 = xn + 2 (n ≥ 1);

or by the definition

x1 = 2, xn+1 = x1 + xn (n ≥ 1).

Remark Sequences that are given by an inductive process often arise


in computer science. In particular, sequences defined by an inductive
process of the form x1 = given, xn+1 = f (xn ) for n ∈ N are especially
8.1 Sequences and their Limits 133

amenable to study using computers. Sequences defined by the process:


y1 = given, yn+1 = gn (y1 , y2 , . . . , yn ) for n ∈ N, can also be treated.

Example 1 If b ∈ R, the sequence B = (b, b, b, . . .), all of whose


terms equalb, is called the constant sequence b. Thus the constant
sequence 1 is the sequence (1, 1, 1, . . .), all of whose terms equal 1, and
the constant sequence 0 is the sequence (0, 0, 0, . . .).

Example 2 The sequence of squares of the natural numbers is the se-


quence S = (12 , 22 , 32 , . . .) = (n2 : n ∈ N), which, of course, is the
same as the sequence (1, 4, 9, . . . , n2 , . . .).

Example 3 If a ∈ R, then the sequence A = (an : n ∈ N) is the sequence


1
A = (a, a2 , a3 , . . . , an , . . .). In particular, if a = 2, then we obtain
the sequence
   
1 1 1 1 1
:n∈N = , , , ··· , n, ··· .
2n 2 4 8 2

Example 4 As noted earlier the sequence of (2n : n ∈ N) of even


natural numbers can be defined inductively by

x1 = 2, xn+1 = xn + 2 (n = 1, 2, . . .).

or by the definition

y1 = 2, yn+1 = y1 + yn (n = 1, 2, . . .)

Example 5 The Fibonacci sequence F = (fn : n ∈ N) is given by


the inductive definition

f1 = 1, f2 = 1, fn+1 = fn−1 + fn (n ≥ 2).


134 Chapter 8. Sequences of Real Numbers

The first ten terms of the Fibonacci sequence are seen to be F =


(1, 1, 2, 3, 5, 8, 13, 21, 34, 55, . . .).

We now introduce some important ways of constructing new sequences


from given ones.

Definition If X = (xn ) and Y = (yn ) are sequences of real numbers,


then we define their sum to be the sequence

X + Y = (xn + yn : n ∈ N),

their difference to be the sequence

X − Y = (xn − yn : n ∈ N),

and their product to be the sequence

X · Y = (xn yn : n ∈ N).

If c ∈ R we define the multiple of X by c to be the sequence

cX = (cxn : n ∈ N).

Finally, ifZ = (zn ) is a sequence of real numbers with zn 6= 0 for all


n ∈ N, then we define the quotient of X and Z to be the sequence
 
X xn
= :n∈N .
Z zn

For example, if X and Y are the sequences


 
1 1 1 1
X = (2, 4, 6, . . . , 2n, . . .), Y = , , , ··· , , ··· ,
1 2 3 n
8.1 Sequences and their Limits 135

then we have

2n2 + 1
 
3 9 19
X +Y = , , , ··· , , ··· ,
1 2 3 n

2n2 − 1
 
1 7 17
X −Y = , , , ··· , , ··· ,
1 2 3 n
X · Y = (2, 2, 2, . . . , 2, . . .),

4X = (8, 16, 24, . . . , 8n, . . .)


X
= (2, 8, 18, . . . , 2n2 , . . .).
Y
We note that if Z denotes the sequence

Z = (0, 2, 0, . . . , 1 + (−1)n , . . .),

X
then we can define X + Z, X − Z and X · Z; but Z is not defined since
some of the terms of Z are equal to 0.

The Limit of a Sequence

There are a number of different limit concepts in real analysis. The


notion of limit of a sequence is the most basic.

Definition Let X = (xn ) be a sequence of real numbers. A real number


x is said to be a limit of (xn ) if for every ε > 0 there exists a natural
number K(ε) such that for alln ≥ K(ε), the terms xn satisfy |xn − x| <
ε.

If x is a limit of the sequence, we also say that X = (xn ) converges to


x (or has a limitx). If a sequence has a limit, we say that the sequence
is convergent; if it has no limit, we say that the sequence is divergent.

Remark The notation K(ε) is used to make it explicit that the choice
136 Chapter 8. Sequences of Real Numbers

of K depends on the value of ε > 0; however, it is often convenient to


write K instead of K(ε). In many cases, a “small” value of ε will usually
require a “large” value of K in order to guarantee that the distance
|xn − x| between xn and x is less than ε for all n ≥ K = K(ε).

Notation When a sequence X = (xn ) has a limit x in R, we will use


the notation

lim X = x or lim(xn ) = x or lim xn = x.


n→∞

Also we will sometimes use the symbolismxn → x, which indicates the


intuitive idea that the values xn “approach” the number x as n → ∞.

Example 6 Show thatlim n1 = 0.




Solution
1
Here xn = n and x = 0. We have to show that for every ε > 0 there
is a natural number K(ε) such that if n ≥ K(ε), then

1
− 0 = 1 < ε.

n (8.1)
n

To show this, we note that, by the Archimedean Property, corresponding


to any ε > 0 there is a natural number exceeding 1ε . Now if K(ε) is a
natural number with K(ε) > 1ε , then for any n ∈ N such that n ≥ K(ε),
1
we will have n > ε so that n1 < ε. Thus (8.1) is proved, and hence
lim n1 = 0


1

Example 7 lim n2 = 0.

Solution
1
Here xn = n2 andx = 0. We have to show that for every ε > 0 there
8.1 Sequences and their Limits 137

is a natural number K(ε) such that if n ≥ K(ε), then



1 1
n2 − 0 = n2 < ε. (8.2)

By the Archimedean Property corresponding to any ε > 0 there is a


natural number K(ε) such thatK(ε) > √1 . Then if n ≥ K(ε) then
ε
n> √1 2
so that n > 1
and hence it follows thatε > 1
ε ε n2 . This proves
1

(8.2) and hencelim n2 = 0.

Example 8 The sequence (0, 2, 0, 2, . . . , 0, 2, . . .) does not converge


to 0. Can it converge to any other real number?

Solution
(
0, when n is odd
Here xn = .
2, when n is even
If we choose ε = 1, then, for any natural number K, one can always
select an even number n > K, for which the corresponding value xn = 2
and for which |xn − 0| = |2 − 0| = 2 > 1. Thus, the number 0 is not
the limit of the given sequence.

The sequence doesn’t converge to 2, because if we choose ε = 1, then,


for any value of K, one can always select an odd number n > K, for
which the corresponding value xn = 0 and for which |xn −2| = |0−2| =
2 > 1. Thus, the number 2 is not the limit of the given sequence.

Also, because the terms of the sequence are alternately 0 and 2, they
never accumulate near any other value. Therefore, the sequence diverges.

Example 9 Show that the sequence (−1)n+1 n−1



n is divergent.

Solution
138 Chapter 8. Sequences of Real Numbers

Figure 8.1:

Take a positive ε smaller than 1 so that the bands shown in Fig. 8.1
about the lines y = 1 and y = −1 do not overlap. Any ε < 1 will do.
Convergence to 1 would require every point of the graph beyond a certain
index K to lie inside the upper band, but this will never happen. As soon
as a point (n, an ) lies in the upper band, the next point (n + 1, an+1 )
will lie in the lower band. Hence the sequence cannot converge to 1.
Likewise, it cannot converge to −1. On the other hand, because the
terms of the sequence get alternately closer to 1 and −1, they never
accumulate near any other value. Therefore, the sequence diverges.
 
Example 10 lim 3n+2n+1 = 3.

Solution

Given ε > 0, we have to obtain the inequality



3n + 2
n + 1 − 3 < ε (8.3)

when n is sufficiently large. We first simplify the expression on the left.



3n + 2 3n + 2 − 3n − 3 −1 1 1
n + 1 − 3 = = n + 1 = n + 1 < n.

n+1
8.1 Sequences and their Limits 139

1
Now if the inequality n < ε is satisfied i.e., if n > 1ε , then the inequality
(8.3) holds. If K(ε) is a natural number such that K(ε) > 1ε , then for
any n ∈ N such that n ≥ K(ε), (8.3) holds. Hence the limit of the
sequence is 3.

Example 11 If 0 < b < 1, then lim(bn ) = 0.

Solution

We will use elementary properties of the natural logarithm function.


If ε > 0 is given, we see that

ln ε
bn < ε ⇔ n ln b < ln ε ⇔ n >
ln b

(The last inequality is reversed because ln b < 0 for 0 < b < 1. Then
1 1
ln b < 0 and hence n · ln b < ln ε ⇔ n > ln b · ln ε. Thus if we choose K
to be a number such that K > ln ε/ ln b, then we will have 0 < bn < ε
for all n ≥ K. Thus we have lim(bn ) = 0.

For example, if b = 0.8, and if ε = 0.01 is given, then we would


need K > ln .01/ ln .8 ≈ 20.6377. Thus K = 21 would be an appropriate
choice for ε = .01.

Theorem 1 (Uniqueness of Limits) A sequence of real numbers can


have at most one limit.

Proof Suppose that x0 and x00 are both limits of (xn ). For each ε > 0
there exist K 0 such that

|xn − x0 | < ε/2 for all n ≥ K 0 ,

and there exists K 00 such that

|xn − x00 | < ε/2 for alln ≥ K 00 .


140 Chapter 8. Sequences of Real Numbers

We let K be the larger of K 0 and K 00 . Then for n ≥ K we apply the


Triangle Inequality to get

|x0 − x00 | = |x0 − xn + xn − x00 |


ε ε
≤ |x0 − xn | + |xn − x00 | < + = ε.
2 2

Since ε > 0 is an arbitrary positive number, using the Theorem “If a ∈ R


is such that 0 ≤ a < ε for every positive ε, then a = 0”, we conclude
that |x0 − x00 | = 0. This implies x0 − x00 = 0 or x0 = x00 .

For x ∈ R and ε > 0, recall that the ε−neighborhood of x is the set

Vε (x) = {u ∈ R : |u − x| < ε}

Since u ∈ Vε (x) is equivalent to |u − x| < ε, the definition of convergence


of a sequence can be formulated in terms of neighborhoods.

Definition Let X = (xn ) be a sequence of real numbers. A real number


x is said to be a limit of (xn ) if for every ε > 0 there exists a natural
number K(ε) such that for all n ≥ K(ε), the terms xn belong to the
ε-neighborhoodVε (x).

We give several different ways of saying that a sequence xn converges


to x in the following theorem.

Theorem 2 Let X = (xn ) be a sequence of real numbers, and let x ∈ R.


The following statements are equivalent.

(a) X converges tox.

(b) For every ε > 0, there is a natural number K(ε) such that for all
n ≥ K(ε), the terms xn satisfy |xn − x| < ε.
8.1 Sequences and their Limits 141

(c) For every ε > 0, there is a natural number K(ε) such that for all
n ≥ K(ε), the terms xn satisfy x − ε < xn < x + ε.

(d) For every ε-neighborhood Vε (x) of x, there is a natural number K(ε)


such that for all n ≥ K(ε), the terms xn belong to Vε (x).

Proof The equivalence of (a) and (b) is just the definition. The
equivalence of (b), (c), and (d) follows from the following implications:

|xn − x| < ε ⇔ −ε < xn − x < ε ⇔ x − ε < xn < x + ε ⇔ xn ∈ Vε (x)

Conjecture a Value for Limit The definition of the limit of a sequence


of real numbers is used to verify that a proposed value x is indeed the
limit. It does not provide a means for initially determining what that
value of x should be. Later results will contribute to this end, but quite
often it is necessary in practice to arrive at a conjectured value of the
limit by direct calculation of a number of terms of the sequence. Com-
puters can be very helpful in this respect, but since they can calculate
only a finite number of terms of a sequence, such computations do not
in any way constitute a proof, but only a conjecture, of the value of the
limit.

Tails of Sequences

It is important to realize that the convergence (or divergence) of a se-


quence X = (xn ) depends only the “ultimate behavior” of the terms.
By this we mean that if, for any natural numberm, we drop the first m
terms of the sequence, then the resulting sequence Xm converges if and
only if the original sequence converges, and in this case, the limits are
the same.

Definition If X = (x1 , x2 , . . . , xn , . . .) is a sequence of real numbers


142 Chapter 8. Sequences of Real Numbers

and if m is a given natural number, then the m-tail of X is the sequence

Xm = (xm+n : n ∈ N) = (xm+1 , xm+2 , . . .).

For example, the 2-tail of the the sequence X = (3, 6, 9, . . . , 3n, . . .),
is the sequence X2 = (9, 12, . . . , 3n + 6, . . .).

Theorem 3 Let X = (xn : n ∈ N) be a sequence of real numbers and let


m ∈ N. Then the m-tail Xm = (xm+n : n ∈ N) of X converges if and
only if X converges. In this case, lim Xm = lim X.

Proof We note that for any p ∈ N, the pth term of Xm is the (p + m)the
term of X. Similarly, if q > m, then the qth term of X is the (q − m)th
term of Xm .

Assume X converges to x. Then given any ε > 0, if the terms of X for


n ≥ K(ε) satisfy |xn − x| < ε, then the terms of Xm for k ≥ K(ε) − m
satisfy |xk − x| < ε. Thus we can take Km (ε) = K(ε) − m, so that Xm
also converges to x.

Conversely, if the terms of Xm for k ≥ Km (ε) satisfy |xk − x| < ε,


then the terms of X for n ≥ Km (ε) + m satisfy |xn − x| < ε. Thus we
can take K(ε) = Km (ε) + m.

Therefore, X converges to x if and only if Xm converges to x. This


completes the proof.

Definition We shall sometimes say that a sequence X ultimately has


a certain property, if some tail of X has this property.

For example, we say that the sequence (3, 4, 5, 5, 5, . . . , 5, . . .) is


“ultimately constant”. On the other hand, the sequence (3, 5, 3, 5, . . . , 3, 5, . . .)
is not ultimately constant. The notion of convergence can be stated us-
ing this terminology. A sequence X converges to x if and only if the
8.1 Sequences and their Limits 143

terms of X are ultimately in every ε-neighborhood of x.

Theorem 4 Let (xn ) be a sequence of real numbers and let x ∈ R. If


(an ) is a sequence of positive real numbers with lim(an ) = 0 and if for
some constant C > 0 and some m ∈ N we have

|xn − x| ≤ Can for all n ≥ m,

then it follows that lim(xn ) = x.

Proof If ε > 0 is given, then since lim(an ) = 0, we know there exists


K = K(ε/C) such that n ≥ K implies

an = |an − 0| < ε/C.

Therefore it follows that if both n ≥ K and n ≥ m, then

|xn − x| ≤ Can < C(ε/C) = ε.

Since ε > 0 is arbitrary, we conclude that x = lim(xn ).


 
1
Example 12 If a > 0, then show that lim 1+na = 0.

Solution

Since a > 0, it follows that 0 < na < 1 + na, and therefore, 0 <
1 1
1+na < na , which evidently implies that

1
1+na − 0 ≤ a1 n1 for all n ∈ N.


Since lim n1 = 0, Theorem 4 with C = a1 and m = 1, implies




 
1
lim = 0.
1 + na

1

Example 13 Show that lim 2n = 0.
144 Chapter 8. Sequences of Real Numbers

1 1
Proof Since 0 < n < 2n for all n ∈ N we have 0 < 2n < n from which
it follows that
1 1
n − 0 ≤ for all n ∈ N.
2 n

Since we know lim n1 = 0, Theorem 4 with C = 1 and m = 1 implies




lim 21n = 0.


Example 14 If 0 < b < 1, then show that lim(bn ) = 0.


1 1
Proof Since 0 < b < 1, we can writeb = 1+a , where a = b − 1, so that
n
a > 0. By Bernoulli’s Inequality we have(1 + a) ≥ 1 + na. Hence

1 1 1
0 < bn = ≤ < .
(1 + a)n 1 + na na

1 1

Since we know lim n = 0, Theorem 4 with C = a and m = 1 implies
n
lim (b ) = 0.

Example 15 If c > 0, then show that lim(c1/n ) = 1.

Solution
1
Case 1) When c = 1, (c n ) is the constant sequence (1, 1, 1, . . .) and
it evidently converges to 1.

Case 2) If c > 1, then c1/n = 1 + dn for some dn > 0. Hence by


Bernoulli’s Inequality

c = (1 + dn )n ≥ 1 + ndn for n ∈ N.

Therefore we have
c − 1 ≥ ndn ,

so that
c−1
dn ≤ .
n
8.1 Sequences and their Limits 145

Consequently we have

|c1/n − 1| = dn ≤ (c − 1) n1 for n ∈ N.

Since, lim n1 = 0, Theorem 4 with C = c − 1 and m = 1 implies




lim(c1/n ) = 1 when c > 1.


1
Case 3) If c is such that 0 < c < 1; then c1/n = 1+hn for some hn > 0.
Hence by Bernoulli’s Inequality it follows that

1 1 1
c= n
≤ < ,
(1 + hn ) 1 + nhn nhn

1
from which it follows that 0 < hn < nc for n ∈ N. Therefore we have

hn 1
0 < 1 − c1/n = < hn <
1 + hn nc

so that

|c1/n − 1| < 1c n1 for n ∈ N.




Since lim n1 = 0, Theorem 4 with C = 1



c and m = 1 implies lim(c1/n ) =
1 when 0 < c < 1.

Example 16 Show that lim(n1/n ) = 1.

Solution

We note that n1/n > 1 for n > 1. Hence we can write n1/n = 1 + kn
for some kn > 0 when n > 1. Hence n = (1 + kn )n for n > 1. By the
Binomial Theorem, if n > 1 we have

1 1
n = 1 + nkn + n(n − 1)kn2 + · · · ≥ 1 + n(n − 1)kn2 ,
2 2
146 Chapter 8. Sequences of Real Numbers

and hence it follows that

n − 1 ≥ 12 n(n − 1)kn2 .

2
Hence kn2 ≤ n for n > 1. Now if ε > 0 is given, it follows from the
Archimedean Property that there exists a natural number Nε such that
2
Nε < ε2 . It follows that if n ≥ sup{2, Nε } then n2 < ε2 , and hence it
follows that  1/2
2
0 < n1/n − 1 = kn ≤ < ε.
n

Since ε > 0 is arbitrary, we conclude that lim(n1/n ) = 1.

Example 17 If lim (xn ) = x > 0, show that there exists a natural


number K such that if n ≥ K, then 12 x < xn < 2x.

Solution

Since lim (xn ) = x > 0, by the definition of limit, for every ε > 0
there exists a natural number K(ε) such that for all n ≥ K(ε), the terms
xn satisfy |xn − x| < ε.

Here x > 0.

1. As x > 0, we take ε = x. Then there exists a natural number K1 =


K(x) such that for all n ≥ K1 , the terms xn satisfy |xn − x| < x.
Thus for all n ≥ K1 , the terms xn satisfy xn − x < x. Adding x to
both sides of the inequality, it follows that

for all n ≥ K1 , the terms xn satisfy xn < 2x. . . . (i)

1. Also x2 > 0. Take ε = x2 . Then there exists a natural number K2 =


K(x/2) such that for all n ≥ K2 , the terms xn satisfy |xn − x| < x2 .
8.1 Sequences and their Limits 147

Thus for all n ≥ K2 , the terms xn satisfy − x2 < xn −x < x2 . Adding


x to both sides of the first inequality yields

x
for all n ≥ K2 , the terms xn satisfy 2 < xn . . . . (ii)

Take K = max{K1 , K2 }. Then from (i) and (ii) it follows that for all
n ≥ K, the terms xn satisfy 21 x < xn < 2x.

Exercises

1. The sequence (xn ) is defined by the following formulas for the nth
term. Write the first five terms in each case:

(a) xn = n + 1 (b) xn = n−1 n
(−1)n
(c) xn = 1 + (−1)n , (d) xn = n ,

1 1
(e) xn = n(n+1) , (f) xn = n2 +2 .

2. The first few terms of a sequence (xn ) are given below. Assuming that
the “natural pattern” indicated by these terms persists, give a formula
for the nth term xn .

(a) −1, − 12 , − 13 , − 14 , · · · (b) 5, 7, 9, 11, . . . ,

(c) 12 , − 41 , 1
8,
1
− 16 , · · · , (d) 12 , 2 3 4
3, 4, 5, ···,

(e) 1, 4, 9, 16, . . . (f) 3, 3, 3, · · · , 4

3. List the first five terms of the following inductively defined sequences.

(a) x1 = 1, xn+1 = xn + 1

(b) x1 = 1, xn+1 = 3xn + 1,

(c) y1 = 2, yn+1 = 21 (yn + 2


yn )
(zn+1 +zn )
(d) z1 = 1, z2 = 2, zn+2 = (zn+1 −zn )
148 Chapter 8. Sequences of Real Numbers

(e) s1 = 3, s2 = 5, sn+2 = sn + sn+1


 
2n
4. For any b ∈ R, prove that lim n+1 = 2.

5. Use the definition of the limit of a sequence to establish the following


limits.
   
n 2n
(a) lim n2 +1 = 0, (b) lim n+1 = 2,
   
3n+1 n2 −1
(c) lim 2n+5 = 32 , (d) lim 2n 2 +3 = 12 .

6. Show that
   
1 2n
(a) lim √n+7 = 0, (b) lim n+2 = 2,
√   n

n
(c) lim n+1 = 0, (d) lim (−1)
n2 +1
n
= 0.

7. Prove that lim(xn ) = 0 if and only if lim(|xn |) = 0. Give an example


to show that the convergence of (|xn |) need not imply the convergence
of (xn ).
√ 
8. Show that if xn ≥ 0 for all n ∈ N and lim(xn ) = 0, then lim xn =
0.

9. Prove that if lim(xn ) = x and if x > 0, then there exists a natural


number M such that xn > 0 for all n ≥ M .
 
1
10. Show that lim n1 − n+1 = 0.

11. Show that lim 31n = 0.




12. Let b ∈ R satisfy 0 < b < 1. Show that lim(nbn ) = 0. [Hint : Use
the Binomial Theorem]
1
13. Show that lim((2n) n ) = 1.
 2
14. Show that lim nn! = 0.
8.1 Sequences and their Limits 149

2n 2n
≤ 2( 32 )n−2 ]

15. Show that lim n! = 0. [Hint : if n ≥ 3, then 0 < n!

Answers
√ √ √ 1 2 3
1. (a) ( 2, 3, 4, · · · ) (b) 0, 2, 3, 4, ···

(c) 0, 2, 0, 2,0 (e)1/2, 1/6, 1/12, 1/20, 1/30

2. (a) xn = − n1 (f) xn = 3

3. (b) 1, 4, 13, 40,121 (d) 1,2,3,5,4

5. (a) We have 0 < n/(n2 + 1) < n/n2 = 1/n. Given ε > 0, let
K(ε) ≥ 1/e

(c) We have |(3n + 1)/(2n + 5) − 3/2| = 13/(4n + 10) < 13/4n.Given

ε = 0, let K(ε) ≥ 13/4ε.


√ √
6. (a) 1/ n + 7 < 1/ n

(b) |2n/(n + 2) − 2| = 4/(n + 2) < 4/n


√ √
(c) n/(n + 1) < 1/ n (d) |(−1)n n/(n2 + 1)| ≤ 1/n.


8.0 < xn < ε ⇔ 0 < xn < ε2 .

10.|1/n − 1/(n + 1)| = 1/n(n + 1) < 1/n2 ≤ 1/n

12. Let b = 1/(1 + a) where a > 0. Since (1 + a)n > 12 n(n − 1)a2 , we have
that 0 < nbn ≤ n/[ 12 n(n − 1)a2 ] ≤ 2/[(n − 1)a2 ]. Thus lim(nbn ) = 0.

14. If n > 3, then 0 < n2 /n! < n/(n − 2)(n − 1) < 1/(n − 3).

Objective Type Questions

1. Which one of the following is not a sequence?

(a) (1, 2, 1, 2,1, . . . ) (b) (1, 3, 5, 7, ... , (2n − 1), ...)


150 Chapter 8. Sequences of Real Numbers

1 1 1 1 1

(c) 2 , 6 , 12 , 18 , 24 , ... (d) {1, 2, 3, 4}
1
2. The first five terms of the sequence (xn ), where xn = n(n+1) , are

(a) 1, 1, 1, 1,1 (b) 21 , 1 1 1 1


3, 6, 7, 8

(c) 21 , 1 1 1 1
6 , 12 , 18 , 24 (d) none of these
1 2 3

3. nth term of the sequence 0, 2, 3, 4, · · · is
n+1 n−1
(a) n (b) n

n n
(c) n−1 (d) n+1

5 6 7
· · · , 1 + n2 , · · · is

4. The limit of the sequence 3, 2, 3, 4, 5,

(a) 1 (b) 2

(c) 3 (d) 4

5. The sequence { n}

(a) converges to 0 (b) converges to 1



(c) converges to 2 (d) diverges
 
1
6. If a > 0, then lim 1+na =

(a) 0 (b) 1

(c) 2 (d) doesn’t exist

Answers to Objective Type Questions

1. (d) 2. (d) 3. (b)

4. (a) 5. (d) 6. (a)


Chapter 9
Limit Theorems

9.1 Limit Theorems

In this chapter we shall obtain some results that often enable us to


evaluate the limits of certain sequences of real numbers. These results
will expand our collection of convergent sequences rather extensively.
We begin by the definition of bounded sequence.

Definition A sequence X = (xn ) of real numbers is said to be bounded


if there exists a real number M > 0 such that |xn | ≤ M for all n ∈ N.

Thus a sequence X = (xn ) is bounded if and only if the set {xn : n ∈


N} of its values is bounded in R.

Theorem 1 A convergent sequence of real numbers is bounded.

Proof Suppose that lim(xn ) = x and let ε = 1. Then there is a natural


number K = K(1) such that if n ≥ K then |xn − x| < 1. Hence, by the

151
152 Chapter 9. Limit Theorems

Triangle Inequality, if n ≥ K

|xn | = |xn − x + x| ≤ |xn − x| + |x| < 1 + |x| .

That is, if n ≥ K, |xn | < |x| + 1. If we set

M = sup{|x1 |, |x2 |, . . . , |xK−1 |, |x| + 1},

then it follows that |xn | ≤ M for all n ∈ N.

This completes the proof

Remark Converse of Theorem 1 is not true. For example, the sequence


(1, 2, 1, 2, . . . ) is bounded, but not convergent.

Example 1 The sequence (n) is divergent.

Solution

We know that every convergent sequence is bounded. Hence if the


sequence X = (n) is convergent, X = (n) must be bounded, so that
there exists a real number M > 0 such that n = |n| < M for all n ∈ N.
But this violates the Archimedean Property. This contradiction shows
that the sequence (n) is divergent.

Example 2 The sequence ((−1)n ) is bounded (take M = 1), so we


cannot readily get a conclusion using Theorem 1. However, assume that
a = lim X exists. Let ε = 1 so that there exists a natural number K1
such that

|(−1)n − a| < 1 for all n ≥ K1

If n is an odd natural number with n ≥ K1 , this gives | − 1 − a| < 1,


so that
| 1 + a| = | − (1 + a)| < 1
9.1 Limit Theorems 153

so that −1 < 1 + a < 1

and hence

−2 < a < 0. (9.1)

On the other hand, if n is an even natural number with n ≥ K1 , the


inequality |(−1)n − a| < 1 gives

|1 − a| < 1

so that −1 < 1 − a < 1

and hence −2 < −a < 0,

so multiplying by −1, we obtain

0 < a < 2. (9.2)

Since a cannot satisfy both of the inequalities (9.1) and (9.2), the hy-
pothesis that Xis convergent leads to a contradiction. Therefore the
sequence X is divergent.

Algebra of Sequences

Sometimes it may not be easy to prove the convergence of a sequence


by direct application of the definition of convergence. In such situations
if we represent the given sequence as a sum or difference or product of
sequences, then it may be easy to evaluate the limit of the sequence.

Theorem 2 (a) Let X = (xn ) and Y = (yn ) be sequences of real


numbers that converge to x and y, respectively, and let c ∈ R. Then the
sequences X + Y, X − Y, X · Y , and cX converge to x + y, x − y, xy,
and cx, respectively.
154 Chapter 9. Limit Theorems

(b) If X = (xn ) converges to x and Z = (zn ) is a sequence of nonzero


real numbers that converges to z and if z 6= 0, then the quotient sequence
X
Z converges to xz .

Proof (a) To show thatlim(xn + yn ) = x + y, we need to estimate the


magnitude of |(xn + yn ) − (x + y)|. To do this we use the Triangle
Inequality to obtain

|(xn + yn ) − (x + y)| = |(xn − x) + (yn − y)|


≤ |xn − x| + |yn − y|.

By hypothesis, if ε > 0 there exists a natural number K1 such that if


ε
n ≥ K1 , then |xn − x| < 2; also there exists a natural number K2 such
ε
that if n ≥ K2 , then |yn − y| < 2. Hence if K(ε) = sup{K1 , K2 }, it
follows that if n ≥ K(ε) then

|(xn + yn ) − (x + y)| ≤ |xn − x| + |yn − y|

1
< 2ε + 21 ε = ε.

Since ε > 0 is arbitrary, we have X + Y = (xn + yn ) converges to x + y.

Precisely the same argument can be used to show that X − Y =


(xn − yn ) converges to x − y.

To show that X ·Y = (xn yn ) converges to xy, we make the estimate

|xn yn − xy| = |(xn yn − xn y) + (xn y − xy)|


≤ |xn (yn − y)| + | (xn − x)y|
= |xn ||yn − y| + | xn − x| |y|. (9.3)

By Theorem 1, every convergent sequence of real numbers is bounded,


9.1 Limit Theorems 155

hence the convergence of (xn ) implies that there exists a real number
M1 > 0 such that

|xn | ≤ M1 for all n ∈ N.

Now we set
M = sup{M1 , |y|}.

Hence, from (9.3), we have the estimate

|xn yn − xy| ≤ M |yn − y| + M |xn − x|.

From the convergence of X and Y we conclude that if ε > 0 is given,


then there exist natural numbers K1 and K2 such that if n ≥ K1 then
ε ε
|xn − x| < 2M , and if n ≥ K2 then |yn − y| < 2M . Now let K(ε) =
sup{K1 , K2 }; then, if n ≥ K(ε) we infer that

|xn yn − xy| ≤ M |yn − y| + M |xn − x|


 ε   ε 
< M +M =ε
2M 2M
Since ε > 0 is arbitrary, this proves that the sequence X · Y = (xn yn )
converges to xy.

The fact that cX = (cxn ) converges to cx can be proved in the same


way; it can also be deduced by taking Y to be the constant sequence
(c, c, c, . . .). The details are left as an exercise.

(b) We next show that if Z = (zn ) is a sequence of nonzero


  real numbers
1
that converges to a nonzero limit z, then the sequence zn of reciprocals
1
converges to z. First let
α = 12 |z|

so that α > 0. Since lim(zn ) = z, there exists a natural number K1 such


156 Chapter 9. Limit Theorems

that if n ≥ K1 then |zn − z| < α. Now using the result “if a, b ∈ R,


then| |a| − |b| | ≤ |a − b|”, we have | |zn | − |z | | < |zn − z|,

i.e., −|zn − z| < |zn | − |z | < |zn − z |.

Hence it follows that

−α ≤ −|zn − z| ≤ |zn | − |z| for n ≥ K1 ,

whence it follows that 12 |z| = |z| − α ≤ |zn | for n ≥ K1 . Therefore


1 2
|zn | ≤ |z| for n ≥ K1 so we have the estimate

− 1 = z − zn =
1 1
|z − zn |
zn z zn z |zn z|

2
≤ |z|2 |z − zn | for all n ≥ K1 .

Now, if ε > 0 is given, there exists a natural number K2 such that if


n ≥ K2 then
1 2
|zn − z| < 2 ε|z| .

Therefore, it follows that if we let

K(ε) = sup{K1 , K2 },

then

1
zn − z1 ≤ ε for all n > K(ε).

Since ε > 0 is arbitrary, it follows that


 
1 1
lim = .
zn z
 
1
The proof of (b) is now completed by taking Y to be the sequence zn
9.1 Limit Theorems 157
 
xn 1
= xz .

and using the fact that X · Y = zn converges tox z

By mathematical induction, some of the results in Theorem 2 can be


extended to a finite number of convergent sequences as described in the
following corollary.

Corollary If A = (an ), B = (bn ), . . . , Z = (zn ) are convergent se-


quences of real numbers, then their sum

A + B + · · · + Z = (an + bn + · · · + zn )

is a convergent sequence and

lim(an + bn + · · · + zn ) = lim(an ) + lim(bn ) + · · · + lim(zn ).

Also their product

A · B · · · Z = (an bn · · · zn )

is a convergent sequence and

lim(an bn · · · zn ) = (lim (an )) (lim(bn )) · · · (lim(zn )).

Remark In particular, if k ∈ N and if A = (an ) is a convergent sequence,


then
lim(akn ) = (lim(an ))k .
2n+1

Example 3 Show that lim n =2

Solution

If we let X = (2), the constant sequence with its entries 2, and Y =


1
, then 2n+1
 
n n = X + Y . Hence it follows that lim(X + Y ) = lim X+
158 Chapter 9. Limit Theorems

lim Y = 2 + 0 = 2.
 
2n+1
Example 4 Show that lim n+5 =2

Solution

Since the sequences (2n + 1) and (n + 5) are not convergent it is not


possible readily to use limit of quotient of sequence Theorem [Part (b)
of Theorem 2] directly. However, if we write

1
2n + 1 2+ n
= 5
n+5 1+ n

we can obtain the given sequence as one to which Theorem 2 applies when
we take X = 2 + n1 andZ = 1 + n5 . [The terms of the sequence Z
 

never become 0. Hence all the hypotheses of Theorem 2 are satisfied].


Since lim X = 2 and lim Z = 1 6= 0 we deduce that we have
 
2n + 1 2
lim = = 2.
n+5 1
 
2n
Example 5 lim n2 +1 =0

We cannot apply Theorem 2 directly as both sequences (2n) and


2
(n + 1) are divergent. We note that

2n 2
= 1 .
n2 + 1 n+ n

Still Theorem 2 cannot be applied here because n + n1 is not a conver-




gent sequence. [One argument is this. If Z = n + n1 is convergent, the




convergence of Y = n1 implies Z − Y is convergent. But Z − Y = (n)




which is known to be divergent. Hence a contradiction.] However, if we


9.1 Limit Theorems 159

write
2
2n n
=
n2 + 1 1 + n12
2 1
 
then we can apply the theorem since lim n = 0 and lim 1 + n2 =
1 6= 0. Therefore  
2n 0
lim = = 0.
n2 + 1 1
Example 6 Let X = (xn ) be a sequence of real numbers that converges
tox ∈ R. Let p be a polynomial; for example, let

p(t) = ak tk + ak−1 tk−1 + · · · + a1 t + a0 ,

where k ∈ N and aj ∈ R for j = 0, 1, . . . , k. It follows, from Theorem


2, that the sequence (p(xn )) converges to p(x).

Example 7 Let X = (xn ) be a sequence of real numbers that converges


p(t)
to x ∈ R. Let r be a rational function (that is, r(t) = q(t) , where p
and q are polynomials). Suppose that q(xn ) 6= 0 for all n ∈ N and that
p(x)
q(x) 6= 0. Then the sequence (r(xn )) converges to r(x) = q(x) .

Theorem 3 If X = (xn ) is a convergent sequence of real numbers and


if xn ≥ 0 for all n ∈ N, then x = lim(xn ) ≥ 0.

Proof Suppose the conclusion is not true and that x < 0; then ε = −x
is positive. Since X converges to x, there is a natural number K such
that

|x − xn | < ε for all n ≥ K

i.e., such that −ε < xn − x < ε for all n ≥ K

i.e., such that x − ε < xn < x + ε for all n ≥ K.

In particular, we have xK < x + ε = x + (−x) = 0. But this contradicts


160 Chapter 9. Limit Theorems

the hypothesis that xn ≥ 0 for all n ∈ N. Therefore this contradiction


implies that x ≥ 0. This completes the proof.

Remark Since any tail of a convergent sequence has the same limit,
the hypotheses of Theorem 3 can be weakened to apply to the tail of a
sequence. i.e., if X = (xn ) is ultimately positive in the sense that there
exists m ∈ N such that xn ≥ 0 for all n ≥ m, then the limit x ≥ 0 will
hold.

We now give a useful result that is formally stronger than Theorem 3.

Theorem 4 If X = (xn ) and Y = (yn ) are convergent sequences of real


numbers and if xn ≤ yn for all n ∈ N, then lim(xn ) ≤ lim(yn ).

Proof Let zn = yn − xn so that Z = (zn ) = Y − X and zn ≥ 0 for all


n ∈ N. It follows from Theorems 2 and 3 that

0 ≤ lim Z = lim(yn ) − lim(xn ),

so that lim(xn ) ≤ lim(yn ). This completes the proof.

The next result says that if all the terms of a convergent sequence
satisfy an inequality of the form a ≤ xn ≤ b, then the limit of the
sequence satisfies the same inequality. Thus if the sequence is convergent,
one may “pass to the limit” in an inequality of this type.

Theorem 5 If X = (xn ) is a convergent sequence and if a ≤ xn ≤ b for


all n ∈ N, thena ≤ lim(xn ) ≤ b.

Proof Let Y be the constant sequence(b, b, b, . . .). Using Theorem 4,


we have
lim X ≤ lim Y = b.

Next, let Z be the constant sequence (a, a, a, · · · ). Then it follows


9.1 Limit Theorems 161

from Theorem 4 that


a = lim Z ≤ lim X.

Combining, we obtain

a ≤ lim X ≤ b.

The next result asserts that if a sequence Y is squeezed between two


sequences that converge to the same limit, then it must also converge to
this limit.

Theorem 6 (Squeeze Theorem) Suppose that X = (xn ), Y = (yn ),


and Z = (zn ) are sequences of real numbers such that

xn ≤ yn ≤ zn for all n ∈ N,

and that lim(xn ) = lim(zn ). Then Y = (yn ) is convergent and

lim(xn ) = lim(yn ) = lim(zn ).

Proof Let w = lim(xn ) = lim(zn ). If ε > 0 is given, then it follows from


the convergence of X and Z to w that there exists a natural number K
such that if n ≥ K then

|xn − w| < ε and |zn − w| < ε. (9.4)

Hence if n ≥ K then

−ε < xn − w < ε and − ε < zn − w < ε. (9.5)

From the hypothesis we have xn ≤ yn ≤ zn for all n ∈ N. Hence

xn − w ≤ yn − w ≤ zn − w for all n ∈ N. (9.6)


162 Chapter 9. Limit Theorems

From (9.5) and (9.6) it follows that

−ε < yn − w < ε

for all n ≥ K. i.e.,

|yn − w| < ε for all n ≥ K.

Since ε > 0 is arbitrary, this implies that lim(yn ) = w.

Example 8 lim sinn n = 0.




Solution

We cannot apply Part (b) of the Theorem 2 directly, since neither


the sequence (sin n) nor the sequence (n) is convergent. It does not
appear that a simple algebraic manipulation will enable us to reduce
the sequence into one to which Theorem 2 will apply. Let us try with
Squeeze Theorem (Theorem 6). We note that −1 ≤ sin n ≤ 1, and hence
it follows that

− n1 ≤ sin n
n ≤ 1
for all n ∈ N.
n

Hence, noting that lim n1 = 0, we can apply the Squeeze Theorem to




infer that lim sinn n = 0.




Theorem 7 Let the sequence X = (xn ) converge to x. Then the sequence


(|xn |) of absolute values converges to |x|. That is, if x = lim(xn ), then
|x| = lim(|xn |).

Proof It follows from the Corollary 1 to Triangle Inequality (in page


66) that

| | xn | − |x| | ≤ |xn − x| for all n ∈ N.

The convergence of (|xn |) to |x| is then an immediate consequence of the


9.1 Limit Theorems 163

convergence of (xn ) to x. [Explanation : Let ε > 0. Since (xn ) converges


to x, there exists K(ε) such that |xn − x| < ε for all n ≥ K(ε). Then
| |xn | − |x | | < ε for all n ≥ K(ε). Hence lim (|xn |) = |x|.]

Theorem 8 Let X = (xn ) be a sequence of real numbers that converges


√ 
to x and suppose that xn ≥ 0. Then the sequence xn of positive
√  √
square roots converges and lim xn = x.

Proof Since xn ≥ 0 for all n, it follows from Theorem 3 that x =


lim(xn ) ≥ 0 so the assertion makes sense. We now consider the two
cases: (i) x = 0 and (ii) x > 0.

(i) If x = 0, let ε > 0 be given. Since xn → 0 there exists a natural


number K such that if n ≥ K then |xn − 0| < ε2 . Hence noting that
xn ≥ 0,
0 ≤ xn = xn − 0 < ε2 .

Therefore, using Example 4 of chapter 4, 0 ≤ xn < ε for n ≥ K. Since

ε > 0 is arbitrary, this implies that xn → 0.

(ii) If x > 0, then x > 0 and we note that
√ √  √ √ 
√ √ xn − x xn + x xn − x
xn − x = √ √ = √ √
xn + x xn + x
√ √ √
Since xn + x≥ x > 0, it follows that


 
√ 1
xn − x ≤ √ |xn − x|. (9.7)
x
√ √
The convergence of xn → x is now an easy consequence of the fact
that xn → x. [Explanation : Let ε > 0. Since (xn ) converges to x, there

exists K(ε) such that |xn − x| < x ε for all n ≥ K(ε). Then from (9.7),
164 Chapter 9. Limit Theorems

xn − √x < ε for all n ≥ K(ε).


√ √
Hence lim ( xn ) = x.]
 
Example 9 Show that lim √1n = 0.

Solution
1 1

havefor all n ∈ N,
We  n > 0 and lim n = 0. Hence by Theorem 8,

lim √1n = 0 = 0.

For certain types of sequences, the following result provides a quick


and easy “ratio test” for convergence.

Theorem 9 (Ratio Test) Let (xn ) be a sequence of positive real numbers


xn+1
such that L = lim xn exists. If L < 1, then (xn ) converges and
lim(xn ) = 0.
 
xn+1 xn+1
Proof Since xn ≥ 0 for all n, by Theorem 3, lim xn ≥ 0 and
hence it follows that L ≥ 0. By assumption L < 1. Let r be a number
such that L < r < 1, and let ε = r − L > 0. Then there exists a number
K ∈ N such that if n ≥ K then

xn+1

xn − L < ε.

It follows from this that if n ≥ K, then

xn+1
−ε < − L < ε.
xn
xn+1
i.e., if n ≥ K, xn < L + ε = L + (r − L) = r.

Therefore, if n ≥ K, we obtain

0 < xn+1 < xn r < xn−1 r2 < · · · < xK rn−K+1 .


9.1 Limit Theorems 165

xK
If we setC = rK
, we see that 0 < xn+1 < Crn+1 for alln ≥ K. Since
0 < r < 1, it follows from Example 14 of chapter 8 (page 144), that
lim(rn ) = 0 and therefore if follows, from Theorem 4, that lim(xn ) = 0.

Example 10 Show that lim 2nn = 0.




Solution
n
Let the given sequence be (xn ). Then xn = 2n . We have

2n
 
xn+1 n+1 1 1
= n+1 · = 1+ .
xn 2 n 2 n
 
so that lim xxn+1
n
= 1
2. Since 1
2 < 1, it follows from ratio test that
lim 2nn = 0.


Exercises

1. For xn given by the following formulas, establish either the conver-


gence or divergence of the sequence X = (xn ):
n (−1)n n
(a) xn = n+1 , (b) xn = n+1 ,

n2 2n2 +3
(c) xn = n+1 (d) xn = n2 +1 .

2. Give an example of two divergent sequences X, Y such that their


sum X + Y converges.

3. Give an example of two divergent sequences X, Y such that their


product XY converges.

4. Show that if X and Y are sequences such that X and X + Y are


convergent, then Y is convergent.

5. Show that if X and Y are sequences such that X converges to x 6= 0


and XY converges, then Y converges.
166 Chapter 9. Limit Theorems

6. Show that the sequence (2n ) is not convergent.

7. Show that the sequence ((−1)n n2 ) is not convergent.

8. Find the limits of the following sequences:


 2   n

(a) lim 2 + n1 , (b) lim (−1)
(n+2)
√   
(c) lim √n−1
n+1
, (d) lim nn+1

n
√ √ √
9. Let yn = n + 1 − n for n ∈ N. Show that (yn ) and ( nyn )
converge.
1
10. Show that if zn = (an + bn ) n where 0 < a < b, then lim(zn ) = b.

11. Apply ratio test to the following sequences, where a, b satisfy 0 <
a < 1, b > 1.
 
bn n 23n
 
(a) (an ), (b) 2n (c) bn (d) 32n

12. Give an example


  of a convergent sequence(xn ) of positive numbers
xn+1
such that lim xn = 1.

13. Give an example


  of a divergent sequence (xn ) of positive numbers
xn+1
such that lim xn = 1.

14. Let X = (xn ) be a sequence of positive real numbers such that


xn+1
lim xn = L > 1. Show that X is not a bounded sequence and
hence is not convergent.

15. Discuss the convergence of the following sequences, where a, b satisfy


0 < a < 1, b > 1.
bn bn n!
  
(a) (n2 an ), (b) n2 (c) n! (d) nn .
 1
16. Let (xn ) be a sequence of positive real numbers such that lim xnn =
L < 1. Show that there exists a number r with 0 < xn < rn for all suf-
9.1 Limit Theorems 167

ficiently large n ∈ N. Use this to show that lim(xn ) = 0.

17. Give an example


  of a convergent sequence (xn ) of positive numbers
1/n
such that lim xn = 1.

18. Give an example


 of a divergent sequence (xn ) of positive real num-
1/n
bers such thatlim xn = 1.

19. Suppose that (xn ) is a convergent sequence and (yn ) is such that for
any ε > 0 there exists M such that |xn − yn | < ε for all n ≥ M . Does
it follow that (yn ) is convergent? Explain.
 n+1 n+1 
20. If 0 < a < b, show that lim a an +b
+bn = b.

21. Use Squeeze Theorem to prove the following:


 2
  2

(a) lim n1/n = 1. (b) lim (n!)1/n = 1.

Answers

1. (a) lim(xn ) = 1 (c) xn ≥ n/2, so the sequence diverges

2. Take X = ((−1)n ), Y = ((−1)n+1 ), then X + Y = (0, 0, 0, . . .)

3. Take X = ((−1)n ), Y = ((−1)n+1 ), then XY = (−1, −1, . . .)

4. Y = (X + Y ) − X.

8. (a) 4 (b) 0 (c) 1 (d) 0.



9. lim(yn ) = 0 and lim( nyn ) = 12 .
b 1 8
11. (a) L = a (b) L = 2 (c) L = b (d) L = 9

15. (a) Converges to 0 (c) Converges to 0.

17. (1)
18.(n)
168 Chapter 9. Limit Theorems

19. Yes.
Chapter 10
Monotone Sequences

10.1 Monotone Sequences

There are many instances, however, in which there is no obvious candi-


date for the limit of a sequence, even though a preliminary analysis may
suggest that convergence probably does take place. We discuss these
situations now.

Definition Let X = (xn ) be a sequence of real numbers. We say that


X is increasing if it satisfies the inequalities

x1 ≤ x2 ≤ · · · ≤ xn ≤ xn+1 ≤ · · ·

We say that X is decreasing if it satisfies the inequalities

x1 ≥ x2 ≥ · · · ≥ xn ≥ xn+1 ≥ · · ·

169
170 Chapter 10. Monotone Sequences

We say that X is monotone if it is either increasing, or it is decreasing.

The following sequences are increasing:

(1, 2, 3, 4, . . . , n, . . .),

(1, 2, 2, 3, 3, 3, . . .),

(a, a2 , a3 , . . . , an , . . .) if a > 1.

The following sequences are decreasing:


 
1 1 1
1, , , · · · , , . . . ,
2 3 3
 
1 1 1
1, , 2 , · · · , n−1 , . . . ,
2 2 2

(b, b2 , b3 , . . . , bn . . .) if 0 < b < 1.

The following sequences are not monotone:

(+1, −1, +1, . . . , (−1)n+1 , . . .),

(−1, +2, −3, . . . , (−1)n n, . . .).

The following sequences are not monotone, but they are “ultimately”
monotone:
(7, 6, 2, 1, 2, 3, 4, . . .),
 
1 1 1
−2, 0, 1, , , , ··· .
2 3 4
Theorem 1 (Monotone Convergence Theorem) A monotone se-
quence of real numbers is convergent if and only if it is bounded. Further:
10.1 Monotone Sequences 171

(a) If X = (xn ) is a bounded increasing sequence, then

lim(xn ) = sup{xn : n ∈ N}

(b) If Y = (yn ) is a bounded decreasing sequence, then

lim(yn ) = inf{yn : n ∈ N}.

Proof First of all we note that a convergent sequence is bounded.

Conversely, let X be a bounded monotone sequence. Then X is either


increasing or decreasing.

(a) We first treat the case where X = (xn ) is a bounded increasing


sequence. By hypothesis, there exists a real number M such that xn ≤ M
for all n ∈ N. Thus the subset {xn : n ∈ N} of the set of real numbers
is bounded above. Hence by the Supremum Property of real numbers,
supremum of the set {xn : n ∈ N} exists. Let

x∗ = sup{xn : n ∈ N}.

Claim x∗ = lim(xn ).

If ε > 0 is given, then x∗ − ε is not an upper bound for the set


{xn : n ∈ N}, and hence there exists a natural number K = K(ε) such
that xK is a member of the set {xn : n ∈ N} and x∗ − ε < xK . But
since (xn ) is an increasing sequence it follows that

x ∗ −ε < xK ≤ xn ≤ x∗ for all n ≥ K.

Therefore it follows that x∗ − ε < xn < x∗ + ε for all n ≥ K,

i.e., −ε < xn − x∗ < ε for all n ≥ K


172 Chapter 10. Monotone Sequences

i.e., |xn − x∗ | < ε for all n ≥ K.

Since ε > 0 is arbitrary, we have (xn ) converges to x∗.

(b) If Y = (yn ) is a bounded decreasing sequence, then it is clear that


X = −Y = (−yn ) is a bounded increasing sequence. We have seen in
part (a) that lim X = sup{−yn : n ∈ N}. On the other hand, lim X =
− lim Y ; and also [using the result in Chapter 5 (page 5.1): “Let S be a
nonempty subset of R that is bounded below. Then inf S = − sup{−s :
s ∈ S}], we have

sup{−yn : n ∈ N} = − inf{yn : n ∈ N}.

Therefore

lim Y = − lim X = − sup {−yn : n ∈ N} = inf{yn : n ∈ N}.

This completes the proof.

Remark The Monotone Convergence Theorem establishes the existence


of the limit of a bounded monotone sequence. It also gives us a way
of calculating the limit of the sequence provided we can evaluate the
supremum in case (a), or the infimum in case (b). Sometimes it is
difficult to evaluate this supremum (or infimum), but once we know
that it exists, it is often possible to evaluate the limit by other methods.
 
Example 1 Show that lim √1n = 0.

Solution
 
We know that X = √1 is bounded and decreasing. Hence by part
n
10.1 Monotone Sequences 173

(b) of Theorem 1,
   
1 1
lim √ = inf √ :n∈N . (10.1)
n n
o n
Clearly 0 is a lower bound for the set √1
: n ∈ N , and it is not difficult
n
n o
to show that 0 is the infimum of the set √1n : n ∈ N ; hence (10.1)
 
implies lim √1n = 0.
 
Aliter We know that X = √1n is bounded and decreasing, and hence
 
it converges to some real numberx. Since X = √1n converges to x,
it follows, from Part (a) of Theorem 2 of chapter 9 (page 153), that
X · X = n1 converges to x2 . As n1 converges to 0, we have x2 = 0,
 

and hence x = 0.

Example 2 Examine the convergence of the sequence(xn ), where


1 1 1
xn = 1 + 2 + 3 + ··· + n for n ∈ N.

Solution
1
Since xn+1 = xn + n+1 > xn , we see that (xn ) is an increas-
ing sequence. By the Monotone convergence Theorem the question of
whether the sequence is convergent or not is reduced to the question
of whether the sequence is bounded or not. [Attention! Attempts to
use direct numerical calculations to arrive at a conjecture concerning
the possible boundedness of the sequence (xn ) lead to inconclusive frus-
tration. A computer run will reveal the approximate values xn ≈ 11.4
forn = 50, 000, and xn ≈ 12.1 forn = 100, 000. Such numerical facts
may lead the casual observer to conclude that the sequence is bounded,
which is not true]. However, the sequence is in fact divergent, which is
174 Chapter 10. Monotone Sequences

established by noting that


   
1 1 1 1 1
x 2n = 1+ + + + ··· + + ··· + n
2 3 4 2n−1 + 1 2
   
1 1 1 1 1
> 1+ + + + ··· + + ··· + n
2 4 4 2n 2
| {z } | {z }
1/2 2n−1 /2n =1/2

1 1 1
= 1+ + + ··· +
2 2 2
n
= 1+ .
2

Hence the sequence (xn ) is unbounded, and therefore by Theorem 1


it is divergent.

Example 3 Consider the sequence (xn ) defined inductively by

1
x1 = 2; xn+1 = 2 + , n ∈ N.
xn

Assuming the convergence of the sequence (xn ), show that lim(xn ) =



1 + 2.

Solution

If we let x = lim(xn ), then we also have x = lim(xn+1 ) since the 1-


tail (xn+1 ) converges to the same limit. Further, we see that xn ≥ 2, so
that x = lim xn ≥ 2 and xn 6= 0 for all n ∈ N. Therefore, we may apply
the limit theorems for sequences to obtain.

1 1
x = lim(xn+1 ) = 2 + =2+ .
lim(xn ) x

Thus, the limit x is a solution of the quadratic equation x2 − 2x − 1 = 0,


√ √
which gives x = 1 ± 2 of which x = 1 + 2 is possible (since xn ≥ 2
10.1 Monotone Sequences 175

and hence x must be ≥ 2).

Attention! The issue of convergence must not be ignored or casually


assumed. The following example illustrates this.

Example 4 (Convergence is important, other wise absurd conclusions


will be obtained ) Consider the sequence (yn ) defined by y1 = 1, yn+1 =
2yn + 1. Assuming the ‘convergence’ (actually wrong! The sequence is
not convergent) with lim(yn ) = y, we would obtain y = 2y + 1, so that
y = −1. Of course, this is absurd as always yn ≥ 1 and hence y must be
≥ 1.

Remark The absurdity in the above Example is due to the wrong as-
sumption that the sequence is convergent. Hence, hereinafter we first
examine the convergence of the sequence before finding its limit.

Example 5 Let Y = (yn ) be defined inductively by y1 = 1, yn+1 =


1
4 (2yn + 3) for n ≥ 1. Show that lim Y = 32 .

Solution

Direct calculation shows that y2 = 54 . Hence we have y1 < y2 < 2.


We show, by induction, that yn < 2 for all n ∈ N. Indeed this is true for
n = 1, 2. If yk < 2 holds for somek ∈ N, then

yk+1 = 14 (2yk + 3) < 1


4 (4 + 3) = 7
4 < 2,

so that yk+1 < 2. Therefore yn < 2 for all n ∈ N.

We now show, by induction, that yn < yn+1 for all n ∈ N. The truth
of this assertion has been verified for n = 1. Now suppose that yk < yk+1
for somek; then2yk + 3 < 2yk+1 + 3, whence it follows that

yk+1 = 41 (2yk + 3) < 41 (2yk+1 + 3) = yk+2 .


176 Chapter 10. Monotone Sequences

Thus yk < yk+1 implies that yk+1 < yk+2 . Therefore yn < yn+1 for all
n ∈ N.

We have shown that the sequence Y = (yn ) is increasing and bounded


above by 2. It follows from the Monotone convergence Theorem that Y
converges to a limit that is at most 2. In this case it is not so easy
to evaluate lim(yn ) by calculating sup{yn : n ∈ N}. However, there is
another way to evaluate its limit. Since yn+1 = 14 (2yn + 3) for all n ∈ N,
the nth term in the 1-tail Y1 of Y has a simple algebraic relation to the
nth term of Y . Since, we have y = lim Y1 = lim Y , it therefore follows,
from Theorem 2 of chapter 9 (page 153), that

y = 41 (2y + 3),

from which it follows that y = 32 .

Example 6 Let Z = (zn ) be the sequence of real numbers defined by



z1 = 1, zn+1 = 2zn for n ∈ N. Show that lim(zn ) = 2.

Solution

Note that z1 = 1 and z2 = 2; hence 1 ≤ z1 < z2 < 2. We claim that
the sequence Z is increasing and bounded above by 2. To show this we
will show, by induction, that 1 ≤ zn < zn+1 < 2 for all n ∈ N. This
fact has been verified for n = 1. Suppose that it is true for n = k; then
2 ≤ 2zk < 2zk+1 < 4, whence it follows that
√ √ p √
1< 2 ≤ zk+1 = 2zk < 2zk+1 < 4 = 2.


Noting that zk+2 = 2zk+1 , the above implies

1 ≤ zk+1 < zk+2 < 2.


10.1 Monotone Sequences 177

Hence the validity of the inequality1 ≤ zk < zk+1 < 2, implies the
validity of 1 ≤ zk+1 < zk+2 < 2. Therefore 1 ≤ zn < zn+1 < 2 for all
n ∈ N.

Since Z = (zn ) is a bounded increasing sequence, it follows from


the Monotone convergence Theorem that it converges to a number z =
sup{zn }. It may be shown directly that sup{zn } = 2, so that z = 2.
Alternatively we may use the method employed in Example 3. The

relation zn+1 = 2zn gives a relation between the nth term of the 1-tail
Z1 of Z and the nth term ofZ. Hence we have lim Z1 = z = lim Z.
Moreover, it follows that the limit z must satisfy the relation

z= 2z.

Hence z must satisfy the equation z 2 = 2z which has the roots z = 0, 2.


Since the terms of z = (zn ) all satisfy 1 ≤ zn ≤ 2, it follows that we
must have 1 ≤ z ≤ 2. Therefore z = 2.

The Calculation of Square Roots

Example 7 Let a > 0; we will construct a sequence (sn ) of real numbers



that converges to a.
 
Let s1 > 0 be arbitrary and define sn+1 = 12 sn + san forn ∈ N.

We now show that the sequence (sn ) converges to a.

We first show that s2n ≥ a forn ≥ 2. Since sn satisfies the quadratic


equation s2n − 2sn+1 sn + a = 0, this equation has a real root. Hence
the discriminant 4s2n+1 − 4a must be nonnegative; that is s2n+1 ≥ a for
n ≥ 1.
178 Chapter 10. Monotone Sequences

To see that (sn ) is ultimately decreasing, we note that for n ≥ 2 we


have
(s2n − a)
 
1 a 1
sn − sn+1 = sn − sn + = · ≥ 0.
2 sn 2 sn
Hence, sn+1 ≤ sn for all n ≥ 2. It follows from the Monotone Conver-
gence Theorem that s = lim(sn ) exists. Moreover, it follows that the
limit s must satisfy the relation

1 a
s= s+ .
2 s

whence it follows that


1 1a a
2s = or s =
2s s or s2 = a.

Thus s = a.

For the purposes of calculation, it is often important to have an estimate



of how rapidly the sequence (sn ) converges to a. As above, we have
√ √
a ≤ sn for all n ≥ 2, whence it follows that san ≤ a ≤ sn . Thus we
have
√ a (s2n −a)
0 ≤ sn − a ≤ sn − sn for n ≥ 2.
= sn

Using this inequality we can calculate a to any desired degree of accu-
racy.

Euler’s Number

We conclude this chapter by introducing a sequence that converges to


one of the most important “transcendental” numbers in mathematics,
second in importance only to π.
n
Example 8 Let en = 1 + n1 for n ∈ N. We will now show that the
sequence E = (en ) is bounded and increasing; hence it is convergent.
The limit of this sequence is the famous Euler number e, whose ap-
10.1 Monotone Sequences 179

proximate value is 2.718281 828 459045 . . . , which is taken as the base


of the “natural” logarithm.

If we apply the Binomial Theorem, we have


 n
1 n 1 n(n − 1) 1 n(n − 1)(n − 2) 1
en = 1+ =1+ · + · 2+ · 3
n 1 n 2! n 3! n

n(n − 1) · · · 2 · 1 1
+ ··· + · n.
n! n
If we divide the powers of n into the terms in the numerators of the
binomial coefficients, we get
    
1 1 1 1 2
en = 1 + 1 + 1− + 1− 1−
2! n 3! n n
    
1 1 2 n−1
+ ··· + 1− 1− ··· 1 − .
n! n n n
Similarly we have
    
1 1 1 1 2
en+1 =1+1+ 1− + 1− 1−
2! n+1 3! n+1 n+1
    
1 1 2 n−1
+ ··· + 1− 1− ··· 1 −
n! n+1 n+1 n+1
    
1 1 2 n
+ 1− 1− ··· 1 − .
(n + 1)! n+1 n+1 n+1

Note that the expression for en contains n + 1 terms, while that for
en+1 contains n + 2 terms. Moreover, each term appearing in en is less
than or equal to the corresponding term in en+1 , and en+1 has one more
positive term. Therefore we have 2 ≤ e1 < e2 < · · · < en < en+1 < · · · ,
so that the terms of E are increasing.
180 Chapter 10. Monotone Sequences

To show that the terms of E are bounded above, we note that if


p 1 1
p = 1, 2, . . . , n, then1 − n < 1. Moreover 2p−1 ≤ p! so that p! ≤ 2p−1 .
Therefore, if n > 1, then we have

1 1 1
2 < en < 1 + 1 + + 2 + · · · + n−1 .
2 2 2

Being the finite sum of ‘geometric progression’ with common ratio 12 , we


have
1 1 1 1
+ + · · · + n−1 = 1 − n−1 < 1,
2 22 2 2
and hence we deduce that 2 < en < 3 for all n ∈ N. The Monotone
Convergence Theorem implies that the sequence E converges to a real
number that is between 2 and 3. We define the number e to be the limit
of this sequence.

By refining our estimates we can find closer rational approximations to


e, but we cannot evaluate it exactly, since e is an irrational number.
However, it is possible to calculate e to as many decimal places as desired.

Exercises
1
1. Let x1 > 1 and xn+1 = 2 − xn for n ≥ 2. Show that (xn ) is bounded
and monotone. Find the limit.

2. Let y1 = 1 and yn+1 = 2 + yn . Show that (yn ) is convergent and
find the limit.
1
3. Let a > 0 and let z1 > 0. Define zn+1 = (a + zn ) 2 for n ∈ N. Show
that (zn ) converges and find the limit.
1
4. Let x1 = a > 0 and xn+1 = xn + xn . Determine if (xn ) converges or
diverges.

5. Let (xn ) be a bounded sequence, and for each n ∈ N let sn =


10.1 Monotone Sequences 181

sup{xk : k ≥ n} and tn = inf{xk : k ≥ n}. Prove that (sn ) and


(tn ) are convergent. Also prove that if lim(sn ) = lim(tn ), then (xn ) is
convergent. [One calls lim(sn ) the limit superior of (xn ), and lim(tn )
the limit inferior of (xn ).]

6. Let (an ) be an increasing sequence, (bn ) a decreasing sequence, and


assume that an ≤ bn for all n ∈ N. Show that lim(an ) ≤ lim(bn ),
and thereby deduce the Nested Intervals Theorem from the Monotone
Convergence Theorem

7. Let A be an infinite subset of R that is bounded above and let


u = sup A. Show there exists an increasing sequence (xn ) with xn ∈ A
for all n ∈ N such that u = lim(xn ).

8. Establish the convergence or the divergence of the sequence (yn ),


where
1 1 1
yn := n+1 + n+2 + ··· + 2n for n ∈ N.
1 1 1
9. Let xn := 12 + 22 + ··· + n2 for eachn ∈ N. Prove that (xn ) is
increasing and bounded, and hence converges. [Hint: Note that if k ≥ 2,
1
then k2 ≤ k1 (k − 1) = 1
(k−1) − k1 ]

10. Establish the convergence and find the limits of the following se-
quences.
 n+1   2n 
(a) 1 + n1 (b) 1 + n1
 n  n 
1
(c) 1 + n+1 (d) 1 − n1

11. Calculate the number en for n = 2, 4, 8, 16.

12. Use a calculator to compute en for n = 50, n = 100 and n = 1000.

13. Use a computer to compute en for n = 1, 000.


182 Chapter 10. Monotone Sequences

1
14. Let x1 = 8 and xn+1 = 2 xn + 2 for n ∈ N. Show that (xn ) is
bounded and monotone. Find the limit.

15. Let x1 ≥ 2 and xn+1 = 1 + xn − 1 forn ∈ N. Show that (xn ) is
decreasing and bounded below by 2. Find the limit.
√ √
16. Let x1 = p, where p > 0, and and xn+1 = p + xn for n ∈ N.
Show that (xn ) converges. Find the limit also.

17. Use the method in Example 5 to



i) calculate 2, correct to within 4 decimals.

ii) calculate 5, correct to within 5 decimals.

Answers

1. The limit is 1.

2. The limit is 2.

4. (xn ) is increasing.

5. (sn ) is decreasing and (tn ) is increasing. Also tn ≤ xn ≤ sn for n ∈ N.


1 1 1 1 1
8. Note yn = (n+1) + (n+2) + ··· + 2n < (n+1) + (n+1) +

1 n
··· + = < 1.
(n + 1) (n + 1)

1
10. (a) e (b) e2 (c) e (d) e

11.e2 = 2.25, e4 = 2.441406, e8 = 2.565785, e16 = 2.637928.

12.e50 = 2.691588, e100 = 2.704814, e1000 = 2.716924

14. (xn ) is a bounded decreasing sequence. The limit is 4.


10.1 Monotone Sequences 183

15. The limit is 2.



16. (yn ) is increasing. The limit is y = 21 (1 + 1 + 4p).

17. (i) Note that if n ≥ 2, then 0 ≤ sn − 2 ≤ s2n − 2.
√ (s2 −5) (s2 −5)
(ii) Note that 0 ≤ sn − 5 ≤ n√5 ≤ n2
Chapter 11
Subsequences and the
Bolzano-Weierstrass Theorem

In this chapter we shall introduce the notion of a subsequence of a se-


quence of real numbers. This notion is somewhat more general than that
of a tail of a sequence and is often useful in establishing the divergence
of a sequence. We shall also prove the important Bolzano-Weierstrass
Theorem.

11.1 Subsequences

Definition Let X = (xn ) be a sequence of real numbers and let n1 <


n2 < · · · < nk < · · · be a strictly increasing sequence of natural

184
11.1 Subsequences 185

numbers. Then the sequence X 0 = (xnk ) in R given by

(xn 1 , xn2 , xn3 , . . . , xnk , . . .)

is called a subsequence of X.
1 1 1

For example, the following are subsequences of X = 1, 2, 3, ··· :
 
1 1 1 1
, , , ··· , , ··· ,
3 4 5 n+2
 
1 1 1 1
, , , ··· , , ··· ,
1 3 5 2n − 1
 
1 1 1
, , ··· , , ··· .
2! 4! (2n)!
1

The following sequences are not subsequences of X := n :
 
1 1 1 1 1 1
, , , , , , ··· ,
2 1 4 3 6 5
 
1 1 1
, 0, , 0, , 0, · · · .
1 3 5
Of course, any tail of a sequence is a subsequence; in fact, the m-tail
corresponds to the sequence of indices:

n1 = m + 1, n2 = m + 2, . . . , nk = m + k, . . .

However, not every subsequence of a given sequence need be a tail of


the sequence.

Subsequences of convergent sequences also converge to the same limit,


as we now show.
186 Chapter 11. Subsequences and the Bolzano-Weierstrass Theorem

Theorem 1 If a sequence X = (xn ) of real numbers converges to a real


numberx, then any subsequence of X also converges to x.

Proof Let ε > 0 be given and let K(ε) be such that if n ≥ K(ε),
then |xn − x| < ε. Since n1 < n2 < · · · < nk < · · · is an
increasing sequence of natural numbers, it is easily proved (by induction)
thatnk ≥ n. Hence, if k ≥ K(ε) we also have nk ≥ k ≥ K(ε) so
that|xnk − x| < ε. Therefore the subsequence (xnk ) also converges to
x, as claimed.

Example 1 lim(bn ) = 0 if 0 < b < 1.

Solution

Takexn = bn . Since0 < b < 1, then xn+1 = bn+1 < bn = xn so


that the sequence (xn ) is decreasing. It is also clear that 0 ≤ xn ≤ 1, so
it follows from the Monotone Convergence Theorem that the sequence
is convergent. Letx = lim xn . Since (x2n ) is a subsequence of (xn ) it
follows from the previous Theorem thatx = lim(x2n ). On the other hand
it follows from the relation x2n = b2n = (bn )2 = (xn )2 that

x = lim(x2n ) = (lim(xn ))2 = x2 .

Hence x2 − x = 0. i.e., x(x − 1) = 0.

Therefore we must either have x = 0 orx = 1. Since the sequence


(xn ) is decreasing and bounded above byb < 1, we deduce thatx = 0.
1
Example 2 lim(c n ) = 1 for c > 1.

Solution
1
Since c > 1, note that if zn = c n , then zn > 1 and zn+1 < zn
for all n ∈ N. Thus by the Monotone Convergence Theorem, the limit
11.1 Subsequences 187

z = lim(zn ) exists. Hence by Theorem 1 it follows that z = lim(z2n ).


On the other hand it follows from the relation

1 1 1 1
z2n = c 2n = (c n ) 2 = zn2

that
1 1
z = lim(z2n ) = (lim(zn )) 2 = z 2 .

Therefore we have z 2 = z whence it follows that either z = 0 or z = 1.


Since zn > 1 for all n ∈ N, we deduce that z = 1.

The use of subsequences makes it easy to provide a test for the di-
vergence of a sequence.

Theorem 2 (Divergence Criterion) Let X = (xn ) be a sequence of


real numbers. Then the following statements are equivalent:

(i) The sequence X = (xn ) does not converge to x ∈ R.

(ii) There exists an ε0 > 0 such that for any k ∈ N, there exists nk ∈ N
such that nk ≥ k and |xnk − x| ≥ ε0 .

(iii) There exists an ε0 > 0 and a subsequence X 0 = (xnk ) of X such


that |xnk − x| ≥ ε0 for all k ∈ N.

Proof (i) ⇒ (ii) If X = (xn ) does not converge to x, then for some
ε0 > 0 it is impossible to find a natural number K(ε) such that for all
n ≥ K(ε), the terms xn satisfy |xn − x| < ε holds. That is, for each
k ∈ N it is not true that for all n ≥ k the inequality |xn − x| < ε0
holds. In other words, for every k ∈ N there exists a natural number
nk ≥ k such that |xnk − x| ≥ ε0 .

(ii) ⇒ (iii) Let ε0 be as in (ii) and let n1 ∈ N be such that n1 ≥ 1 and


|xn1 −x| ≥ ε0 . Now let n2 ∈ N be such that n2 > n1 and |xn2 −x| ≥ ε0 ;
188 Chapter 11. Subsequences and the Bolzano-Weierstrass Theorem

let n3 ∈ N be such that n3 > n2 and |xn3 − x| ≥ ε0 . Continue in this


way to obtain a subsequence X 0 = (xnk ) of X such that |xnk − x| ≥ ε0
for all k ∈ N.

(iii) ⇒ (i) Suppose X = (xn ) has a subsequence X 0 = (xnn ) satisfying


the condition in (iii). Then X cannot converge to x; for if it did, then,
by Theorem 1 the subsequence X 0 would also converge to x. But this is
impossible, since none of the terms of X 0 belongs to the ε0 -neighborhood
of x.

This completes the proof.

Since all subsequences of a convergent sequence must converge to the


same limit, we have Part (i) in the following result. Part (ii) follows
from the fact that a convergent sequence is bounded.

Divergence Criteria If a sequence X = (xn ) of real numbers has either


of the following properties, then X is divergent.

(i) X has two convergent subsequences X 0 = (xnk ) and X 00 = (xrk )


whose limits are not equal.

(ii) X is unbounded.

Example 3 The sequence ((−1)n ) is divergent.

Solution

If the sequence X = ((−1)n ) were convergent to a number x, then


every subsequence of X must converge tox. Since there is a subse-
quence X 0 = (1, 1, 1, · · · ) converging to +1 and another subsequence
X 00 = (−1, −1, −1, · · · ) converging to−1, by Divergence Criteria, we
conclude that X must be divergent.
1 1
Example 4 The sequence (1, 2, 3, 4, · · · ) is divergent.
11.1 Subsequences 189

Argument 1: The given is the sequence Y = (yn ), where


(
n, if n is odd
yn = 1
.
n, if n is even

It can easily be seen that the sequence is not bounded; hence, it cannot
be convergent.
1 1 1

Argument 2: we notice that although a subsequence 2, 4, 6, ··· of
Y converges to 0, the entire sequence Y does not converge to 0. Indeed
there is a subsequence (3, 5, 7, . . .) of Y that remains outside the 1-
neighborhood of 0; hence Y does not converge to 0.

Example 5 The sequence S = (sin n) is divergent.

Solution

We recall that sin π6 = 21 = sin 5π 1


 
6 and that sin x > 2 for x in the
intervalI1 = 6 , 6 . Since the length of I1 is 5π
π 5π π 2π

6 − 6 = 3 > 2, there
are at least two natural numbers lying inside I1 ; we let n1 be the first
1
such number. Similarly, for each k ∈ N, sin x > 2 for x in the interval
 
π 5π
Ik = + 2π(k − 1), + 2π(k − 1) .
6 6

Since the length of Ik is greater than 2, there are at least two natural
numbers lying inside Ik ; we let nk be the first one. The subsequence
S 0 = (sin nk ) of S obtained in this way has the property that all of its
values lie in the interval [ 12 , 1].

Similarly, if k ∈ N and Jk is the interval


 
7π 11π
Jk = + 2π(k − 1), + 2π(k − 1) ,
6 6
190 Chapter 11. Subsequences and the Bolzano-Weierstrass Theorem

then it is seen that sin x < − 12 for all x ∈ Jk and the length of Jk is
greater than 2. Let mk be the first natural number lying Jk . Then the
subsequence S 00 = (sin mk ) of S has the property that all of its values
lie in the interval [−1, − 12 ].

Given any real number c, it is readily seen that at least one of the
subsequences S 0 and S 00 lies entirely outside of the 12 -neighborhood of c.
Therefore c cannot be a limit of S. Since c ∈ R is arbitrary, we deduce
that S is divergent.

The Existence of Monotone Subsequences

While not every sequence is a monotone sequence, we will now show that
every sequence has a monotone subsequence.

Theorem 3 (Monotone Subsequence Theorem) If X = (xn ) is


a sequence of real numbers, then there is a subsequence of X that is
monotone.

Proof For the purpose of this proof we will say that the mth term xm
is a “peak” if xm ≥ xn for all n ∈ N with m ≤ n. (That is, xm is never
exceeded by any term that follows it in the sequence.)

We will consider two cases, depending on whether X has infinitely


many, or finitely many, peaks.

Case 1: X has infinitely many peaks. In this case, we order the peaks by
increasing subscripts. Thus we have the peaks xm1 , xm2 , . . . , xmk , . . . ,
where m1 < m2 < · · · < mk < · · · . Since each of the term is a peak, we
have
xm1 ≥ xm2 ≥ · · · ≥ xmk ≥ · · ·

Hence, the subsequence (xmk ) of peaks is a decreasing subsequence of


X.
11.1 Subsequences 191

Case 2: X has finite number (possibly zero) of peaks. Let these peaks
be xm1 , xm2 , . . . , xmr . Let s1 = mr + 1 (the first index beyond the last
peak). Since xs1 is not a peak, there exists s2 > s1 such that xs1 < xs2 .
Since xs2 is not a peak, there exists s3 > s2 such that xs2 < xs3 . If we
continue in this manner, we obtain an increasing subsequence (xsn ) of
X.

This completes the proof .

Remark It is not difficult to see that a given subsequence may have one
subsequence that is increasing, and another subsequence that is decreas-
ing.

The Bolzano-Weierstrass Theorem

We will now use the Monotone Subsequence Theorem to prove the Bolzano-
Weierstrass Theorem, which states that every bounded sequence has a
convergent subsequence. Because of the importance of this theorem we
will also give a second proof of it based on the Nested Interval Property.

Theorem 4 (The Bolzano-Weierstrass Theorem) A bounded se-


quence of real numbers has a convergent subsequence.

First Proof It follows from the Monotone Subsequence Theorem that if


X = (xn ) is a bounded sequence, then it has a subsequence X 0 = (xnk )
that is monotone. Since this subsequence is also bounded, it follows from
the Monotone Convergence Theorem that the subsequence is convergent.

Second Proof Since the set of values {xn : n ∈ N} is bounded; this set
is contained in an interval I1 = [a, b]. We take n1 = 1.

We now bisect I1 into two equal subintervals I10 and I100 , and divide
192 Chapter 11. Subsequences and the Bolzano-Weierstrass Theorem

the set of indices{n ∈ N : n > 1} into two parts:

A 1 = {n ∈ N : n > n1 , xn ∈ I10 }, B 1 = {n ∈ N : n > n1 , xn ∈ I100 }.

If A 1 is infinite, we take I2 = I10 and let n2 be the smallest natural


number in A 1 . (By the well-ordering property of N, every non empty
subset of Nhas a least element, hence A 1 must have a least element). If
A 1 is a finite set, then B1 must be infinite, and we take I2 = I100 and let
n2 be the smallest natural number inB1 .

We now bisect I2 into two equal subintervals I20 and I200 , and divide
the set {n ∈ N : n > n2 } into two parts:

A2 = {n ∈ N : n > n2 , xn ∈ I20 }, B2 = {n ∈ N : n > n2 , xn ∈ I200 }

If A2 is infinite, we take I3 = I20 and let n3 be the smallest natural


number in A2 . If A2 is a finite set, then B2 must be infinite, and we take
I3 = I200 and let n3 be the smallest natural number in B2 .

We continue in this way to obtain a sequence of nested intervals


I1 ⊇ I2 ⊇ . . . ⊇ Ik ⊇ . . . and a subsequence (xnk ) of X such that
xnk ∈ Ik for k ∈ N. Since the length of Ik is equal to 2b−a
k−1 , and since
 b−a
inf 2k−1 : k ∈ N = 0, it follows that there is a (unique) common point
ξ ∈ Ik for all k ∈ N. Moreover, since xnk and ξ both belong to Ik , we
have
b−a
|xnk − ξ| ≤ ,
2k−1
Whence it follows that the subsequence (xnk ) of X converges to ξ.

Remark The above Theorem is sometimes called the Bolzano-Weierstrass


Theorem for sequences, since there is another version of it that deals with
bounded sets in R.
11.1 Subsequences 193

It is readily seen that a bounded sequence can have various subse-


quences converging to different limits; for example, the sequence ((−1)n )
has subsequences that converge to −1, and other subsequences that con-
verge to +1. It also has subsequences that do not converge, for example,
the sequence itself is not convergent.

Let X be a sequence of real numbers and let X 0 be a subsequence of


X. Then X 0 is a sequence in its own right, and so it has subsequences.
We note that if X 00 is a subsequence of X 0 , then it is also a subsequence
of X.

Theorem 5 Let X = (xn ) be a bounded sequence of real numbers and


let x ∈ R have the property that every convergent subsequence of X
converges to x. Then the sequence X converges to x.

Proof Suppose M > 0 is a bound for the sequenceX = (xn ), so that


|xn | ≤ M for all n ∈ N. Suppose also that the sequence X does not
converge to x. It follows from Theorem 2 that there exist an ε0 > 0 and
a subsequence X 0 = (xnk ) of X such that

|xnk − x| ≥ ε0 for all k ∈ N. . . . (15.3)

Since X 0 is a subsequence of X, it follows that M is also a bound for X 0 .


Hence it follows from the Bolzano-Weierstrass Theorem that X 0 has a
convergent subsequence X 00 . As we have noted, X 00 is also a subsequence
of X; hence it converges to x by hypothesis. Consequently it ultimately
belongs to the ε0 -neighborhood ofx. Since every term of X 00 is also a
term of X 0 , this gives us a contradiction to (15.3).

Exercises

1. Give an example of an unbounded sequence that has a convergent


subsequence.
194 Chapter 11. Subsequences and the Bolzano-Weierstrass Theorem

1
2. Show that if 0 < c < 1, then lim(c n ) = 1.

3. Show that the following sequences are divergent

a) 1 − (−1)n + n1 b) sin nπ
 
4

4. Let X = (xn ) and Y = (yn ) be given sequences, and let the


“shuffled” sequence Z = (zn ) be defined by z1 = x1 , z2 = y1 , . . . ,
z2n−1 = xn , z2n = yn , . . .. Show that Z is convergent if and only if X
and Y are convergent and lim X = lim Y.
1
5. Let xn = n n for n ∈ N.

(a) Show that the inequality xn+1 < xn is equivalent to the inequality
n
1 + n1 < n, and infer that the inequality is valid for n ≥ 3. Conclude
that (xn ) is ultimately decreasing and that x := lim(xn ) exists.

(b) Use the fact that the subsequence (x2n ) also converges to x to show

that x = x. Conclude that x = 1.

6. Suppose that every subsequence of X = (xn ) has a subsequence that


converges to 0. Show that lim X = 0.

7. Establish the convergence and find the limits of the following


sequences:
  n2  1 n
(a) 1 + n12
 
, (b) 1 + 2n ,
 2n2  n 
(c) 1 + n12 , (d) 1 + n2 .

8. Let (xn ) be a bounded sequence and for each n ∈ N let sn = sup{xk :


k ≥ n} and S := inf{sn }. Show that there exists a subsequence of (xn )
that converges to S.

9. Suppose that xn ≥ 0 for all n ∈ N and that lim((−1)n xn ) exists.


Show that (xn ) converges.
11.1 Subsequences 195

10. Show thatif (xn) is unbounded, then there exists a subsequence (xn )
1
such that lim xnk = 0.
(−1)n
11. If xn = n , find the subsequence of (xn ) that is constructed
in the second proof of the Bolzano-Weierstrass Theorem, when we take
I1 = [−1, 1].

12. Let (xn ) be a bounded sequence and s = sup{xn : n ∈ N}. Show


that if s ∈
/ {xn : n ∈ N}, then there is a subsequence of (xn ) that
converges to s.

13. Let (In ) be a nested sequence of closed bounded intervals. For each
n ∈ N, let xn ∈ In . Use the Bolzano-Weierstrass Theorem to give a
proof of the Nested Interval Theorem.
fn+1
14. Let (fn ) be the Fibonacci sequence and let xn = fn . Prove that

lim(xn ) = 12 (1 + 5).

15. Show that lim (3n)1/2n = 1.
  
1 3n
16. Show that lim 1 + 2n = e3/2 .

17. Give an example to show that Theorem 5 fails if the hypothesis that
X is a bounded sequence is droppoed.

Answers
1
1. For example x2n−1 = 2n − 1 and x2n = 2n
1
7. (a) e (b) e 2 (c) e2 (d) e2

10. Choose n1 ≥ 1 so that |xn1 | > 1, then choose n2 > n1 so that


|xn2 | > 2, and, in general, choose nk > nk−1 so that |xnk | > k.

11. (x2n−1 ) = −1, − 31 , − 15 , · · · .



196 Chapter 11. Subsequences and the Bolzano-Weierstrass Theorem

12. Choose n1 ≥ 1 so that xn1 ≥ s − 1, then choose n2 > n1 so that


xn2 > s − 21 , and, in general, choose nk > nk−1 so that xnk > s − k1 .
Chapter 12
The Cauchy Criterion

The Monotone Convergence Theorem is extraordinarily useful and im-


portant, but it has the significant drawback that it applies only to se-
quences that are monotone. It is important for us to have a condition
implying the convergence of a sequence of real numbers that does not
require the knowledge of the value of the limit, and is not restricted to
monotone sequences. The Cauchy Criterion, which will be established
in this chapter, is such a condition.

12.1 Cauchy sequence

Definition A sequence X = (xn ) of real numbers is said to be a Cauchy


sequence if for every ε > 0 there is a natural number H(ε) such that for
all natural numbers n, m ≥ H(ε), the terms xn , xm satisfy |xn − xm | <
ε.

197
198 Chapter 12. The Cauchy Criterion

It will be seen below that the Cauchy sequences are precisely the
convergent sequences. In proving this we first show that a convergent
sequence is a Cauchy sequence.

Lemma 1 If X = (xn ) is a convergent sequence of real numbers, then


X is a Cauchy sequence.

Proof If x = lim X, then, given ε > 0 there is a natural number K 2ε




such that if n ≥ K 2ε then |xn − x| < 2ε . Thus, if H(ε) = K 2ε and


 

if n, m ≥ H(ε), then (using the Triangle Inequality) we have

|xn − xm | = |(xn − x) + (x − xm )|
ε ε
≤ |xn − x| + |xm − x| < + = ε.
2 2

Since ε > 0 is arbitrary, it follows that (xn ) is a Cauchy sequence.

In order to establish that a Cauchy sequence is convergent, we shall


need the following result.

Lemma 2 A Cauchy sequence of real numbers is bounded.

Proof Let X = (xn ) be a Cauchy sequence and let ε = 1. If H = H(1)


and n ≥ H, then |xn − xH | ≤ 1. Hence, by the Triangle Inequality we
have
|xn | = |xn − xH + xH | ≤ |xn − xH | + |xH |

that is |xn | ≤ |xH | + 1 for n ≥ H. If we set

M = sup{|x1 |, |x2 |, . . . , |xH−1 |, |xH | + 1},

then it follows that |xn | ≤ M for all n ∈ N.

We now present the important Cauchy Convergence Criterion.


12.1 Cauchy sequence 199

Theorem 1 (Cauchy Convergence Criterion) A sequence of real


numbers is convergent if and only if it is a Cauchy sequence.

Proof We have seen, in Lemma 1, that a convergent sequence is a


Cauchy sequence.

Conversely, let X = (xn ) be a Cauchy sequence; we shall show that


X is convergent to some real number. First we observe from Lemma
2 that the Cauchy sequence X is bounded. Therefore, by the Bolzano-
Weierstrass Theorem, there is a subsequence X 0 = (xnk ) of X that con-
verges to some real number x∗ . We shall complete the proof by showing
that X converges to x∗ .

Since X = (xn ) is Cauchy sequence, given ε > 0 there is a natural


number H 2ε such that if n, m ≥ H 2ε then
 

ε
|xn − xm | < 2. . . . (15.3)

Since the subsequence X 0 = (xnk ) converges to x∗, there is a natural


number K ≥ H 2ε belonging to the set {n1 , n2 , . . .} such that


ε
|xK − x ∗ | < .
2
ε

Since K ≥ H 2 , it follows from (15.3) with m = K that

|xn − xK | < 2ε for n ≥ H 2ε .




Therefore, if n ≥ H 2ε , we have


|xn − x ∗ | = |(xn − xK ) + (xK − x∗)|

≤ |xn − xK | + |xK − x ∗ |
ε ε
< + = ε.
2 2
200 Chapter 12. The Cauchy Criterion

Since ε > 0 is arbitrary, we infer thatlim(xn ) = x∗. Therefore the


sequence X is convergent.

We shall now give some examples of applications of the Cauchy Cri-


terion.
1

Example 1 Show that the sequence n is Cauchy and hence is conver-
gent.

Solution
1
Take (xn ) = n.

1

To show directly that n is a Cauchy sequence we note that if ε > 0
2
is given, then there is a natural number H = H(ε) such that H > ε.
1 1 ε 1 1 ε
Hence, if n, m ≥ H, then we have n ≤ H < 2 and m ≤ H < 2.
Therefore it follows that if n, m ≥ H, then

1 1 1 1 2
|xn − xm | = −
≤ + ≤ < ε.
n m n m H

1

Since ε > 0 is arbitrary, we infer that n is a Cauchy sequence;
therefore, it follows from the Cauchy Convergence Criterion that it is a
convergent sequence.

Example 2 Verify the convergence of the sequence X = (xn ) defined


by

x1 = 1, x2 = 2 and xn = 21 (xn−2 + xn−1 ) and n > 2.

Also find the limit of the sequence.

Solution

It can be shown by mathematical induction that 1 ≤ xn ≤ 2 for all


n ∈ N. Some calculation shows that the sequence X is not monotone.
12.1 Cauchy sequence 201

However, since the terms are formed by averaging, it is readily seen that
1
|xn − xn+1 | = 2n−1 for n ∈ N.

(This can be proved by induction.) Thus, if m > n, we may employ the


Triangle Inequality to obtain

|xn − xm | ≤ |xn − xn+1 | + |xn+1 − xn+2 | + · · · + |xm−1 − xm |

1 1 1
= + + · · · + m−2
2n−12 n 2
 
1 1 1 1
= 1+ + · · · + m−n−1 < n−2 .
2n−1 2 2 2
1 ε
Therefore, given ε > 0, if n is chosen so large that 2n < 4 and if m ≥ n,
then it follows that |xn − xm | < ε. Therefore, X is a Cauchy sequence
in R. By the Cauchy Convergence Criterion (Theorem 1) we infer that
the sequence X converges to a number x.

To evaluate the limitx, we might first “pass to the limit” in the rule
of definition xn = 12 (xn−1 + xn−2 ) to conclude that x must satisfy the
relation x = 21 (x + x), which is true, but not informative to get the value
of x. Hence we must try something else.

Since X converges to x, so does the subsequence X 0 with odd indices.


By induction, it can be established that

1 1 1
x2n+1 = 1 + + + · · · + 2n−1
2 23 2
 
2 1
=1 + 1− n .
3 4
It follows from this that x = lim X = lim X 0 = 1 + 2
3 = 53 .

When Is a Sequence Not Cauchy


202 Chapter 12. The Cauchy Criterion

If a sequence is not Cauchy, then it can be verified using the negation of


the definition of Cauchy sequence: That is, a sequence is not Cauchy
if there exists ε0 > 0 such that for every natural number H there exist
at least one n > H and at least one m > H such that |xn − xm | ≥ ε0 .

Example 3 The sequence (1 + (−1)n ) is not a Cauchy sequence.

We have to find an ε0 > 0 such that for every natural number H


there exist at least one n > H and at least one m > H such that
|xn − xm | ≥ ε0 .

For the terms xn = 1 + (−1)n , we observe that if n is even, then


xn = 2 and xn+1 = 0. If we take ε0 = 2, then for any H we can choose
an even number n > H and let m = n + 1 to get

|xn − xn+1 | = 2 = ε0 .

We conclude that (xn ) is not a Cauchy sequence.

Remark We emphasize that to prove a sequence (xn ) is a Cauchy se-


quence, we may not assume a relationship between m andn, since the
required inequality |xn − xm | < ε must hold for all n, m ≥ H(ε). But
to prove a sequence is not a Cauchy sequence, we may specify a relation
between n and m as long as arbitratily large values of n and m can be
chosen so that |xn − xm | ≥ ε0 .

Example 4 Let Y = (yn ) be the sequence of real numbers given by

1 1 1 1 1 (−1)n+1
y1 = , y2 = − , . . . , yn = − + ··· + , ···
1! 1! 2! 1! 2! n!
12.1 Cauchy sequence 203

Clearly, Y is not a monotone sequence However, if m > n, then

(−1)n+2 (−1)n+3 (−1)m+1


ym − yn = + + ··· + .
(n + 1)! (n + 2)! m!

Since 2r−1 ≤ r! , it follows that if m > n, then (applying Triangle


inequality)

1 1 1
|ym − yn | ≤ + + ··· +
(n + 1)! (n + 2)! m!

1 1 1 1
≤ n
+ n+1 + · · · + m−1 < n−1 .
2 2 2 2
Therefore, it follows that (yn ) is a Cauchy sequence. Hence it converges
to a limity. At the present moment we cannot evaluate y directly; how-
ever, passing to the limit (with respect tom) in the above inequality, we
obtain
1
|yn − y| ≤ .
2n−1
Hence we can calculate y to any desired accuracy by calculating the
terms yn for sufficiently largen. By doing so it can be shown that y is
approximately equal to 0.632120559. (The exact value of y is 1 − 1e .)

Example 5 The sequence 11 + 12 + · · · + n1 diverges.




Let H = (hn ) be the sequence defined by


1 1 1
hn = 1 + 2 + ··· + n for n ∈ N,

If m > n, then
1 1
hm − h n = + ··· + .
n+1 m
1 m−n n
Since each of these m−n terms exceeds m, then hm −hn > m = 1− m .
In particular, if m = 2n we have h2n − hn > 12 . This shows that H is
not a Cauchy sequence; therefore H is not a convergent sequence.
204 Chapter 12. The Cauchy Criterion

Definition We say that a sequence X = (xn ) of real numbers is con-


tractive if there exists a constant C, 0 < C < 1, such that

|xn+2 − xn+1 | ≤ C|xn+1 − xn |

for all n ∈ N. The number C is called the constant of the contractive


sequence.

Theorem 2 Every contractive sequence is a Cauchy sequence, and there-


fore is convergent.

Proof If we successively apply the defining condition for a contractive


sequence, we can work our way back to the beginning of the sequence as
follows:

|xn+2 − xn+1 | ≤ C | xn+1 − xn | ≤ C 2 |xn − xn−1 |

≤ C 3 |xn−1 − xn−2 |

≤ · · · ≤ C n |x2 − x1 |.

For m > n, we estimate | xm −xn | by first applying the Triangle Inequal-


ity and then using the formula for the sum of a geometric progression.
This gives

|xm − xn | ≤ |xm − xm−1 | + |xm−1 − xm−2 | + · · · + |xn+1 − xn |

≤ (C m−2 + C m−3 + · · · + C n−1 ) |x2 − x1 |

= C n−1 (C m−n−1 + C m−n−2 + · · · + 1) |x2 − x1 |


1 − C m−n
 
n−1
= C |x2 − x1 |
1−C
12.1 Cauchy sequence 205
 
1
≤ C n−1 |x2 − x1 |.
1−C
Since0 < C < 1, we knowlim(C n ) = 0. Therefore, we infer that (xn ) is a
Cauchy sequence. It now follows from the Cauchy Convergence Criterion
that (xn ) is a convergent sequence.

In the process of calculating the limit of a contractive sequence, it is


often very important to have an estimate of the error at the nth stage. In
the next result we give two such estimates: the first one involves the first
two terms in the sequence and n; the second one involves the difference
xn − xn−1 .

Corollary If X = (xn ) is a contractive sequence with constant C, 0 <


C < 1, and if x∗ = lim X, then:
C n−1
(i) |x ∗ −xn | ≤ 1−C | x2 − x1 |,
C
(ii) |x ∗ −xn | ≤ 1−C | xn − xn−1 |.

Proof We have seen in the preceding proof that if m > n, then

C n−1
|xm − xn | ≤ |x2 − x1 |.
1−C

If we let m → ∞ in this inequality (with n fixed), we obtain (i).

To prove (ii), recall that if m > n, then

|xm − xn | ≤ |xm − xm−1 | + · · · + |xn+1 − xn |.

Since it is readily established, using induction, that

|xn+k − xn+k−1 | ≤ C k |xn − xn−1 |


206 Chapter 12. The Cauchy Criterion

we infer that

|xm − xn | ≤ (C m−n + · · · + C 2 + C) |xn − xn−1 |

C
≤ |xn − xn−1 |.
1−C
We now let m → ∞ in this inequality to obtain assertion (ii).

Example 6 Approximate the solution of the cubic equation x3 −7x+2 =


0 that lies between 0 and 1.

Solution

This can be accomplished by means of an iteration procedure as


x3 +2
follows. We first rewrite the equation as x = 7 and use this to define
a sequence. We assign x1 an arbitrary value between 0 and 1, and then
define
xn+1 = 71 (x3n + 2), n ∈ N.

Because0 < x1 < 1, it follows that 0 < xn < 1 for all n ∈ N.

Moreover, we have

|xn+2 − xn+1 | = | 17 (x3n+1 + 2) − 1 3


7 (xn + 2)|
1 3
= 7 |xn+1 − x3n |
1 3
= 7 |xn+1 + xn+1 xn + x2n | |xn+1 − xn |
3
≤ 7 |xn+1 − xn |

Therefore, (xn ) is a contractive sequence and hence there exists r such


that lim(xn ) = r. If we pass to the limit on both sides of the equality
x3n +2 r 3 +2
xn+1 = 7 , we obtain r = 7 and hence r3 − 7r + 2 = 0. Thus r is
a solution of the equation.
12.1 Cauchy sequence 207

We can approximate r by choosing x1 and calculating x2 , x3 . . . suc-


cessively. For example, if we takex1 = 0.5, we obtain (to nine decimal
places) :

x2 = 0.303571429, x3 = 0.289710830, x4 = 0.289188016,

x5 = 0.289169244, x6 = 0.289168571, etc.

To estimate the accuracy, we note that |x2 − x1 | < 0.2. Thus, after n
3n−1
steps it follows that we are sure that |x ∗ −xn | ≤ 7n−2 ·20 . Thus, when
n = 6, we are sure that

35 243
|x ∗ −x6 | ≤ = < 0.0051.
74 · 20 48020

Actually the approximation is substantially better than this. In fact,


since |x6 − x5 | < 0.000 0005, it follows from assertion (ii) of the above
3
Corollary that |x ∗ −x6 | ≤ 4 |x6 − x5 | < 0.000 0004. Hence the first
five decimal places of x6 are correct.

Exercises

1. Give an example of a bounded sequence that is not a Cauchy sequence.

2. Show directly from the definition that the following are Cauchy se-
quences.
n+1 1 1
 
(a) n (b) 1 + 2! + ··· + n!

3. Show directly from the definition that the following are not Cauchy
sequences.
 
(−1)n
(a) ((−1)n ) (b) n + n , (c) (ln n)

4. Show directly that if (xn ) and (yn ) are Cauchy sequences, then
208 Chapter 12. The Cauchy Criterion

(xn + yn ) and (xn yn ) are Cauchy sequences.

5. Let (xn ) be a Cauchy sequence such that xn is an integer for all


n ∈ N. Show that (xn ) is ultimately constant.

6. Show directly that a bounded, monotone increasing sequence is a


Cauchy sequence.

7. If x1 < x2 are arbitrary real numbers and xn = 12 (xn−2 + xn−1 ) for


n > 2, show that (xn ) is convergent. What is its limit?
1
8. If y1 < y2 are arbitrary real numbers and yn = 3 yn−1 + 23 yn−2 for
n > 2, show that (yn ) is convergent. What is its limit?

9. If 0 < r < 1 and |xn+1 − xn | < rn for all n ∈ N, show that (xn ) is a
Cauchy sequence.
1
10. If x1 > 0 and xn+1 = 2+xn for n ≥ 1, show that (xn ) is a contractive
sequence. Find the limit.

11. The polynomial equation x3 −5x+1 = 0 has a root r with 0 < r < 1.
Use an appropriate contractive sequence to calculate r within 10−4 .

12. If xn = n, show that (xn ) satisfies |xn+1 − xn | = 0, but that it is
not a Cauchy sequence.

13. Let p be a given natural number. Give an example of a sequence (xn )


that is not a Cauchhy sequence, but that satisfies lim |xn+p − xn | = 0.

14. If x1 = 2 and xn+1 = 2+ x1n for n ≥ 1, show that (xn ) is a contractive


sequence. What is its limit?

Answers

1. For example, ((−1)n ).

3. (a) Note that |(−1)n − (−1)n+1 | = 2 for all n ∈ N.


12.1 Cauchy sequence 209

(c) Take m = 2n, so xm − xn = x2n − xn = ln 2n − ln n = ln 2 for all


n.

6. Let u = sup{xn : n ∈ N}. If ε > 0, let H be such that u−ε < xH ≤ u.


If m ≥ n ≥ H, then u − ε < xn ≤ xm ≤ u so that |xm − xn | < ε.

7. lim(xn ) = (1/3)x1 + (2/3)x2 .



10. The limit is 2 − 1.

11. Four iterations give r = 0.20164 to 5 places.


√ √ √ √ √
12. lim( n + 1 − n ) = 0. But, if m = 4n, then 4n − n = n for
all n.

14. The limit is 1 + 2.

Objective Type Questions

1. Pick the False statement.

(a) There are bounded sequences that are not Cauchy.

(b) ((−1)n ) is not a Cauchy sequence

(c) If (xn ) is a Cauchy sequence such that xn is an integer for all


n ∈ N, then (xn ) is ultimately constant.

(d) If (xn ) is a Cauchy sequence such that xn is an integer for all


n ∈ N, then (xn ) is a constant sequence.

2. Pick the False statement.

(a) A bounded, monotone increasing sequence is a Cauchy sequence.

(b) If 0 < r < 1 and |xn+1 − xn | < rn for all n ∈ N, then (xn ) is a
Cauchy sequence.

(c) If x1 < x2 are arbitrary real numbers and xn = 21 (xn−2 + xn−1 )


210 Chapter 12. The Cauchy Criterion

for n > 2, then (xn ) is not bounded.

(d) If x1 < x2 are arbitrary real numbers and xn = 12 (xn−2 + xn−1 ) for
n > 2, then (xn ) is convergent.

3. Pick the true statement.

(a) If (xn ) is an unbounded sequence, then there exists a properly


divergent subsequence.
1
(b) If x1 > 0 and xn+1 = 2+xn for n ≥ 1, then (xn ) is not a contrac-
tive sequence.

(c) If x1 = 2 and xn+1 = 2 + x1n for n ≥ 1, then (xn )is not a contrac-
tive sequence.

(d) none of the above.

Answers to Objective Type Questions

1. (d)

2. (c)

3. (a)
Chapter 13
Properly Divergent Sequences

13.1 Properly Divergent Sequences

For certain purposes it is convenient to define what is meant for a se-


quence (xn ) of real numbers to “tend to ±∞”.

Definition 1 Let (xn ) be a sequence of real numbers.

(i) We say the (xn ) tends to +∞, and write lim(xn ) = +∞, if for every
α ∈ R there exists a natural number K(α) such that if n ≥ K(α), then
xn > α.

(ii) We say that (xn ) tends to −∞, and write lim(xn ) = −∞, if for
every β ∈ R there exists a natural number K(β) such that if n ≥ K(β),
then xn < β.

We say that (xn ) is properly divergent in case we have either


lim(xn ) = +∞ or lim(xn ) = −∞.

211
212 Chapter 13. Properly Divergent Sequences

Attention! The symbols +∞ and −∞ in the above expressions do not


represent real numbers. Results that have been proved in earlier chapters
for conventional limits lim(xn ) = L (for L ∈ R) may not remain true
when lim(xn ) = ±∞.

Definition 2 A sequence (xn ) of real numbers which is neither conver-


gent nor properly divergent is called an oscillatory sequence.

Example 1 Show the sequence (n) is properly divergent.

Solution

If α ∈ R is given, let K(α) be any natural number such thatK(α) >


α. Then forn > K(α), we have xn > α. Hence lim(n) = +∞. Hence
the sequence (n)is properly divergent.

Example 2 The sequence (1, 2, 1, 2, . . . ) is an oscillatory sequence,


since the sequence (1, 2, 1, 2, . . . ) is not convergent and since (1, 2, 1, 2, . . . )
neither diverges to +∞ nor diverges to −∞.

Example 3 Show the sequence (n2 ) diverges to +∞.

Solution

If α ∈ R is given, let K(α) be any natural number such that K(α) >
α. Then for any n ≥ K(α), we have xn = n2 ≥ n > α. Hence lim(n2 ) =
+∞ and the sequence (n2 ) diverges to +∞.

Example 4 Show the sequence (cn ) is properly divergent, where c > 1.

Solution

Letc = 1 + b, where b > 0. If α ∈ R is given, let K(α) be a natural


13.1 Properly Divergent Sequences 213

α
number such that K(α) > b. If n ≥ K(α) , we have

cn = (1 + b)n
≥ 1 + nb , by Bernoulli0 s Inequality
> 1+α
> α.

Therefore lim(cn ) = + ∞ and the sequence (cn ) is properly divergent.

Monotone sequences are particularly simple in regard to their con-


vergence. We have seen in the Monotone Convergence Theorem that a
monotone sequence is convergent if and only if it is bounded. The next
result is a reformulation of the result.

Theorem 1 A monotone sequence of real numbers is properly divergent


if and only if it is unbounded.

a) If (xn ) is an unbounded increasing sequence, then lim(xn ) = +∞.

b) If (xn ) is an unbounded decreasing sequence, then lim(xn ) = −∞ .

Proof (a) Suppose that (xn ) is an increasing sequence. We know that


if (xn ) is bounded, then it is convergent. If (xn ) is unbounded, then for
any α ∈ R there exists n(α) ∈ N such that α < x n(α) . But since (xn )
is increasing, we have α < xn for all n ≥ n(α). Since α is arbitrary, it
follows that lim (xn ) = +∞.

Part (b) is proved in a similar fashion.

The following “comparison theorem” is frequently used in showing that


a sequence is properly divergent.

Theorem 2 (Comparison Theorem) Let (xn ) and (yn )be two se-
quences of real numbers and suppose that
214 Chapter 13. Properly Divergent Sequences

xn ≤ yn for alln ∈ N. . . . (15.3)

a) If lim (xn ) = +∞, then lim (yn ) = +∞

b) If lim (yn ) = −∞, then lim (xn ) = −∞.

Proof (a) If lim (xn ) = +∞ and if α ∈ R is given, then there exists a


natural number K(α) such that if n ≥ K(α), then α < xn . In view of
(15.3), it follows that α < yn for all n ≥ K(α). Since α is arbitrary, it
follows that lim(yn ) = +∞.

The proof of (b) is similar.

Remarks

1. Theorem 2 remains true if condition (15.3) is ultimately true; that


is, if there exists m ∈ N such that xn ≤ yn for all n ≥ m.

2. If condition (15.3) of the Theorem 2 holds and if lim(yn ) = +∞,


it does not follow that lim(xn ) = +∞. Similarly, if (15.3) holds
and if lim(xn ) = −∞, it does not follow that lim(yn ) = −∞. In
using Theorem 2 to show that a sequence tends to +∞ [respec-
tively, −∞] we need to show that the terms of the sequence are
ultimately greater [respectively, less] than or equals to the corre-
sponding terms of a sequence that is known to tend to +∞ [re-
spectively, −∞].

Since it is sometimes difficult to establish an inequality such as (15.3),


the following “limit comparison theorem” is often more convenient to use.

Theorem 3 (Limit Comparison Theorem) Let (xn ) and (yn ) be two


sequences of positive real numbers and suppose that there exists L ∈ R,
L > 0 such that
lim(xn /yn ) = L. (13.1)
13.1 Properly Divergent Sequences 215

Then lim(xn ) = +∞ if and only if lim(yn ) = +∞

Proof Suppose
lim(xn /yn ) = L.
L
Then corresponding to ε = 2 > 0 there exists a natural number K such
that

xn
yn − L < 12 L for all n ≥ K.

xn
i.e., − 12 L < yn − L < 12 L for all n ≥ K.

xn
i.e., 12 L < yn < 32 L for all n ≥ K.

Since each yn is positive, the above implies


   
1 3
L yn < xn < L yn for all n ≥ K. (13.2)
2 2

If lim (xn ) = +∞ and if α ∈ R is given, then there exists a natural


number K(α)such that if n ≥ K(α), thenα < xn . Hence by the second
inequality in (13.2), it follows that α < 23 L yn for all n ≥ K(α).

2
i.e., 3L α < yn for all n ≥ K(α). Since α is arbitrary, it follows that
lim(yn ) = +∞.

If lim(yn ) = +∞ and if α ∈ R is given, then there exists a natural


number K(α)such that if n ≥ K(α), thenα < yn . Hence by the first
inequality in (13.2), it follows that 12 L α < xn for alln ≥ K(α). Since α
is arbitrary, it follows thatlim (xn ) = +∞. This completes the proof.

Remark The conclusion in Theorem 7 need not hold if either L = +∞


orL = 0. For example, if (xn ) = (n) and (yn ) is the constant sequence
(15.3), then lim(xn /yn ) = ∞, lim(xn ) = +∞ but lim(yn ) = 1. Also, if
take (xn ) = (1) and (yn ) = (n), then, lim(xn /yn ) = 0 and lim(yn ) = +∞
but lim(xn ) = 1.
216 Chapter 13. Properly Divergent Sequences

Exercises

1. Show that if (xn ) is an unbounded sequence, then there exists a


properly divergent subsequence.

2. Give examples of properly divergent sequences (xn ) and (yn ) with


yn 6= 0 for all n ∈ N such that:

a) (xn /yn ) is convergent b) (xn /yn ) is properly divergent

3. Show that if xn > 0 for all n ∈ N then lim(xn ) = 0 if and only if


lim(1/xn ) = +∞

4. Establish the proper divergence of the following sequences.


√ √ 
a) ( n) b) n+1
√  √ 
c) n − 1 d) n/ n + 1

5. Is the sequence (n sin n) properly divergent?

6. Let (xn ) be properly divergent and let (yn ) be such that lim(xn yn )
belongs to R. Show that (yn ) converges to 0.

7. Let (xn ) and (yn ) be sequences of positive numbers such that lim(xn /yn ) =
0.

a) Show that if lim(xn ) = +∞, then lim(yn ) = +∞

b) Show that if (yn ) is bounded, then lim(xn ) = 0.

8. Investigate the convergence or the divergence of the following


sequences:
√  √ 
n
a) n2 + 2 b) n2 +1
√  √
n 2
c) √ +1 d) (sin n)
n

9. Let (xn ) and (yn ) be sequences of positive numbers such that lim(xn /yn ) =
13.1 Properly Divergent Sequences 217

+∞,

a) Show that if lim(yn ) = +∞, then lim(xn ) = +∞

b) Show that if (xn ) is bounded, then lim(yn ) = 0.

10. Show that if lim(an /n) = L where L > 0, then lim(an ) = +∞.

Answers

1. If {xn : n ∈ N} is not bounded above, choose nk+1 > nk such that


xnk ≥ k for k ∈ N.

2. Note that |xn − 0| < ε if and only if x1n > 1ε .


√ √ p
4. (a) [ n > a] ⇔ [n > a2 ] (c) n − 1 ≥ n/2 when n ≥ 2.
1
8. (a) n < (n2 + 2) 2
1 1 1 1
(c) Since n < (n2 + 1) 2 , then n 2 < (n2 + 1) 2 /n 2 .
xn
9. (a) Since yn → ∞, there exists K1 such that if n ≥ K1 , then xn ≥ yn .
Now apply Theorem.
Chapter 14
Open and Closed Sets in R

There are special types of sets that play a distinguished role in analysis
– these are the open and the closed sets in R. We start with recalling
the notion of a neighborhood of a point.

14.1 Neighborhood

Definition 1 A neighborhood of a point x ∈ R is any set V that


contains an ε-neighborhood Vε (x) = (x − ε, x + ε) of x for some ε > 0.

While an ε-neighborhood of a point is required to be “symmetric


about the point”, the idea of a (general) neighborhood relaxes this par-
ticular feature, but often serves the same purpose.

Definition 2

(i) A subset G of R is open in R if for each x ∈ G there exists a

218
14.1 Neighborhood 219

neighborhood V of x such that V ⊆ G.

(ii) A subset F of R is closed in R if the complement C(F ) = R\F is


open in R.

Method of showing G is open

To show that a set G ⊆ R is open, it suffices to show that each point


in G has an ε-neighborhood contained in G. In fact, G is open if and
only if for each x ∈ G, there exists εx > 0 such that (x − εx , x + εx ) is
contained in G.

Method of showing F is closed

To show that a set F ⊆ R is closed, it suffices to show that each point


y∈
/ F has an ε-neighborhood disjoint from F. In fact, F is closed if and
only if for each y ∈
/ F there exists εy > 0 such that

F ∩ (y − εy , y + εy ) = ∅.

Example 1 Show that the entire set R = (−∞, ∞) is open.

Solution

For any x ∈ R, we may take ε = 1. Then (x − 1, x + 1) is contained


in R, because y ∈ (x − 1, x + 1) implies y ∈ R . Hence, by the method
of showing openness, R is open.

Example 2 Show that the set G = {x ∈ R : 0 < x < 1} is open.

Solution

For an x ∈ G we may take εx to be the smaller of the numbers x,


1 − x. i.e., let
εx = min {x, 1 − x}.
220 Chapter 14. Open and Closed Sets in R

We show that (x − εx , x + εx ) ⊂ G. For this it is enough to show that


u ∈ (x − εx , x + εx ) implies u ∈ G.

Case 1. εx = x. Then x < 1 − x or 2x < 1. (Fig. 14.1)

Then u ∈ (x − εx , x + εx )

implies u ∈ (x − x, x + x)

implies u ∈ (0, 1) as 2x < 1.

Hence u ∈ G.

Figure 14.1:

Figure 14.2:

Case 2 εx = 1 − x

Then 1 − x < x or 0 < 2x − 1. (Ref. Fig. 14.2).

Then u ∈ (x − εx , x + εx )

implies u ∈ (2x − 1, 1)

implies u ∈ (0, 1)

implies u ∈ G.

Hence G is open.
14.1 Neighborhood 221

Example 3 Show that any open interval J = (a, b) is an open set.

Solution

If x ∈ I, we let εx = min{x − a, b − x}. Proceeding as in Example


2, it can be shown that (x − εx , x + εx ) ⊆ I.

Remark Proceeding as in Example 2 or 3, the intervals (−∞, b) and


(a, ∞) are open sets.

Example 4 The set I = [0, 1] is not open.

Solution

This follows since every ε-neighborhood of 0 ∈ I contains points not


in I (Fig. 14.3).

Figure 14.3:

Method of Vacuous Proof

Suppose that the hypothesis p of an implication p → q is false. Then


the implication p → q is true, because the statement has the form F →
T or F → F, and hence is true. Consequently, if it can be shown that p
is false, then a proof, called a vacuous proof, of the implication p → q
can be given.

Example 5 The empty set ∅ is open in R.

Solution

We give the vacuous proof here. There is no point in ∅ and hence vac-
uously every ε-neighborhood is contained in ∅. That is, the requirement
222 Chapter 14. Open and Closed Sets in R

in Definition 2 (i) is vacuously satisfied. Thus ∅ is open in R.

Example 6 The set I = [0, 1] is closed.

To see this let y ∈


/ I; then either y < 0 or y > 1. If y < 0, we take
εy = |y|, and if y > 1 we take εy = y − 1. Then it can be shown that,
in either case we have I ∩ (y − εy , y + εy ) = ∅.

Remark [0, 1] is called the closed unit interval.

Example 7 The set H = {x : 0 ≤ x < 1} is neither open nor closed.


It is not open as every neighborhood of 0 ∈ H contains points not in I.
It is not closed as every neighborhood of 1 ∈
/ H intersects with H.

Example 8 The empty set ∅ is closed in R.

Solution The empty set is closed since its complement R is open, by


Example 1.

Example 9 R = (−∞, ∞) is closed, since its complement ∅ is open.

Attention! In ordinary situations when applied to doors, and minds,


the words open and closed are antonyms. However, when applied to
subsets of R, these words are not antonyms. For example, we noted
above that the sets ∅, R are both open and closed in R. At this moment
we note that there are no other subsets of R that have both properties. In
addition, there are many subsets of R that are neither open nor closed;
in fact, most subsets of R have this neutral character.

The following basic result describes the manner in which open sets
relate to the operations of the union and intersection of sets in R.

Theorem 1 (Open Set Properties)

(a) The union of an arbitrary collection of open subsets in R is open.


14.1 Neighborhood 223

(b) The intersection of any finite collection of open sets in R is open.

Proof (a) Let {Gλ : λ ∈ Λ} be a family of sets in R that are open, and
let G be their union. i.e.,
[
G= Gλ .
λ∈Λ

Consider an element x ∈ G; by the definition of union, x must belong


to Gλ0 for some λ0 ∈ Λ. Since Gλ0 is open, there exists a neighborhood
V of x such that V ⊆ Gλ0 . But Gλ0 ⊆ G, so that V ⊆ G. Since x is an
arbitrary element of G, we conclude that G is open in R.

(b) Suppose G1 and G2 are open and let G = G1 ∩ G2 . To show that G


is open, we consider any x ∈ G; then x ∈ G1 and x ∈ G2 . Since G1 is
open, there exists ε1 > 0 such that (x − ε1 , x + ε1 ) is contained in G1 .
Similarly, since G2 is open, there exists ε2 > 0 such that (x − ε2 , x + ε2 )
is contained in G2 . If we now take ε to be the smaller of ε1 andε2 ,
then the ε-neighborhood U = (x − ε, x + ε) satisfies both U ⊆ G1
andU ⊆ G2 . Thus, U ⊆ G1 ∩ G2 and hence x ∈ U ⊆ G. Since x is an
arbitrary element of G, we conclude that G is open in R.

It now follows by an Induction argument that the intersection of any


finite collection of open sets is open.

The corresponding properties for closed sets will be established by


using the general De Morgan identities for sets and their components:

Theorem 2 (Closed Set Properties)

(a) The intersection of an arbitrary collection of closed sets in R is


closed.

(b) The union of any finite collection of closed sets in R is closed.


224 Chapter 14. Open and Closed Sets in R

Proof (a) Let {Fλ : λ ∈ Λ} be a family of closed sets in R and let


T
F = Fλ . We show that F is closed by showing that its complement
λ∈Λ
C(F ) = R\F is open. For each λ ∈ Λ, Fλ is closed ⇒ its complement
S
C(Fλ ) is open. Hence, by Theorem 1, λ∈Λ C(Fλ ) is open. Now by
De-Morgan law,
\ [
R\( Fλ ) = (R\Fλ ).
λ∈Λ λ∈Λ
T S
i.e., C(F ) =C( λ∈Λ Fλ ) = λ∈Λ C(Fλ ).

Hence, C (F ) is open and consequently, F is closed.

(b) Suppose F1 , F2 , . . . , Fn are closed in R and let F = F1 ∪F2 ∪· · ·∪Fn .


By the De Morgan identity,
n
\
R\(F1 ∪ F2 ∪ · · · ∪ Fn ) = (R\Fi ).
i=1

i.e., C (F ) = C (F1 ) ∩ · · · ∩ C (Fn ).

Since each set C (Fi ) is open, it follows from Part (b) of Theorem 1 that
C (F ) is open. Hence F is closed.

The finiteness restrictions in the above two Theorems cannot be re-


moved. We illustrate these in the following examples.

Example 10 Give an example to show that the intersection of infinitely


many open sets in R need not be open.

Solution

Let Gn = (0, 1 + 1/n) for n ∈ N. Then Gn is open for each n ∈ N,


T∞
by Example 3. However, the intersection G = n=1 Gn is the interval
(0, 1] which is not open. Thus, the intersection of infinitely many open
sets in R need not be open.
14.1 Neighborhood 225

Example 11 Give example to show that the union of infinitely many


closed sets in R need not be closed.

Solution

Let Fn = [1/n, 1] for n ∈ N. Each Fn is closed, but the union



S
F = Fn is the set (0, 1] which is not closed. Thus, the union of
n=1
infinitely many closed sets in R need not be closed.

Theorem 3 (Characterization of Closed Sets)

Let F ⊆ R; then the following assertions are equivalent.

(i) F is a closed subset of R.

(ii) If X = (xn ) is any convergent sequence of elements in F , then lim X


belongs to F .

Proof (i) ⇒ (ii) : Assume that F is a closed subset of R. Let X = (xn )


be a sequence of elements in F and let x = lim X; we wish to show that
x ∈ F. Suppose, on the contrary, that x ∈
/ F ; that is, that x ∈ C (F ),
the complement of F . Since C (F ) is open and x ∈ C (F ), it follows
that there exists an ε-neighborhood Vε of x such that Vε is contained in
C (F ). Since x = lim(xn ), it follows that there exists a natural number
K = K(ε) such that xK ∈ Vε . Therefore we must have xK ∈ C (F );but
this contradicts the assumption that xn ∈ F for all n ∈ N. Therefore,
we conclude that x ∈ F .

(ii) ⇒ (i) : Assume condition (ii). To prove (i), suppose, on the


contrary, that F is not closed, so that G = C (F ) is not open. Then
there exists a point y0 ∈ G such that for each n ∈ N, there is a number
yn ∈ C (G) = F such that |yn − y0 | < 1/n. It follows that y0 = lim(yn ),
and since yn ∈ F for all n ∈ N, the hypothesis (ii) implies that y0 ∈ F,
contrary to the assumption y0 ∈ G = C (F ). Thus the hypothesis that F
226 Chapter 14. Open and Closed Sets in R

is not closed implies that (ii) is not true. Consequently (ii) implies (i),
as asserted.

Definition A point x is a cluster point of a set F if every ε-neighborhood


of x contains a point of F different fromx. Each cluster point of a set F
is the limit of a sequence of points in F .

Example 12 Each point of the closed unit interval [0, 1] is a cluster


point the set; no other point in R is a cluster point of [0, 1].

Example 13 Each point of the closed unit interval [0, 1] is a cluster


point the set [0, 1] ∪ {2} ; no other point in R is a cluster point of
[0, 1] ∪ {2}.(Fig. 14.4)

Figure 14.4:

The next result is closely related to Theorem 3. It states that a set


F is closed if and only if it contains all of its cluster points. The result
follows immediately from Theorems 3 and 4 above. We provide a proof
that uses only the relevant definitions.

Theorem 5 A subset of R is closed if and only if it contains all of its


cluster points.

Proof Let F be a closed set in R and let x be a cluster point of F ;


we will show that x ∈ F . If not, then x belongs to the open setC (F ).
Therefore there exists an ε-neighborhood Vε of x such that Vε ⊆ C (F ).
Consequently Vε ∩ F = ∅, which contradicts the assumption that x is a
cluster point of F .
14.1 Neighborhood 227

Conversely, let F be a subset of R that contains all of its cluster


points; we will show that C (F ) is open. For if y ∈ C (F ), then y is not
a cluster point of F . It follows that there exists an ε-neighborhood Vε
of y that does not contain a point of F (except possiblyy). But since
y ∈ C (F ), it follows that Vε ⊆ C (F ). Since y is an arbitrary element of
C (F ), we deduce that for every point in C (F ) there is an ε-neighborhood
that is entirely contained in C (F ). But this means that C (F ) is open
in R. Therefore F is closed in R.

The Characterization of Open Sets

The idea of an open set in R is a generalization of the notion of an open


interval.

Theorem 6 A subset of R is open if and only if it is the union of


countably many disjoint open intervals in R.

Proof Suppose that G 6= ∅ is an open set in R. For each x ∈ G, let


Ax = {a ∈ R : (a, x] ⊆ G} and let Bx = {b ∈ R : [x, b) ⊆ G}.
Since G is open, it follows that Ax and Bx are not empty. (Reason: G is
open and x ∈ G implies there is an ε > 0 such that (x − ε, x + ε) ⊂ G.
Now take a = x − ε and b = x + ε. Then (a, x] ⊂ (a, x + ε) ⊂ G and
[x, b) ⊂ (x − ε, b) ⊂ G, so that a ∈ Ax andb ∈ Bx . Hence Ax and Bx
are non empty.) If the set Ax is bounded below, we set ax = inf Ax ; if
Ax is not bounded below, we set ax = −∞. Note that in either case
ax ∈
/ G. If the set Bx is bounded above, we set bx = sup Bx ; if Bx is not
bounded above, we set bx = ∞. Note that in either case bx ∈
/ G.

We now define Ix = (ax , bx ); clearly Ix is an open interval containing


x. We claim that Ix ⊆ G. To see this, let y ∈ Ix (then ax < y < bx ) and
suppose that y < x. It follows from the definition of ax that there exists
a0 ∈ Ax with a0 < y, and hence y ∈ (a0 , x] ⊆ G. Similarly, if y ∈ Ix
228 Chapter 14. Open and Closed Sets in R

and x < y, there exists b0 ∈ Bx with y < b0 , and hence it follows that
y ∈ [x, b0 ) ⊆ G. Since y ∈ Ix is arbitrary, we have that Ix ⊆ G.
S
Since x ∈ G is arbitrary, we conclude that Ix ⊆ G. On the other
x∈G
hand, since for each x ∈ G there is an open interval Ix with x ∈ Ix ⊆ G,
S S
we also have G ⊆ Ix . Therefore we conclude that G = Ix .
x∈G x∈G

We claim that if x, y ∈ G and x 6= y, then either Ix = Iy or Ix ∩ Iy =


∅. To prove this suppose that z ∈ Ix ∩ Iy . Then z ∈ Ix and z ∈ Iy and
hence it follows that ax < z < bx and ay < z < by and hence ax < z < by
and ay < z < bx . We will show that ax = ay . If not, it follows from
the Trichotomy Property that either (i) ax < ay , or (ii) ay < ax . In
case (i), then ax < ay < bx and hence ay ∈ Ix = (ax , bx ) ⊆ G, which
contradicts the fact ay ∈
/ G. Similarly, in case (ii), then ay < ax < by and
hence ax ∈ Iy = (ay , by ) ⊆ G, which contradicts the fact that ax ∈
/ G.
Therefore we must have ax = ay and a similar argument implies that
bx = by . Thus ax = ay and bx = by . Hence Ix = Iy . Therefore, we
conclude that if Ix ∩ Iy 6= ∅, then Ix = Iy .

It remains to show that the collection of distinct intervals {Ix : x ∈


G} is countable. To do this, we enumerate the set Q of rational numbers

Q = {r1 , r2 , . . . , rn , . . .}

(such an enumeration is possible as Q is denumerable.) It follows from


the Density Theorem that each interval Ix contains rational numbers;
we select the rational number in Ix that has the smallest index n in this
enumeration of Q. That is, we choose rn(x) ∈ Q such that Irn(x) = Ix
and n(x) is the smallest index n such that Irn = Ix . Thus the set of
distinct intervals Ix , x ∈ G, is put into correspondence with a subset of
N. Hence this set of distinct intervals is countable.
14.1 Neighborhood 229

Remark It can be verified that the representation of G as a disjoint


union of open intervals is uniquely determined.

It does not follow from the preceding theorem that a subset of R is


closed if and only if it is the intersection of a countable collection not
closed intervals. In fact, there are closed sets in R that cannot be
expressed as the intersection of a countable collection of closed intervals
in R. A set consisting of two points is one example. [For, consider {a, b}
with a 6= b . It can never be expressed as the intersection of countable
collection of closed intervals in R.]

We will now describe the construction of a much more interesting


example called the Cantor set.

Cantor Set

The Cantor ternary set F can be described by removing a sequence of


open intervals from the closed unit interval I = [0, 1]. The procedure
follows:

Let F0 = [0, 1]. We first remove the open middle third segment
1 2

3, 3 of [0, 1] to obtain the set

1
F1 = [0, 3] ∪ [ 23 , 1].

Next we remove the open middle third of each of the two closed intervals
in F1 to obtain the set

1
F2 = [0, 9] ∪ [ 29 , 1
3] ∪ [ 23 , 7
9] ∪ [ 89 , 1].

i.e.,
1
F2 = [0, 9] ∪ [ 29 , 3
9] ∪ [ 69 , 7
9] ∪ [ 89 , 1].
230 Chapter 14. Open and Closed Sets in R

We see that F2 is the union of 22 = 4 closed intervals, each of which is


of the form [ 3k2 , k+1
32 ] each having length 3−2 . We next remove the open
middle thirds of each of these sets to get F3 , which is the union of 23 = 8
closed intervals each of length 3−3 . Continuing in this way, we obtain a
sequence of closed sets Fn , such that

(a) F1 ⊃ F2 ⊃ F3 ⊃ · · · ;
k k+1
 
(b) Fn is the union of 2n intervals of the form 3n , 3n each of length
−n
3 .

(c) Fn+1 is obtained from Fn by removing the open middle third of each
of the intervals in Fn .

The set
\
F= Fn
n∈N

is called the Cantor set. Being the intersection of closed sets, the
Cantor set is closed.

Figure 14.5:

Properties of Cantor Set

1. The total length of the removed intervals is 1.

2. The set F contains no nonempty open interval as a subset.


14.1 Neighborhood 231

3. The Cantor set F has infinitely (even uncountably) many points.

Proof of the Properties

Proof of 1:

We note that the first middle third has length 1/3, the next two middle
thirds have lengths that add up to 2/32 , the next four middle thirds
have lengths that add up to 22 /32 , and so on. The total length L of the
removed intervals is given by
∞  n
1 2 2n 1X 2
L = + 2 + · · · + n+1 + · · · = .
3 3 3 3 n=0 3

Using the formula for the sum of a geometric series, we obtain

1 1
L= · = 1.
3 1 − (2/3)

Thus F is a subset of the unit interval [0, 1] whose complement in [0, 1]


has total length 1.
2 n

Note that the total length of the intervals that make up Fn is 3 ,
which has limit 0 as n → ∞. Since F ⊆ Fn for all n ∈ N, we see that if
F can be said to have ‘length’, it must have length 0.

Proof of 2:
T
If F contains a nonempty open interval J = (a, b), then J ⊆ n∈N Fn ,
so that J ⊆ Fn for all n ∈ N, and hence we must have 0 < b − a ≤ (2/3)n
n
for all n ∈ N. As lim 32 = 0, we have b−a = 0, and hence J is empty,
n→∞
a contradiction. Hence the set F contains no nonempty open interval as
a subset.

Proof of 3:
232 Chapter 14. Open and Closed Sets in R

The Cantor set contains all of the endpoints of the removed open in-
tervals, and these are all points of the form 2k /3n where k = 0, 1, . . . , n
for each n ∈ N. There are infinitely many points of this form.

The Cantor set actually contains many more points than those of
the form 2k /3n ; in fact F is an uncountable set. We give an outline of
the argument. We note that each x ∈ [0, 1] can be written in a ternary
(base 3) expansion


X an
x= = (.a1 a2 . . . an . . .)3
n=0
3n

where each an is either 0 or 1 or 2. Indeed, each x that lies in one of


the removed open intervals has an = 1 for some n; for example, each
point in ( 31 , 2
3) has a1 = 1. The endpoints of the removed intervals
have two possible ternary expansions, one having no 1s; for example,
3 = (.100 . . .)3 = (.022 . . .)3 . If we choose the expansion without 1s for
these points, then F consists of all x ∈ [0, 1] that have ternary expansions
with no 1 s; that is, an is either 0 or 2 for all n ∈ N. We now define a
mapping ϕ : F → [0, 1] as follows:
P∞ an  P∞ (an /2)
ϕ n=1 3n = n=1 2n for x ∈ F.

That is, ϕ ((.a1 a2 . . .)3 ) = (.b1 b2 . . .)2 where bn = an /2 for all n ∈ N


and (.b1 b2 . . .)2 denotes the binary representation of a number. Thus ϕ
is a surjection of F onto [0, 1]. If F were countable, then there would
exist a surjection ψ : N → F . Then the composition function ϕ ◦ ψ is
a surjection of Nonto [0, 1]. This implies that [0, 1] is a countable set,
which is a contradiction. Hence F is an uncountable set.

Exercises
14.1 Neighborhood 233

1. Let x ∈ (0, 2). Choose an appropriate εx such that |u − x| < εx


implies u ∈ (0, 1).

2. Show that the intervals (a, ∞) and (−∞, a) are open sets, and that
the intervals [b, ∞) and (−∞, b] are closed sets.

3. Write out the Induction argument in the proof of part (b) of the Open
Set Properties.
1
T 
4. Prove that (0, 1] = n∈N 0, 1 + n .

5. Show that the set N of natural numbers is a closed set.

6. Show that A = {1/n : n ∈ N} is not closed set, but that A ∪ {0} is a


closed set.

7. Show that the set Q of rational numbers is neither open nor closed.

8. Show that if G is an open set and F is a closed set, then G\F is an


open set and F \G is a closed set.

9. A point x ∈ R is said to be an interior point of A ⊆ R in case there


is a neighborhood V of x such that V ⊆ A. Show that a set A ⊆ R is
open if and only if every point of A is an interior point of A.

10. A point x ∈ R is said to be a boundary point of A ⊆ R in case


every neighborhood V of x contains points in A and points in C (A).
Show that a set A and its complement C (A) have exactly the same
boundary points.

11. Show that a set G ⊆ R is open if and only if it does not contain any
of its boundary points.

12. Show that a set F ⊆ R is closed if and only if it contains all of its
boundary points.
234 Chapter 14. Open and Closed Sets in R

13. If A ⊆ R, let A◦ be the union of all open sets that are contained in
A; the set A◦ is called the interior of A. Show that A◦ is an open set,
that it is the largest open set contained in A, and that a point z belongs
to A◦ if and only if z is an interior point of A.

14. Using the notation of the preceding exercise, let A, B be sets in R.


Show that A◦ ⊆ A, (A◦ )◦ = A◦ , and that (A ∩ B)◦ = A◦ ∩ B ◦ . Show
also that A◦ ∪ B ◦ ⊆ (A ∪ B)◦ , and give an example to show that the
inclusion may be proper.

15. If A ⊆ R, let A− be the intersection of all closed sets containing A


the set A− is called the closure of A. Show that A− is a closed set, that
it is the smallest closed set containing A, and that a point ω belongs to
A− if and only if ω is either an interior point or a boundary point of A.

16. Using the notation of the preceding exercise, let A, B be sets in R.


Show that we have A ⊆ A− , (A− )− = A− , and that (A∪B)− = A− ∪B − .
Show that (A ∩ B)− = A− ∩ B − , and give an example to show that the
inclusion may be proper.

17. Give an example of a set A ⊆ R such that A◦ = ∅ and A− = R.

18. Show that if F ⊆ R is a closed nonempty set that is bounded above,


then sup F belongs to F .

19. Show that each point of the Cantor set F is a cluster point of F.

20. Show that each point of the Cantor set F is a cluster point of C(F).

Answers

1. If |x−u| < inf{x, 1−x}, then u < x+(1−x) = 1 and u > x−x = 0,
so that 0 < u < 1. (Ref. following Figures 14.6 and 14.7).
14.1 Neighborhood 235

Figure 14.6:

Figure 14.7:

3. Since the union of two open sets is open, then G1 ∪ · · · ∪ Gk ∪Gk+1 =


(G1 ∪ · · · ∪ Gk ) ∪ Gk+1 is open.

5. The complement of N is the union (−∞, 1) ∪ (1, 2) ∪ · · · of open


intervals.

7. We use the Corollary to Density Theorem: “If x and y are real


numbers with x < y, then there exists an irrational number z such that
x < z < y”. This implies that every neighborhood of x in Q contains a
point not in Q.

10. x is a boundary point of A ⇔ every neighborhood V of x contains


points in A and points in C (A) ⇔ x is a boundary point C (A).

12. The sets F and C (F ) have the same boundary points. Therefore F
contains all of its boundary points ⇔ C (F ) does not contain any of its
boundary points ⇔ C (F ) is open.

13. x ∈ A◦ ⇔ x belongs to an open set V ⊆ A ⇔ x is an interior point


of A.

15. Since A− is the intersection of all closed sets containingA, then by


the result “the intersection of an arbitrary collection of closed sets in R
236 Chapter 14. Open and Closed Sets in R

is closed”, A− is a closed set containingA. Since C (A− ) is open, then


z ∈ C (A− ) ⇔ z has a neighborhood Vε (z) in C (A− ) ⇔ z is neither an
interior point nor a boundary point of A.
Chapter 15
Complex Numbers and their
Properties

Definition: Complex Number

A complex number is any number of the form z = a + ib where a


and b are real numbers and i is the imaginary unit. The real number
a in z = a + ib is called the real part of z; the real number b is called
the imaginary part of z.The real and imaginary parts of a complex
number z are abbreviated Re(z) and Im(z), respectively. For example,
if z = 4 − 9i, then Re(z) = 4 and Im(z) = −9.A real constant multiple of
the imaginary unit is called a pure imaginary number. Two complex
numbers are equal if their corresponding real and imaginary parts are
equal.

Definition: Equality

237
238 Chapter 15. Complex Numbers and their Properties

Complex numbers z1 = a1 + ib1 and z2 = a2 + ib2 are equal, z1 = z2 , if


a1 = a2 and b1 = b2 .

Arithmetic Operations : Complex numbers can be added, subtracted


multiplied, and divided. If z1 = a1 + ib1 and z2 = a2 + ib2 , these
operations are defined as follows.

Addition: z1 + z2 = (a1 + ib1 ) + (a2 + ib2 ) = (a1 + a2 ) + i (b1 + b2 )

Subtraction: z1 − z2 = (a1 + ib1 ) − (a2 + ib2 ) = (a1 − a2 ) + i (b1 − b2 )

Multiplication: z1 ·z2 = (a1 + ib1 ) (a2 + ib2 ) = a1 a2 −b1 b2 +i (b1 a2 + a1 b2 )

Division:

z1 a1 + ib1
= , a2 6= 0, or b2 6= 0
z2 a2 + ib2
a1 a2 + b1 b2 b1 a2 − a1 b2
= 2 2 +i 2
a2 + b2 a2 + b22

Remark: The familiar commutative, associative, and distributive laws


hold for complex

numbers:
(
z1 + z2 = z2 + z1
Commutative laws:
z1 z2 = z2 z 1
(
z1 + (z2 + z3 ) = (z1 + z2 ) + z3
Associative laws:
z1 (z2 z3 ) = (z1 z2 ) z3
Distributive law: z1 (z2 + z3 ) = z1 z2 + z1 z3

Addition, Subtraction, and Multiplication

(i ) To add (subtract ) two complex numbers, simply add (subtract ) the


corresponding real and imaginary parts.
239

(ii ) To multiply two complex numbers, use the distributive law and the
fact that i2 = −1.

EXAMPLE: Addition and Multiplication

If z1 = 2 + 4i and z2 = −3 + 8i, find (a) z1 + z2 and (b) z1 z2 .

Solution (a) By adding real and imaginary parts, the sum of the two
complex numbers z1 and z2 is

z1 + z2 = (2 + 4i) + (−3 + 8i) = (2 − 3) + (4 + 8) i = −1 + 12i.

(b) By the distributive law and i2 = −1, the product of z1 and z2 is

z1 z2 = (2 + 4i) (−3 + 8i)


= (2 + 4i) (−3) + (2 + 4i) (8i)
= −6 − 12i + 16i + 32i2
= (−6 − 32) + (16 − 12)i
= −38 + 4i.

Zero and Unity: The zero in the complex number system is the
number 0 + 0i and the unity is 1 + 0i. The zero and unity are denoted
by 0 and 1, respectively. The zero is the additive identity in the
complex number system since, for any complex number z = a + ib, we
have z + 0 = z. To see this, we use the definition of addition:

z + 0 = (a + ib) + (0 + 0i) = a + 0 + i(b + 0) = a + ib = z.

Similarly, the unity is the multiplicative identity of the system since,


240 Chapter 15. Complex Numbers and their Properties

for any complex number z, we have z · 1 = z · (1 + 0i) = z.

Conjugate: If z is a complex number, the number obtained by changing


the sign of its imaginary part is called the complex conjugate, or
simply conjugate, of z and is denoted by the symbol z. In other words,
if z = a + ib, then its conjugate is z = a − ib. For example, if z = 6 + 3i,
then z = 6 − 3i;

Properties of conjugates :

z1 + z2 = z 1 + z 2 , z1 − z2 = z 1 − z 2
 
z1 z1
z1 z2 = z 1 z 2 , = , z = z.
z2 z2
z + z = (a + ib) + (a − ib) = 2a

zz = (a + ib) (a − ib) = a2 + b2 .
z+z z−z
Re (z) = and Im (z) = .
2 2i

Division

To divide z1 by z2 , multiply the numerator and denominator of z1 /z2


by the conjugate of z2 . That is,

z1 z1 z 2 z1 z 2
= · =
z2 z2 z 2 z2 z 2

and then use the fact that z2 z 2 is the sum of the squares of the real and
imaginary parts of z2 .
241

EXAMPLE : Division

If z1 = 2 − 3i and z2 = 4 + 6i , find z1 / z2 .

Solution We multiply numerator and denominator by the conjugate of


the denominator z 2 = 4 + 6i.

z1 2 − 3i 2 − 3i 4 − 6i 8 − 12i − 12i + 18i2 −10 − 24i


= = = 2 2
=
z2 4 + 6i 4 + 6i 4 − 6i 4 +6 52

Inverses: In the complex number system, every number z has a unique


additive inverse. As in the real number system, the additive inverse
of z = a + ib is its negative, −z, where −z = −a − ib. For any complex
number z, we have z + (−z) = 0. Similarly , every nonzero complex
number z has a multiplicative inverse. In symbols, for z 6= 0 there
exists one and only one nonzero complex number z −1 such that zz −1 = 1.
The multiplicative inverse z −1 is the same as the reciprocal 1/z.

EXAMPLE: Reciprocal

Find the reciprocal of z = 2 − 3i.

Solution By the definition of division we obtain

1 1 1 2 + 3i 2 + 3i 2 + 3i
= = = = .
z 2 − 3i 2 − 3i 2 + 3i 4+9 13

That is,
1 2 3
= z −1 = + i.
z 13 13

 
2 3
zz −1 = (2 − 3i) + i = 1.
13 13
242 Chapter 15. Complex Numbers and their Properties

15.1 Complex Plane

A complex number z = x + iy is uniquely determined by an ordered


pair of real numbers (x, y). For example, the ordered pair (2, −3)
corresponds to the complex number z = 2 − 3i.

Complex Plane: Because of the correspondence between a complex


number z = x + iy and one and only one point (x, y) in a coordinate
plane The horizontal or x -axis is called the real axis because each point
on that axis represents a real number. The vertical or y-axis is called
the imaginary axis because a point on that axis represents a pure
imaginary number.

Vectors In other courses you have undoubtedly seen that the num-
bers in an ordered pair of real numbers can be interpreted as the com-
ponents of a vector. Thus, a complex number z = x + iy can also be
viewed as a two dimensional position vector, that is, a vector whose
initial point is the origin and whose terminal point is the point (x, y).
This vector interpretation prompts us to define the length of the vector z
p
as the distance x2 + y 2 from the origin to the point (x, y). This length
is given a special name as given in the following definition.

Definition: Modulus

The modulus of a complex number z = x + iy, is the real number


p
|z| = x2 + y 2 .

The modulus |z| of a complex number z is also called the absolute


value of z.

EXAMPLE: Modulus of a Complex Number


15.1 Complex Plane 243
q
2
If z = 2−3i, then the modulus of the number to be |z| = 22 + (−3) =
√ q
2
13. If z = −9i, then |−9i| = (−9) = 9.

Properties

2

|z| = zz and |z| = zz


z1 |z1 |
|z1 z2 | = |z1 | |z2 | and =
z2 |z2 |

Distance between two points

Distance between two points z1 = x1 + iy1 and z2 = x2 + iy2 in the


complex plane is the same as the distance between the origin and the
point (x2 − x1 , y2 − y1 ); that is,

|z| = |z2 − z1 | = | (x2 − x1 ) + i (y2 − y1 ) |

or q
2 2
|z2 − z1 | = (x2 − x1 ) + (y2 − y1 ) .

EXAMPLE: Set of Points in the Complex Plane

Describe the set of points z in the complex plane that satisfy |z| = |z −i|.

|z| = |z − i|.
q
2
p
x2 + y 2 = x2 + (y − 1)
2
x2 + y 2 = x2 + (y − 1)

x2 + y 2 = x2 + y 2 − 2y + 1
244 Chapter 15. Complex Numbers and their Properties

Figure 15.1: Horizontal line is the set of points satisfying |z| = |z − i|.

The last equation yields y = 21 . Since the equality is true for arbitrary
x,
1
y=
2
is an equation of the horizontal line shown in Figure 15.1. Complex
numbers satisfying |z| = |z − i| can then be written as z = x + 12 i.

Properties:

|z1 + z2 | ≤ |z1 | + |z2 |

|z1 + z2 | ≥ |z1 | − |z2 |

|z1 − z2 | ≤ |z1 | + |z2 |


15.1 Complex Plane 245

|z1 − z2 | ≥ |z1 | − |z2 |

|z1 + z2 + z3 + · · · + zn | ≤ |z1 | + |z2 | + |z3 | + · · · + |zn | .

EXAMPLE : An Upper Bound



−1
Find an upper bound for z4 −5z+1 , if |z| = 2.

Solution Since the absolute value of a quotient is the quotient of the


absolute values. Thus with | − 1| = 1, we want to find a positive real
number M such that
1
≤ M.
|z 4 − 5z + 1|
To accomplish this task we want the denominator as small as possible.
We can write

4
z − 5z + 1 = z 4 − (5z − 1) ≥ z 4 − |5z − 1| .

But to make the difference in the last expression as small as possible, we


want to make |5z − 1| as large as possible.

|5z − 1| = |5z| + |−1| = 5 |z| + 1.

Using |z| = 2,


z − 5z + 1 ≥ z 4 − |5z − 1| ≥ |z|4 − (5 |z| + 1)
4
246 Chapter 15. Complex Numbers and their Properties

4
= |z| − 5 z |−1| = |16 − 10 − 1| = 5.

Hence for |z| = 2 we have

1 1
≤ .
|z 4 − 5z + 1| 5

15.2 Polar Form of Complex Numbers

Polar coordinate system is superimposed on the complex plane with the


polar axis coinciding with the positive x -axis and the pole O at the
origin. Then x, y, r and θ are related by

x = r cos θ, y = r sin θ .

These equations enable us to express a nonzero complex number z =


x + iy as
z = (r cos θ) + i(r sin θ)

or
z = r (cos θ + i sin θ)

is the polar form or polar representation of the complex number z.

EXAMPLE : A Complex Number in Polar Form



Express − 3 − i in polar form.

Solution: With x = − 3 and y = −1 we obtain
r
√ 2 2
r = |z| = − − 3 + (−1) = 2.

√ √ √
Now y/x = −1/(− 3) = 1/ 3, and so a calculator gives tan−1 (1/ 3)
15.2 Polar Form of Complex Numbers 247

= π/6, which is an angle whose terminal side is in the first quadrant.



But since the point (− 3, −1) lies in the third quadrant, we take the
√ √
solution of tan θ = −1/(− 3) = 1/ 3 to be θ = arg(z )=π/6+π = 7π/6.
The polar form of the number is
 
7π 7π
z = 2 cos + isin .
6 6

The argument θ of a complex number that lies in the interval −π <


θ ≤ π is called the principal value of arg (z) or the principal argu-
ment of z. The principal argument of z is unique and is represented by
the symbol Arg (z), that is,

−π < Arg (z) ≤ π.

Multiplication and Division: The polar form of a complex num-


ber is especially convenient when multiplying or dividing two complex
numbers. Suppose
z1 = r1 (cosθ 1 + isinθ1 )

and
z2 = r2 (cosθ 2 + isinθ2 ) ,

where θ1 and θ2 are any arguments of z1 and z2 , respectively. Then

z1 z2 = r1 r2 [cos θ1 cos θ2 − sin θ1 sin θ2 + i (sin θ1 cos θ2 + cos θ1 sin θ2 )]


(15.1)
and, for z2 6= 0,

z1 r1
= [cos θ1 cos θ2 + sin θ1 sin θ2 + i (sin θ1 cos θ2 − cos θ1 sin θ2 )].
z2 r2
(15.2)
248 Chapter 15. Complex Numbers and their Properties

From the addition formulas for the cosine and sine, (15.1) and (15.2) can
be rewritten as

z1 z2 = r1 r2 [cos ( θ1 + θ2 ) + i sin (θ1 + θ2 )]

and
z1 r1
= [cos ( θ1 − θ2 ) + i sin (θ1 − θ2 )]
z2 r2

EXAMPLE : Argument of a Product and of a Quotient



For z1 = i and z2 = − 3 − i that Arg (z1 ) = π/2 and Arg (z2 ) = −5π/6
respectively. Thus arguments for the product and quotient
√ √ √
z1 z2 = i − 3 − i = 1 − 3i and zz12 = −√i3−i = − 14 − 43 i


 
arg (z1 z2 ) = π2 + − 5π = − π3 and arg zz12 = π2 − − 5π 4π
 
2 2 = 3

Remark: z n = rn (cosnθ + i sinnθ).

Example: Power of a Complex Number



Compute z 3 for z = − 3 − i.

+ isin 7π

Solution: The polar form of the given number is z = 2[cos 6 6) ] ,

with r = 2, θ = 6, and n = 3 we get

 √ 3 


7π 7π 7π
− 3 − i = 23 [cos 3 + isin( 3 )] = 8[cos +sin ] = −8i
6 6 2 2

since cos( 7π 7π
2 ) = 0 and sin( 2 )= −1

de Moivre’s Formula:

n
(cos θ + i sin θ) = cos nθ + i sin nθ
15.2 Polar Form of Complex Numbers 249

Example: de Moivre’s Formula



π 3 1
With θ = 6 , cosθ = 2 and sinθ = 2 :

√ !3
3 1  π π
+ i = cos 3θ + i sin 3θ = cos 3 · + i sin(3 · )
2 2 6 6

π π
= cos + isin = i.
2 2
Example 15.2.1. Cube Roots of a Complex Number Find the
three cube roots of z = i.

Solution Keep in mind that we are basically solving the equation w3 =


i. Now with r = 1, θ = arg(i ) = π/ 2, a polar form of the given number
is given by z = cos(π/ 2) + i sin(π/ 2). Put n = 3, we then obtain

√ π
 
+ 2kπ π
1 cos ( 2 ) + isin (cos( 2 +2kπ
3
wk = 3 )] , k = 0, 1, 2.
3

Hence the three roots are,



π π 3 1
k = 0, w0 = cos + isin = + i
6 6 2 2


5π 5π 3 1
k = 1, w1 = cos + isin = + i
6 6 2 2
3π 3π
k = 2, w2 = cos + isin = −i
2 2

Example Find the four roots of the equation z 4 + 4 = 0 (i.e., find the
fourth roots of −4 ) and use them to factorize z 4 + 4 into quadratic
factors with real coefficients.

Solution
1/4
z 4 + 4 = 0 ⇒ z 4 = −4 ⇒ z = (−4)
250 Chapter 15. Complex Numbers and their Properties

Now −4 = 4 (cos π + i sin π) = 4 (cos (π + 2kπ) + i sin (π + 2kπ))


    
π + 2kπ π + 2kπ
. . . (−4)1/4 = 2 cos + i sin .
4 4

Hence
√  π  π 
c0 = 2 cos + i sin = 1 + i.
4 4

    
3π 3π
c1 = 2 cos + i sin = − 1 + i.
4 4

  
 
5π 5π
c2 = 2 cos + i sin = − 1 − i.
4 4

    
7π 7π
c3 = 2 cos + i sin = 1 − i.
4 4

z 4 + 4 = [z − (1 + i)] [z − (1 − i)] [z − (−1 − i)] [z − (−1 + i)]

= (z 2 + 2z + 2) (z 2 − 2z + 2).

Example 15.2.2. Find all cube roots of 1.

Solution

1 = cos 0 + i sin 0 = cos 2kπ + i sin 2kπ

so
1 2kπ 2kπ
1 3 = cos + i sin , k = 0, 1, 2.
3 3
. . . Hence the required values are

2π 2π 4π 4π
cos 0 + i sin 0, cos + i sin , cos + i sin .
3 3 3 3

or,
15.2 Polar Form of Complex Numbers 251

1, cos 2π 2π
3 ± i sin 3

    
4π 4π 2π 2π
since cos + i sin = cos 2π − + i sin 2π −
3 3 3 3

2π 2π
= cos − i sin
3 3

or 1, cos 120◦ ± i sin 120◦ or, 1, − 12 ± i 3
2 .

15.2.1 Principal nth Root

For a given complex number z 6= 0, arg(z) is infinite-valued. In like


1/n 1/n
manner, z is n-valued; that is, the symbol z represents the set of
n nth roots wk of z. The unique root of a complex number z (obtained
by using the principal value of arg(z) with k = 0) is naturally referred
to as the principal nth root of w.

Example The principal cube root of 1 is obtained by putting k = 0 in


the values in Example 15.2.2, and that value is 1. From Example 15.2.1,

the principal cube root of i (by putting k = 0) is w0 = 12 3 + 12 i.

Example 15.2.3. Fourth Roots of a Complex Number Find the


four fourth roots of z = 1 + i.
√ √
Solution In this case r = 12 + 12 = 2 and θ = arg (z) = π/4 so
that

z = 1 + i = 2 [cos (π/4) + i sin(π/4)]

= 2 [cos (π/4 + 2kπ) + i sin(π/4 + 2kπ)]
252 Chapter 15. Complex Numbers and their Properties

so the four roots of z = 1 + i are given by


√ 1/4  
π/4 + 2kπ
 
π/4 + 2kπ

wk = 2 cos + i sin , k = 0, 1, 2, 3.
4 4

With the aid of a calculator we obtain


√ 1/4  π π

w0 = 2 cos 16 + i sin 16 ≈ 1.1664 + 0.2320 i
√ 1/4  9π 9π

w1 = 2 cos 16 + i sin 16 ≈ −0.2320 + 1.1664 i
√ 1/4 
cos 17π 17π

w2 = 2 16 + i sin 16 ≈ −1.1664 − 0.2320 i
√ 1/4  25π 25π

w3 = 2 cos 16 + i sin 16 ≈ 0.2320 − 1.1664 i

15.3 Sets of Points in the Complex Plane

15.3.1 Circles

q
2 2
Suppose z0 = x0 + i y0 . Since |z − z0 | = (x − x0 ) + (y − y0 ) is the
distance between the points z = x + iy and z0 = x0 + i y0 , the points
z = x + iy that satisfy the equation

|z − z0 | = ρ, ρ > 0, (15.3)

Lie on a circle of radius ρ centered at the point z0 . See Figure 15.2.


15.3 Sets of Points in the Complex Plane 253

Figure 15.2: Circle of radius ρ.

Example: Two Circles

(a) |z| = 1 is an equation of a unit circle centered at the origin.

(b) By rewriting |z − 1 + 3i| = 5 as |z − (1 − 3i )| = 5, we can see


that the equation describes a circle of radius 5 centered at the point z0 =
1−3i.

Disks and Neighborhoods:

The points z that satisfy the inequality |z − z0 | ≤ ρ can be either


on the circle |z − z0 | = ρ or within the circle. We say that the set
of points defined by |z − z0 | ≤ ρ is a disk of radius ρ centered at
z0 . But the points z that satisfy the strict inequality |z − z0 | < ρ lie
within, and not on, a circle of radius ρ centered at the point z0 . This
set is called a neighborhood of z0 .Occasionally , we will need to use a
neighborhood of that also excludes zo . Such a neighborhood is defined
by the simultaneous inequality 0 < |z− z0 | < ρ and is called a deleted
neighborhood of z0 . For example, |z| < 1 defines a neighborhood of
the origin, whereas 0 < |z| < 1 defines a deleted neighborhood of the
254 Chapter 15. Complex Numbers and their Properties

origin; |z − 3 + 4i| < 0.01 defines a neighborhood of 3−4i, whereas the


inequality 0 < |z − 3 + 4i| < 0.01 defines a deleted neighborhood of 3
− 4i.

Open Sets:

A point z0 is said to be an interior point of a set S of the complex


plane if there exists some neighborhood of z0 that lies entirely within
S. If every point z of a set S is an interior point, then S is said to be
an open set. See Figure 15.3. For example, the inequality Re(z ) > 1
defines a right half-plane, which is an open set. All complex numbers z
= x + iy for which x > 1 are in this set.

Figure 15.3: Open set

If every neighborhood of a point z0 of a set S contains at least one point


of S and at least one point not in S, then z0 is said to be a boundary
point of S.For the set of points defined by Re(z ) ≥ 1, the points on the
vertical line x = 1 are boundary points. The points that lie on the circle
|z − i| = 2 are boundary points for the disk |z − i| ≤ 2 as well as for the
15.3 Sets of Points in the Complex Plane 255

Figure 15.4:

neighborhood |z − i| < 2 of z = i. The collection of boundary points


of a set S is called the boundary of S. The circle |z − i| = 2 is the
boundary for both the disk |z − i| ≤ 2 and the neighborhood |z − i| <
2 of z = i. A point z that is neither an interior point nor a boundary
point of a set S is said to be an exterior point of S ; in other words,
z0 is an exterior point of a set S if there exists some neighborhood of
z0 that contains no points of S. Figure 15.5 shows a typical set S with
interior, boundary, and exterior.
256 Chapter 15. Complex Numbers and their Properties

Figure 15.5: Interior, boundary, and exterior of a set S.

Annulus

The set S1 of points satisfying the inequality ρ1 < |z − z0 | exterior


to the circle of radius ρ centered at z0 , whereas the set S2 of points
satisfying |z − z0 | < ρ2 lie interior to the circle of radius ρ2 centered at
z0 . Thus, if 0 < ρ1 < ρ2 , the set of points satisfying the simultaneous
inequality
ρ1 < |z − z0 | < ρ2 , (15.4)

is the intersection of the sets S1 and S2 . This intersection is an open


circular ring centered at z0 . Figure 15.4(d) illustrates such a ring cen-
tered at the origin. The set defined by (15.4) is called an open circular
annulus. By allowing ρ1 = 0, we obtain a deleted neighborhood of z0 .

Domain: If any pair of points z1 and z2 in a set S can be connected


by a polygonal line that consists of a finite number of line segments
joined end to end that lies entirely in the set, then the set S is said to
be connected. An open connected set is called a domain. The set of
numbers z satisfying Re(z )6= 4 is an open set but is not connected since
it is not possible to join points on either side of the vertical line x = 4
by a polygonal line without leaving the set (bear in mind that the points
15.3 Sets of Points in the Complex Plane 257

on the line x = 4 are not in the set). A neighborhood of a point z0 is a


connected set.

Regions: A region is a set of points in the complex plane with all, some,
or none of its boundary points. Since an open set does not contain any
boundary points, it is automatically a region. A region that contains
all its boundary points is said to be closed. The disk defined by |z −
z0 | ≤ ρ is an example of a closed region and is referred to as a closed
disk. A neighborhood of a point z0 defined by |z − z0 | < ρ is an open
set or an open region and is said to be an open disk. If the center z0 is
deleted from either a closed disk or an open disk, the regions defined by
0 < |z − z0 | ≤ ρ or 0 < |z − z0 | < ρ are called punctured disks.
A punctured open disk is the same as a deleted neighborhood of z0 . A
region can be neither open nor closed; the annular region defined by the
inequality 1 ≤ |z − 5| < 3 contains only some of its boundary points
(the points lying on the circle |z − 5| = 1), and so it is neither open nor
closed.

Bounded Sets: Finally, we say that a set S in the complex plane is


bounded if there exists a real number R > 0 such that |z| < R for
every z in S. That is, S is bounded if it can be completely enclosed
within some neighborhood of the origin. In Figure 15.6, the set S shown
in color is bounded because it is contained entirely within the dashed
circular neighborhood of the origin. A set is unbounded if it is not
bounded.
258 Chapter 15. Complex Numbers and their Properties

Figure 15.6: The set S is bounded since some neighborhood of the origin
encloses S entirely.
Chapter 16
Complex Functions

A function f from a set A to a set B is a rule of correspondence that


assigns to each element in A one and only one element in B.

Suppose that f is a function from the set A to the set B.

• If f assigns to the element a in A the element b in B, then we say


that b is the image of a under f, or the value of f at a, and we
write b = f (a).

• The set A—the set of inputs—is called the domain of f and the
set of images in B —the set of outputs—is called the range of f.

• We denote the domain and range of a function f by Dom(f ) and


Range(f ), respectively.

Definition: Complex Function

259
260 Chapter 16. Complex Functions

A complex function is a function f whose domain and range are


subsets of the set C of complex numbers.

A complex function is also called a complex-valued function of a


complex variable.

Examples

(a) Let A = C, the entire complex plane, and define f by

f (z) = z 2 .


Then the domain is C and the range is w = z 2 |z ∈ C . If for the
same function if we take the domain as A = {z ∈ C : |z| < 1} , then

the range is w = z 2 |z ∈ C, |z| < 1 .

(b) The expression g (z) = z + 2Re(z) also defines a complex function.


Some values of g are:

g (i) = i + 2Re (i) = i + 2 (0) = i

and

g (2 − 3i) = 2 − 3i + 2Re (2 − 3i) = 2 − 3i + 2 (2) = 6 − 3i.

Attention! While saying the domain of a function we mean only the set
of points on which the function is defined. When the domain of definition
of a function is not mentioned, we agree that the largest possible set is
to be taken. For example, the functions f1 and f2 defined by
16.1 Real and Imaginary Parts 261

f1 (z) = z 3 + 2 i z − 3 and f2 (z) = | z |

are functions of complex variables, where f 1 and f 2 have the entire


complex plane as the domain; while f 3 given by

1
f3 (z) =
z2 + 1

is undefined at the two points z = i and z = − i in the complex plane


and hence the domain of f 3 is the entire complex plane except the two
points z = i and z = − i. Similarly, for the function

1
f4 (z) = ,
z

the domain of definition is taken to be the set of nonzero complex num-


bers.

Note that f 2 defined above is a real valued function of a complex


variable z while others are complex valued functions of a complex
variable z.

16.1 Real and Imaginary Parts

If w = f (z) is a complex function, then the image of a complex


number z = x + iy under f is a complex number w = u + iv.

We can express any complex function w = f (z) in terms of two real


functions as:

f (z) = u (x, y) + iv (x, y) .


262 Chapter 16. Complex Functions

The functions u(x, y) and v (x, y) are called the real and imaginary
parts of f , respectively.

Example: Real and Imaginary Parts of a Function

If f (z ) = z 2 , then

f (z) = f (x + i y) = (x + i y)2 = x2 − y 2 + i 2xy.

Hence u(x, y) = x2 − y 2 and v(x, y) = 2xy.

Example: Real and Imaginary Parts of a Function

Find the real and imaginary parts of the functions:

1. f (z) = z 2 − (2 + i) z and

2. g (z) = z + 2Re(z)

Solution In each case, we replace the symbol z by x + iy, then simplify.

2
1. f (z) = (x + iy) −(2 + i) (x + iy) = x2 −2x+y−y 2 +(2xy − x − 2y) i.

1. Since g (z) = x + iy + 2Re(x + iy) = 3x + iy, we have u (x, y) = 3x


and v (x, y) = y.

Definition: Complex Exponential Function

The function ez defined by:

ez = ex cos y + i ex sin y
16.1 Real and Imaginary Parts 263

is called the complex exponential function.

Example 16.1.1. Find the values of the complex exponential function


ez , where z = 2 + πi.

Solution For z = 2 + πi, we have x = 2 and y = π, and so e2+πi =


e2 cosπ + ie2 sinπ. Since cosπ = −1 and sinπ = 0, this simplifies to
e2+πi = −e2 .
Chapter 17
Complex Functions as
Mappings

Recall that if f is a real-valued function of a real variable, then the graph


of f is a curve in the Cartesian plane. Graphs are used extensively to
investigate properties of real functions in elementary courses. However,
we will see that the graph of a complex function lies in four-dimensional
space, and so we cannot use graphs to study complex functions. In
this section we discuss the concept of a complex mapping, which was
developed by the German mathematician Bernhard Riemann to give a
geometric representation of a complex function. The basic idea is this.
Every complex function describes a correspondence between points in
two copies of the complex plane. Specifically, the point z in the z-plane
is associated with the unique point w = f (z) in the w-plane. We use
the alternative term complex mapping in place of complex function

264
17.1 Mappings 265

when considering the function as this correspondence between points in


the z-plane and points in the w-plane. The geometric representation
of a complex mapping w = f (z) attributed to Riemann consists of two
figures: the first, a subset S of points in the z-plane, and the second, the
set S 0 of the images of points in S under w = f (z) in the w-plane.

17.1 Mappings

A useful tool for the study of real functions in elementary calculus is the
graph of the function. Recall that if y = f (x) is a real-valued function
of a real variable x, then the graph of f is defined to be the set of all
points (x, f (x)) in the two-dimensional Cartesian plane. An analogous
definition can be made for a complex function. However, if w = f (z) is
a complex function, then both z and w lie in a complex plane. It follows
that the set of all points (z, f (z)) lies in four-dimensional space (two
dimensions from the input z and two dimensions from the output w). Of
course, a subset of four-dimensional space cannot be easily illustrated.
Therefore:

We cannot draw the graph a complex function.

The concept of a complex mapping provides an alternative way of


giving a geometric representation of a complex function. As described
in the section introduction, we use the term complex mapping to refer to
the correspondence determined by a complex function w = f (z) between
points in a z-plane and images in a w-plane. If the point z0 in the z-
plane corresponds to the point w0 in the w-plane, that is, if w0 = f (z0 ),
then we say that f maps z0 onto w0 or, equivalently, that z0 is mapped
onto w0 by f. As an example of this type of geometric thinking, consider
266 Chapter 17. Complex Functions as Mappings

the real function f (x) = x + 2. Rather than representing this function


with a line of slope 1 and y-intercept (0, 2), consider how one copy of the
real line (the x-line) is mapped onto another copy of the real line (the
y-line) by f. Each point on the x-line is mapped onto a point two units
to the right on the y-line (0 is mapped onto 2, while 3 is mapped onto
5, and so on). Therefore, the real function f (x) = x + 2 can be thought
of as a mapping that translates each point in the real line two units to
the right. You can visualize the action of this mapping by imagining the
real line as an infinite rigid rod that is physically moved two units to
the right. On order to create a geometric representation of a complex
mapping, we begin with two copies of the complex plane, the z-plane
and the w-plane, drawn either side-by-side or one above the other. A
complex mapping is represented by drawing a set S of points in the z-
plane and the corresponding set of images of the points in S under f in
the w-plane. This idea is illustrated in Figure 17.1 where a set S in the
z-plane is shown in Figure 17.1(a) and a set labelled S 0 , which represents
the set of the images of points in S under w = f (z), is shown in Figure
17.1(b). From this point on we will use notation similar to that in Figure
17.1 when discussing mappings.

Notation:S0

If w = f (z) is a complex mapping and if S is a set of points in the


z-plane, then we call the set of images of the points in S under f the
image of S under f, and we denote this set by the symbol S 0 . The set
S is called the pre-image of S 0 under f.

Example: Image of a Line under w = z 2

Find the image of the vertical line x = 1 (Fig. 17.2) under the complex
mapping w = z 2 and represent the mapping graphically.
17.1 Mappings 267

Figure 17.1:

Solution: Let C be the set of points on vertical line x = 1 or, equiv-


alently, the set of points z = 1 + iy with −∞ < y < ∞. The real and
imaginary parts of w = z 2 are u (x, y) = x2 − y 2 and v (x, y) = 2xy,
respectively. For a point z = 1 + iy in C, we have u (1, y) = 1 − y 2
and v (1, y) = 2y. This implies that the image of S is the set of points
w = u + iv satisfying the simultaneous equations:

u = 1 − y2

and
v = 2y

for −∞ < y < ∞.


268 Chapter 17. Complex Functions as Mappings

Figure 17.2: The vertical line x = 1; i.e., the line Rez = 1.

Figure 17.3: The image of C is the parabola u = 1 − 14 v 2 .

The above equations are parametric equations in the real parameter


y, and they define a curve in the w−plane. We can find a Cartesian
17.1 Mappings 269

equation in u and v for this curve by eliminating the parameter y.


 v 2 v2
u=1− =1−
2 2

Since y can take on any real value and since v = 2y, it follows that v
can take on any real value in . Consequently, C 1 the image of C− is
a parabola in the w -plane with vertex at (1,0) and u-intercepts at (0,
±2). . In conclusion, we have shown that the vertical line x = 1 shown
in Figure 17.2 is mapped onto the parabola u = 1 − 14 v 2 shown in Figure
17.3 by the complex mapping w = z 2 .

If x = x(t) and y = y(t) are real-valued functions of a real variable


t, then the set C of all points (x (t) , y (t)) , where a ≤ t ≤ b, is called
parametric curve. The equations x = x (t) , y = y (t) , a ≤ t ≤ b are
called parametric equations of C.

Definition: Parametric Curves in the Complex Plane

if x(t) and y(t) are real-valued functions of a real variable t, then the set
C consisting of all points

z (t) = x (t) + iy (t) , a ≤ t ≤ b,

is called a parametric curve or a complex parametric curve. The


complex-valued function of the real variable t,

z (t) = x (t) + iy (t)

is called a parametrization of C.

Common Parametric Curves in the Complex Plane


270 Chapter 17. Complex Functions as Mappings

• Line A parametrization of the line containing the points z0 and


z1 is:
z (t) = z0 (1 − t) + z1 t, −∞ < t < ∞

• Line Segment A parametrization of the line segment from z0 to


z1 is:
z (t) = z0 (1 − t) + z1 t, 0 ≤ t ≤ 1

• Ray A parametrization of the ray emanating from z0 and contain-


ing z1 is:
z (t) = z0 (1 − t) + z1 t, 0 ≤ t < ∞

• Circle A parametrization of the circle centered at z0 with radius


r is:
z (t) = z0 + r(cost + isint), 0 ≤ t ≤ 2π.

In exponential notation, this parametrization is:

z (t) = z0 + reit , 0 ≤ t ≤ 2π.

Image of a Parametric Curve under a Complex Mapping

If w = f (z) is a complex mapping and if C is a curve parametrized


by z (t) , a ≤ t ≤ b, then

w (t) = f (z (t)) , a≤t≤b

is a parametrization of the image, C 0 of C under w = f (z) .


17.2 Linear Mappings: 271

17.2 Linear Mappings:

We define a complex linear function to be a function of the form

f (z) = az + b

where a and b are any complex constants.

Translations:

A complex linear function

T (z) = z + b, b 6= 0,

is called a translation.

Example: Image of a Square under Translation

Find the image S 1 of the square S with vertices at 1 + i, 2 + i, 2 + 2i,


and 1 + 2i under the linear mapping T (z) = z + 2 − i.

Solution We will represent S and S 1 in the same copy of the complex


plane. The mapping T is a translation, and so S 1 can be determined as
follows. Identifying b = x0 + iy0 = 2 + i(−1) . We plot the vector (2,
-1) originating at each point in S. See figure 17.5. The set of terminal
points of these vectors if S 1 , the image of S and T . Inspection of Figure
17.4 indicates that S 1 is a square with vertices at:

T (1 + i) = (1 + i) + (2 − i) = 3 T (2 + i) = (2 + i) + (2 − i) = 4
272 Chapter 17. Complex Functions as Mappings

T (2 + 2i) = (2 + 2i) + (2 − i) = 4 + i

and
T (1 + 2i) = (1 + 2i) + (2 − i) = 3 + i

Figure 17.4: Image of a Square under Translation

Therefore, the square S shown in color in Figure 17.4 is mapped onto


the square S 1 shown in black by the translation T (z) = z + 2 − i.

Rotation: A complex linear function

R (z) = az, |a| = 1,

is called a rotation.
17.2 Linear Mappings: 273

Magnifications: A complex linear function

M (z) = az, a > 0,

is called a magnification.

Example: Image of a Circle under Magnification

Find the image of the circle C given by |z| = 2 under the linear mapping
M (z ) = 3z.

Figure 17.5: Image of a circle under magnification.

Solution Since M is a magnification with magnification factor of 3,


each point on the circle |z| = 2 will be mapped onto a point with the
274 Chapter 17. Complex Functions as Mappings

same argument but with modulus magnified by 3. Thus, each point in


the image will have modulus 3 · 2 = 6. The image points can have any
argument since the points z in the circle |z| = 2 can have any argument.
Therefore, the image C 1 is the circle |w| = 6 that is centered at the origin
and has radius 6. In Figure 17.5 we illustrate this mapping in a single
copy of the complex plane. Under the mapping M (z ) = 3z, the circle C
shown in Figure 17.5 is mapped onto the circle C’ shown in Figure 17.5.

Image of a Point under a Linear Mapping

Let f (z) = az + b be a linear mapping with a 6= 0 and let z0 be a point in


the complex plane. If the point w0 = f (z0 ) is plotted in the same copy
of the complex plane as z0 , then w0 is the point by

(i). rotating z0 through an angle of Arg(a) about the origin,

(ii). magnifying the result by |a|, and

(iii). translating the result by b.

A complex linear mapping w = az + b with a 6= 0 can distort the size of


a figure in the complex plane, but it cannot alter the basic shape of the
figure.

Example: Image of a Rectangle under a Linear Mapping

Find the image of the rectangle with vertices −1+i, 1+i, 1+2i, and
−1+2i under the linear mapping f(z ) = 4iz + 2 + 3i.

Solution: Let S be the rectangle with the given vertices and let S 1
denote the image of S under f. We will plot S and S 1 in the same copy
17.3 Special Power Functions 275

of the complex plane. Because f is a linear mapping, our foregoing


discussion implies that S 0 has the same shape as S. That is, S 0 is also a
rectangle. Thus, in order to determine S 0 , we need only find its vertices,
which are the images of the vertices of S under f : f (−1 + i ) = −2 −i
f (1 + i ) = −2 + 7i f (1 + 2i ) = −6 + 7i f (−1 + 2i ) = −6 − i.
Therefore, S 0 is the rectangle with vertices −2−i, −2+7i, −6+7i, and
−6−i.

17.3 Special Power Functions

A complex polynomial function is a function of the form

p (z) = an z n + an−1 z n −1 + · · · + a1 z + a0

where n is a positive integer and an , an−1 , . . . , a1 , a0 are complex con-


stants.

Example: Image of a Circular Arc under w = z 2 .


π
Find the image of the circular arc defined by |z| = 2, 0 ≤ argz ≤ 2,
under the mapping w = z 2 .

π
Solution Let C be the circular arc defined by |z| = 2, 0 ≤ arg(z) ≤ 2,
shown in color in Figure 2.18(a), and let C 1 d enote the image of C under
w = z 2 . Since each point in C has modulus 2 and since the mapping w
= z 2 squares the modulus of a point, it follows that each contained in the
circle |w| = 4 centered at the origin with radius 4. Since the arguments
of the points in C take on every value in the interval [0, π/ 2] and since
the mapping w = z 2 doubles the argument of a point, it follows that
the points in C 1 have arguments that take on every value in the interval
276 Chapter 17. Complex Functions as Mappings

[2 · 0, 2 · (π/2)] = [0, π]. That is, the set C 1 is the semicircle defined by
|w| = 4, 0 ≤ arg(w ) ≤ π. In conclusion, we have shown that w = z 2
maps the circular arc C shown in in Figure 17.6(a) onto the semicircle
C 0 shown in black in Figure 17.6(b).

Figure 17.6: The mapping w = z 2 .

Example 4 Image of a Circular Wedge under w = z 3 .


17.3 Special Power Functions 277

Determine the image of the quarter disk defined by the inequalities |z|
≤ 2, 0 ≤ arg(z ) ≤ π/ 2, under the mapping w = z 3 .

Figure 17.7: The mapping w = z 3 .

Solution: Let S denote the quarter disk and let S 1 denote its image
278 Chapter 17. Complex Functions as Mappings

under w = z 3 . Since the moduli of the points in S vary from 0 to 2 and


since the mapping w = z 3 cubes the modulus of a point, it follows that
the moduli of the points in S 1 vary from 03 = 0 to 23 = 8. In addition,
because the arguments of the points in S vary from 0 to π/ 2 and because
the mapping w = z 3 triples the argument of a point, we also have that
the arguments of the points in S 1 vary from 0 to 3π/ 2. Therefore, S 1 is
the set given by the inequalities |w| ≤ 8, 0 ≤ arg(w ) ≤ 3π/ 2.

Definition: Principal Square Root Function

1
The function z 2 defined by:

1 p
z2 = |z|eiArg(z)/2 (17.1)

is called the principal square root function.

Definition: Inverse Function

If f is a one-to-one complex function with domain A and range B , then


the inverse function of f , denoted by f −1 , is the function with domain
B and range A defined by f −1 (z) = w if f (w) = z.

Example: Inverse Function of f (z) = z + 3i.

Show that the complex function f (z) = z + 3i is one-to-one on the


entire complex plane and find a formula for its inverse function.Solve
the equation z = f (w) for w to find a formula for w = f −1 (z) .

Solution: One way of showing that f is one-to-one is to show that the


17.3 Special Power Functions 279

equality f (z1 ) = f (z2 ) implies the equality z1 = z2 . For the function


f (z ) = z + 3i, this follows immediately since z1 +3i = z2 +3i implies
that z1 = z2 .. As with real functions, the inverse function of f can often
be found algebraically by solving the equation z = f (w ) for the symbol
w. Solving z = w + 3i for w, we obtain w = z − 3i and so f −1 (z ) = z
− 3i.

Definition: Principal nth Root Functions

For n ≥ 2, the function z 1/n defined by:


p
z 1/n = n |z| eiArg(z)/n

is called the principal nth root function.

1
Example: Values of z n
1
Find the value of the given principal nth root function z n at the point
z = i where n = 3.

Solution:

For z = i, we have |z| = 1 and Arg (z) = π/2. Substituting these


values with n = 3 we obtain:

√ π 3 1
i1/3 = 1ei( 2 )/3 = eiπ/6 =
3
+ i.
2 2
Chapter 18
Limit of a Function of
Complex Variable

Let a function f be defined at all points z in some deleted1 neighborhood


of z 0 . The expression
lim f (z) = w0
z → z0

means that limit of f (z)as zapproaches z0 is a number w0 . This means


that the values of w = f (z) can be made arbitrarily close to w0 if we
choose the point z close to z 0 but distinct from z0 .

A δ-deleted neighborhood of z0 is the set consisting of all points zin


the δ-neighborhood of z0 except for the point z0 itself (Fig. 18.1).

1 Deleted neighborhood of a point means a neighborhood of the point excluding

the point itself.

280
281

Figure 18.1:

The deleted neighborhood 0 < |z − z0 | < δ consists of all points z in


a δ−neighborhood of z0 except for the point z0 itself.

Formal Definition of Limit

The statement
lim f (z) = w0
z → z0

means that, for each positive numberε, there is a positive number δ such
that
|f (z) − w0 | < ε whenever 0 < |z − z0 | < δ.

Geometrical interpretation of this definition is that, for each ε neigh-


borhood |w − w0 | < ε of w0 , there is a deleted δ neighborhood 0 <
|z − z0 | < δ of z0 such that every point z in it has an image w lying in
the ε neighborhood |w − w0 | < ε. (Fig.18.2 )
282 Chapter 18. Limit of a Function of Complex Variable

Figure 18.2:

Attention! Note that even though all points in the deleted neigh-
borhood 0 < |z − z0 | < δ are to be considered, their images need not
constitute the entire neighborhood |w − w0 | < ε. For example, if f is
defined by

f (z) = w0 ( w0 constant)

the image of z is always the center of the neighborhood |w − w0 | < ε.

Remark Once a δ has been found, it can be replaced by any smaller


positive number, such as δ/3.

Example Let a and b are constants, with a 6= 0; and zis a complex


variable. Then show that lim (az + b) = az0 + b .
z → z0

Solution

Let f (z) = az + b and w0 = az0 + b. Here for each positive numberε,


we must find a positive number δ such that

|f (z) − (az0 + b)| < ε whenever 0 < | z − z0 | < δ.


283

Now

|f (z) − (az0 + b)| = |(az + b) − (az0 + b)| = |a(z − z0 )| = |a| |z − z0 | .

Hence, for any ε > 0

ε
|f (z) − (az0 + b)| < ε whenever 0 < | z − z0 | < .
|a|

ε
Let δ = |a| . Then |f (z) − (az0 + b)| < ε whenever 0 < | z − z0 | <
δ.

Hence, by the formal definition of limit,

lim (az + b) = az0 + b .


z → z0

iz
Example Show that if f (z) = 2 in the open disk |z| < 1, then

iz i
lim f (z) = lim = .
z→1 z→1 2 2

Solution

Take z0 = 1. Clearly 1 is a boundary point of the open disk |z| < 1.


When z is in the region |z| < 1,

z−1
f (z) − i
iz i
2 − 2 =
=
2 .

2

Hence for any such z and any positive number  ,



f (z) − i

< ε whenever 0 < | z − 1 | < 2ε.
2

Therefore corresponding to the ε chosen, we can find a positive real


284 Chapter 18. Limit of a Function of Complex Variable

number δ = 2ε such that



f (z) − i


2

whenever 0 < | z − 1 | < δ and z is in the region |z| < 1.

Hence,
i
lim f (z) = .
z→1 2
i z̄
Example Show that if f (z) = 2 in the open disk |z| < 1, then

iz̄ i
lim f (z) = lim = .
z→1 z→1 2 2

Solution

Proceed as in the solution of previous example. Note that



i z̄ i 1 1 1 1
2 − 2 = 2 |i( z̄ − 1)| = 2 | z̄ − 1| = 2 z − 1 = 2 |z − 1| .

Two Path Test for Non-Existence of Limit at a Point

If f approaches two complex numbers L1 6= L2 for two different


curves or paths through z0 , then lim f (z) does not exist.
z→z0
z
Example If f (z) = z̄ then show by two path test that lim f (z) does
z→0
not exist.
285

Figure 18.3:

Solution If lim f (z) exists, then, by the discussion just above, then it
z→0
could be found by letting the point z = (x, y) approach the origin
0 = (0, 0) in any manner. But when z = (x, 0) is a nonzero point on
the real axis Fig. 18.3,

x + i0 x
f (z) = = = 1;
x − i0 x

and when z = (0, y) is a nonzero point on the imaginary axis,

0 + iy iy
f (z) = =− = −1.
0 − iy iy

Thus, by letting z approach the origin along the real axis, we would find
that the desired limit is 1. By letting z approach the origin along the
imaginary axis, we would find that the desired limit is −1. Since a limit,
if it exists, is unique , we conclude that lim f (z) does not exist for the
z→0
given function.

Theorem: Real and Imaginary Parts of a Limit Suppose that

f (z) = u(x, y) + i v(x, y), z0 = x0 + i y0 andw0 = u0 + i v0 .


286 Chapter 18. Limit of a Function of Complex Variable

Then
lim f (z) = w0
z →z0

if and only if

lim u (x, y) = u0 and lim v (x, y) = v0 .


(x, y) → (x0 , y0 ) (x, y) → (x0 , y0 )

Example Using theorem show that

lim c = c
z → z0

for any complex constant c = a + b i and any z0 .

Solution

Let c = a + ib, then

f (z) = u (x, y) + i v(x, y) = a + i b, z0 = x0 + i y0 and w0 = a + i b.

Since
lim u (x, y) = lim a=a
(x, y) → (x0 , y0 ) (x, y) → (x0 , y0 )

and lim v (x, y) = lim b = b,


(x, y) → (x0 , y0 ) (x, y) → (x0 , y0 )

applying Theorem, we obtain lim f (x) = w0 i.e., lim c = c.


z → z0 z → z0

Example Show that lim z = z0 for any z0 .


z → z0

Solution

Let

f (z) = u + i v = x + i y, and z0 = x0 + i y0 .

Since
lim u (x, y) = lim x = x0
(x, y) → (x0 , y0 ) (x, y) → (x0 , y0 )
287

and lim v (x, y) = lim y = y0 ,


(x, y) → (x0 , y0 ) (x, y) → (x0 , y0 )

applying Theorem, we obtain

lim z = u0 + i v0 = z0 .
z → z0

Example Show that lim (2x − i y 2 ) = 4 i .


z→2i

Solution

By Theorem, with

f (z) = u(x, y) + i v(x, y) = 2x + i (−y 2 ), z0 = x0 + i y0 = 0 + i 2 and


w0 = u0 + i v0 = 0 + i 4,

lim f (z) = lim (2x − i y 2 ) = 4 i = w0


z →z0 z→2i

if and only if

lim u (x, y) = lim u (x, y) = 0 = u0


(x, y) → (x0 , y0 ) (x, y) → (0, 2i)

and lim v (x, y) = −(2i)2 = 4 = v0 .


(x, y) → (0, 2i)

As last two equations are obviously true, we are done.

Theorem (Limits of Sum, Product and Quotient) Suppose that

lim f (z) = w0 and lim F (z) = W0 .


z →z0 z →z0

Then
(i) lim [f (z) + F (z)] = w0 + W0 .
z →z0

(ii) lim [f (z)F (z)] = w0 W0 .


z →z0

f (z) w0
(iii) lim = , provided W0 6= 0.
z →z0 F (z) W0
288 Chapter 18. Limit of a Function of Complex Variable

Proof The above results can be proved directly using the definition
of the limit of a function of a complex variable. But, with the aid of
Theorem, it follows almost immediately from theorems on limits of real-
valued functions of two real variables.

We only prove (ii ) leaving the verifications of (i ) and (iii ) as an


exercise.

To verify (ii ), we write

f (z) = u(x, y) + i v(x, y), F (z) = U (x, y) + i V (x, y),

z0 = x0 + i y0 , w0 = u0 + i v0 , W0 = U0 + i V0 .

Then, according to hypotheses (??) and Theorem 2, the limits as (x, y)


approaches (x0 , y0 ) of the functions u, v, U, and V exist and have
the values u0 , v0 , U0 , and V0 , respectively. So the real and imaginary
components of the product

f (z)F (z) = (u + i v)(U + i V ) = (uU − vV ) + i(vU + uV )

have limits u0 U0 − v0 V0 and v0 U0 + u0 V0 , respectively, as (x, y) ap-


proaches (x0 , y0 ). Hence, by Theorem 2 again, f (z)F (z) has the limit

(u0 U0 − v0 V0 ) + i(v0 U0 + u0 V0 )

as z approaches z0 ; and this is equal to

w0 W0 .

This proves (ii ).


289

Example
lim z n = z0n (n = 1, 2, . . . ).
z → z0

Solution

We have noted in an earlier example that

lim z = z0 .
z → z0

Now
lim z 2 = lim (z · z)
z → z0 z → z0

  
= lim z lim z , using Property (ii) of Theorem
z → z0 z → z0

= z0 z0 = z02 .

Proceeding similarly, or by mathematical induction, we obtain

lim z n = z0n (n = 1, 2, . . . ).
z → z0

Example (Limit of a Polynomial Function)

If P (z) is a polynomial, show that

lim P (z) = P (z0 ).


z → z0

Solution

Let
P (z) = a0 + a1 z + a2 z 2 + · · · + an z n ,
290 Chapter 18. Limit of a Function of Complex Variable

then
a0 + a1 z + a2 z 2 + · · · + an z n

lim P (z) = lim
z → z0 z → z0
     
2
= lim a0 + lim a1 lim z + lim a2 lim z
z → z0 z → z0 z → z0 z → z0 z → z0
  
n
+··· + lim an lim z ,
z → z0 z → z0

using properties (i) and (ii) of Theorem.

= a0 + a1 z0 + a2 z02 + · · · + an z0n , using previous examples.

= P (z0 ).

Example Evaluate lim (z 2 − 2z + 1).


z →1+i

Solution

lim (z 2 − 2z + 1) = lim z 2 − 2 lim z + 1



z →1+i z →1+i z →1+i

= (1 + i)2 − 2(1 + i) + 1

= 1 + 2i − 1 − 2(1 + i) + 1

= −1.

18.1 Continuity

Definition A function f (z) is said to be continuous at the point


z = z0 if f (z0 ) is defined and lim f (z) = f (z0 ) .
z →z0

i.e., f is continuous at z0 if
18.1 Continuity 291

(i ) f (z0 ) exists

(ii) lim f (z) exists


z →z0

(iii) lim f (z) = f (z0 )


z →z0

Formally, f (z) is said to be continuous at the point z = z0 if for every


positive real number  (no matter how small  is but not 0) we can find
a positive real number δ such that

|f (z) − f (z0 )| < ε whenever |z − z0 | < δ.

Attention! Observe that in the definition of limit of f at x0 we take


deleted δ-neighborhood of z0 while in the above definition of continuity
at a point, we take the δ-neighborhood of z0 .

A function of a complex variable is said to be continuous in a


region R if it is continuous at every point in R.

Example The function f (z) = xy 2 + i (2x − y) is continuous in the


complex plane because for any point z0 = x0 + i y0 ,

(i) f (z0 ) = x0 y0 2 + i (2x0 − y0 ) exists

(ii) lim f (z) = x0 y02 + i (2x0 − y0 )exists


z →z0

(iii) lim f (z) = x0 y0 2 + i (2x0 − y0 ) = f (z0 ) .


z →z0

Example The function f (z) = exy + i sin(x2 − 2xy 3 ) is continuous in


the complex plane because for any point z0 = x0 + i y0 ,

lim f (z) = ex0 y0 + i sin(x0 2 − 2x0 y0 3 ) = f (z0 )


z →z0

Theorem A function

f (z) = u(x, y) + i v(x, y)


292 Chapter 18. Limit of a Function of Complex Variable

is continuous at a point z0 = (x0 , y0 ) if and only if its components


functions u and v are continuous there.

Remark In view of Theorem 1, the function f (z) = xy 2 + i (2x − y) in


Example 1 is continuous everywhere in the complex plane because the
component functions xy 2 and 2x − y are polynomials in x and y and are
therefore continuous at each point (x, y).

Theorem If two functions are continuous at a point, their sum and


product are also continuous at that point. Their quotient is continuous
at any such point where the denominator is not zero.

Theorem Continuity of Polynomial Functions Polynomial func-


tions are continuous on the entire complex plane C.

Theorem Continuity of Rational Functions Rational functions are


continuous on their domains.

A Bounding Property If a complex function f is continuous on a


closed and bounded region R, then f is bounded on R. That is, there is
a real constant M > 0 such that |f (z)| < M for all z in R.

18.2 Branches

Definition A branch of a multiple-valued function f is any single-valued


function F that is analytic in some domain at each point z of which the
value F (z) is one of the values f (z).

Definition A branch cut is a portion of a line or curve that is in-


troduced in order to define a branch F of a multiple-valued function f.
Points on the branch cut for F are singular points2 of F, and any point
that is common to all branch cuts of f is called a branch point.
18.2 Branches 293

Example Show that the function f1 defined by


f1 (z) = r eiθ/2 , −π < θ < π (18.1)

is a branch of the multiple-valued function F (z) = z 1/2 .

Solution The domain of the function f1 is the set Dom(f1 ) defined by


|z| > 0, −π < arg(z) < π, shown in Figure 18.4.

Figure 18.4:

The function f1 agrees with the principal square root function f on


this set. Thus, f1 does assign to the input z exactly one of the values
of F (z) = z 1/2 . It remains to show that f1 is a continuous function
on its domain. In order to see that this is so, let z be a point with
|z| > 0, −π < arg(z) < π. If z = x + i y and x > 0, then z = reiθ where
p
r = x2 + y 2 and θ = tan−1 (y/x). Since −π/2 < tan−1 (y/x) < π/2,
the inequality π < θ < π is satisfied. Thus, substituting the expressions
294 Chapter 18. Limit of a Function of Complex Variable

for r and θ in (18.1) we obtain:


p −1
f1 (z) = 4
x2 + y 2 ei tan (y/x)/2
tan−1 (y/x) tan−1 (y/x)
p   p  
4
= x2 + y 2 cos + i 4 x2 + y 2 sin
2 2

Because the real and imaginary parts of f1 are continuous real functions
for x > 0, we conclude that f1 is continuous for x > 0. A similar argu-
ment can be made for points with y > 0 using θ = cot−1 (x/y) and for
points with y < 0 using θ = − cot−1 (x/y). In each case, we conclude
that f1 is continuous. Therefore, the function f1 defined in (18.1) is a
branch of the multiple-valued function F (z) = z 1/2 .
Syllabus

B.Sc. Mathematics: SEMESTER 5

MTS5 B06 : BASIC ANALYSIS

No. of Hours of Lectures/week: 5


No. of Credits: 4
100 Marks [Int:20+Ext:80]

Aims, Objectives and Outcomes

In this course, basic ideas and methods of real and complex analysis are
taught. Real analysis is a theoretical version of single variable calculus.
So many familiar concepts of calculus are reintroduced but at a much
deeper and more rigorous level than in a calculus course. At the same
time there are concepts and results that are new and not studied in the
calculus course but very much needed in more advanced courses. The
aim is to provide students with a level of mathematical sophistication

295
296 Syllabus

that will prepare them for further work in mathematical analysis and
other fields of knowledge, and also to develop their ability to analyse and
prove statements of mathematics using logical arguments. The course
will enable the students

• to learn and deduce rigorously many properties of real number


system by assuming a few fundamental facts about it as axioms. In
particular they will learn to prove Archimedean property, density
theorem, existence of a positive square root for positive numbers
and so on and the learning will help them to appreciate the beauty
of logical arguments and embolden them to apply it in similar and
unknown problems.

• to know about sequences ,their limits, several basic and important


theorems involving sequences and their applications . For example,
they will learn how monotone convergence theorem can be used in
establishing the divergence of the harmonic series, how it helps
in the calculation of square root of positive numbers and how it
establishes the existence of the transcendental number e (Euler
constant).

• to understand some basic topological properties of real number


system such as the concept of open and closed sets, their properties,
their characterization and so on.

• to get a rigorous introduction to algebraic, geometric and topo-


logical structures of complex number system, functions of complex
variable, their limit and continuity and so on. Rich use of geome-
try, comparison between real and complex calculus − areas where
they agree and where they differ, the study of mapping properties
of a few important complex functions exploring the underlying ge-
ometry etc. will demystify student’s belief that complex variable
theory is incomprehensible.

Syllabus

Text (1) Introduction to Real Analysis(4/e) : Robert G Bartle, Donald


R Sherbert John Wiley & Sons(2011) ISBN 978-0-471-43331-6
Syllabus 297

Text (2) Complex Analysis A First Course with Applications (3/e): Den-
nis Zill & Patric Shanahan Jones and Bartlett Learning(2015) ISBN:1-
4496-9461-6
Module-I Text (1) (20 hrs)
1.3: Finite and Infinite Sets-definition, countable sets, denumerability of
Q, union of countable sets, cantor’s theorem
2.1: The Algebraic and Order Properties of R- algebraic properties, basic

results, rational and irrational numbers, irrationality of 2, Order prop-
erties, arithmetic-geometric inequality, Bernoulli’s Inequality
2.2: Absolute Value and the Real Line- definition, basic results, Triangle
Inequality, The real line, ε-neighborhood
2.3: The Completeness Property of R- Suprema and Infima, alternate
formulations for the supremum, The Completeness Property
Module-II Text (1) (21 hrs)
2.4: Applications of the Supremum Property-
√ The Archimedean Prop-
erty, various consequences, Existence of 2, Density of Rational Num-
bers in R, The Density Theorem, density of irrationals
2.5: Intervals-definition, Characterization of Intervals, Nested Intervals,
Nested Intervals Property, The Uncountability of R, [binary, decimal and
periodic representations omitted] Cantor’s Second Proof.
3.1: Sequences and Their Limits- definitions, convergent and divergent
sequences, Tails of Sequences, Examples
3.2: Limit Theorems- sum, difference, product and quotients of sequences,
Squeeze Theorem, ratio test for convergence
3.3: Monotone Sequences-definition, monotone convergence theorem, di-
vergence of harmonic series, calculation of square root, Euler’s number
Module-III Text (1) (18 hrs)
3.4: Subsequences and the Bolzano-Weierstrass Theorem- definition,
limit of subsequences, divergence criteria using subsequence, The Ex-
istence of Monotone Subsequences, monotone subsequence theorem, The
298 Syllabus

Bolzano-Weierstrass Theorem, Limit Superior and Limit Inferior


3.5: The Cauchy Criterion- Cauchy sequence, Cauchy Convergence Cri-
terion, applications, contractive sequence
3.6: Properly divergent sequences-definition, examples, properly diver-
gent monotone sequences, “comparison theorem” , “limit comparison
theorem”
11.1: Open and Closed sets in R- neighborhood, open sets, closed sets,
open set properties, closed set properties, Characterization of Closed
Sets, cluster point, Characterization of Open Sets, The Cantor Set, prop-
erties
Module-IV Text (2) (21 hrs)
1.1: Complex numbers and their properties- definition, arithmetic oper-
ations, conjugate, inverses, reciprocal
1.2: Complex Plane- vector representation, modulus, properties, triangle
inequality
1.3: Polar form of complex numbers polar representation, principal ar-
gument, multiplication and division, argument of product and quotient,
integer powers, de Moivre’s formula.
1.4: Powers and roots- roots, principal nth root
1.5: Sets of points in the complex plane- circles, disks and neighbour-
hoods, open sets, annulus, domains, regions, bounded sets
2.1: Complex Functions- definition, real and imaginary parts of complex
function, complex exponential function, exponential form of a complex
number, Polar Coordinates
2.2: Complex Functions as mappings- complex mapping, illustrations,
Parametric curves in complex planes, common parametric curves, image
of parametric curves under complex mapping [ The subsection ‘Use of
Computers’ omitted]
2.3: Linear Mappings- Translations, Rotations, Magnifications, gen-
eral linear mapping, image of geometric shapes under linear map
2.4: Special Power functions- The power function z n , The power function
Syllabus 299

z 1/n , principal square root function, Inverse Functions, multiple valued


functions
3.1: Limit and Continuity- Limit of a complex function, condition for non
existence of limit, real and imaginary parts of limit, properties of com-
plex limits, continuity, discontinuity of principal square root function,
properties of continuous functions, continuity of polynomial and rational
functions, Bounded Functions, Branches, Branch Cuts and Points
References:

1. Charles G. Denlinger: Elements of Real Analysis Jones and Bartlett


Publishers Sudbury, Massachusetts (2011) ISBN:0-7637-7947-4 [ In-
dian edition:ISBN- 9380853157]
2. David Alexander Brannan: A First Course in Mathematical Analysis
Cambridge University Press,US(2006) ISBN: 9780521684248
3. John M. Howie: Real Analysis Springer Science & Business Me-
dia(2012) [Springer Undergraduate Mathematics Series] ISBN: 1447103416
4. James S. Howland: Basic Real Analysis Jones and Bartlett Publishers
Sudbury,Massachusetts (2010) ISBN:0-7637-7318-2
5. James Ward Brown, Ruel Vance Churchill: Complex variables and ap-
plications(8/e) McGraw-Hill Higher Education, (2009) ISBN: 0073051942
6. Alan Jeffrey: Complex Analysis and Applications(2/e) Chapman and
Hall/CRC Taylor Francis Group(2006)ISBN:978-1-58488-553-5
7. Saminathan Ponnusamy, Herb Silverman: Complex Variables with
Applications Birkhauser Boston(2006) ISBN:0-8176-4457-4
8. Terence Tao: Analysis I & II (3/e) TRIM 37 & 38 Springer Sci-
ence+Business Media Singapore 2016; Hindustan book agency(2015)
ISBN 978-981-10-1789-6 (eBook) & ISBN 978-981-10-1804-6 (eBook)
9. Ajith Kumar & S Kumaresan : A Basic Course in Real Analysis
CRC Press, Taylor & Francis Group(2014) ISBN: 978-1-4822-1638-
7 (eBook - PDF)
10. Hugo D Junghenn : A Course in Real Analysis CRC Press, Taylor &
Francis Group(2015) ISBN: 978-1-4822-1928-9 (eBook - PDF)

You might also like